Superpage
Testis & paratestis


Copyright: 2002-2024, PathologyOutlines.com, Inc.

GU related: Jobs, Fellowships, Conferences, Cases, CME, Board Review

Related chapters: Penis & scrotum, Prostate gland & seminal vesicles

Editorial Board oversight: Debra L. Zynger, M.D. (last reviewed March 2024)
Page views in 2024 to date: 6

Adenocarcinoma
Definition / general
  • Rare malignant neoplasm originating from the epithelium of rete testis and associated with an aggressive clinical behavior
  • Given its rarity, varied histologic presentations and nonspecific immunoprofile, primary adenocarcinoma of collecting ducts and rete testis is a diagnosis of exclusion
Essential features
  • Primary adenocarcinoma of collecting ducts and rete testis is a rare aggressive neoplasm originating from the epithelium of rete testis
  • Diagnosis of exclusion, with the differential diagnosis including mesothelioma, metastatic adenocarcinoma and germ cell tumors, among other entities
  • On histopathologic examination, a mixture of different architectural patterns is commonly seen, such as complex tubulopapillary, glandular, solid and cribriform
  • Cytokeratin 7, cytokeratin AE1 / AE3 and epithelial membrane antigen are positive in the majority of the cases, whereas CD30, inhibin, OCT 3/4 and SALL4 are negative
Terminology
  • Not used anymore: papillary adenocarcinoma of the rete testis, carcinoma of the rete testis
ICD coding
  • ICD-O: 8140/3 - adenocarcinoma, NOS
  • ICD-11:
    • 2C84 - malignant neoplasms of other specified male genital organs
    • XH74S1 - adenocarcinoma, NOS
Epidemiology
  • Extremely rare, with fewer than 100 cases reported in the literature
  • More common in elderly Caucasian males (mean age of 54 years); however, anecdotal reports in young adults (Am J Surg Pathol 2021;45:1720)
Sites
Pathophysiology
  • Anecdotal cases with a history of adenomatous hyperplasia of the rete testis and subsequent development of adenocarcinoma, as well as cases with coexistent adenomatous hyperplasia and adenocarcinoma, have been reported (J Urol 1997;158:1525, Diagn Pathol 2013;8:105)
  • However, more studies are needed to definitively establish a stepwise progression from normal rete testis to adenomatous hyperplasia and subsequent occurrence of primary adenocarcinoma
Etiology
Clinical features
Diagnosis
  • Testicular, paratesticular and metastatic tumors need to be ruled out before arriving at a diagnosis of primary rete testis adenocarcinoma; as such, it represents a diagnosis of exclusion
  • Currently accepted diagnostic criteria for primary adenocarcinoma of the rete testis have been revised from Nochomovitz and Orenstein and include the following:
    • Testicular hilar localization
    • Absence of morphologically similar extrascrotal tumor as a possible primary site
    • Morphology incompatible with other types of testicular or paratesticular neoplasms
    • Supportive immunohistochemical findings and immunohistochemical exclusion of other entities, especially mesothelioma and papillary serous carcinoma (Am J Surg Pathol 2021;45:1720, Mod Pathol 2005;18:S131, Am J Surg Pathol 1984;8:625)
  • Demonstration of transition from normal to neoplastic rete epithelium is a desirable criterion but frequently not identifiable since the normal anatomy tends to be obliterated by the neoplastic process (Am J Surg Pathol 2021;45:1720)
  • Application of these strict criteria led to exclusion or reclassification of many cases that were initially reported as primary rete testis adenocarcinoma in the literature (Am J Surg Pathol 2019;43:670)
Laboratory
Radiology description
  • Rationale for imaging is threefold:
    • Exclusion of an extrascrotal tumor as a possible primary site
    • Visualization of the scrotal tumor
    • Staging
  • Ultrasound may show an ill defined, hypoechoic, heterogeneous mass and increased vascularization (Urol Case Rep 2017;16:72, Oncol Lett 2014;7:455)
  • Intrascrotal calcification may be present on radiographs (Urology 1995;46:468)
  • Potentially improved utility of CT / PET compared to conventional CT in detecting metastases (Urology 2011;77:334)
Prognostic factors
Case reports
Treatment
  • Treatment options for primary rete testis adenocarcinoma are not standardized
  • Treatment consists of radical orchiectomy with or without retroperitoneal lymph node dissection, in combination with adjuvant chemotherapy or radiotherapy
  • Resistance to conventional chemotherapy regimens is a frequent problem (Histopathology 2022;81:77)
Gross description
  • Epicenter of the mass at the testicular hilum, with variable involvement of the testicular parenchyma, spermatic cord or overlying skin by way of direct extension
  • Ill defined, firm tumors ranging from 2 - 13 cm in greatest dimension
  • Cut surface tan, yellow or white; solid to cystic, with focally hemorrhagic areas
  • References: Am J Surg Pathol 2019;43:670, Am J Surg Pathol 2021;45:1720
Gross images

Images hosted on other servers:
Mass in testicular hilum

Mass in testicular hilum

Solid mass with cystic areas

Solid mass with cystic areas

Microscopic (histologic) description
  • Invasive growth pattern with extensive intrarete proliferation and extension beyond the rete testis
  • Architecture: a mixture of architectural patterns among different cases and within the same case, including but not limited to (Am J Surg Pathol 2021;45:1720, Am J Surg Pathol 2019;43:670):
    • Complex tubulopapillary
    • Glandular
    • Solid
    • Cribriform
    • Glomeruloid
    • Retiform
    • Sertoliform
    • Kaposiform
    • Micropapillary
    • Nested
  • Cytomorphology (Histopathology 2022;81:77):
    • Epithelioid or biphasic (epithelioid and sarcomatoid) (J Nat Sci Biol Med 2015;6:211)
    • Cuboidal to columnar cells with marked nuclear atypia, pleomorphism and prominent nucleoli
    • High mitotic activity, with atypical mitoses
    • Cytoplasm varies from pale to eosinophilic
    • Desmoplastic stroma is a common finding
    • Psammomatous calcifications and tumor necrosis may be observed (Urology 1995;46:468)
  • Areas of gradual transition from normal rete testis lining to dysplastic and malignant epithelium, if present, lend support to a primary rete testis origin
Microscopic (histologic) images

Contributed by Rafael E. Jimenez, M.D.
Areas of central necrosis

Areas of central necrosis

Complex architecture

Complex architecture

Transition of epithelium Transition of epithelium Transition of epithelium

Transition of epithelium


Complex tubuloglandular pattern

Complex tubuloglandular pattern

Cribriform pattern

Cribriform pattern

High grade cytomorphology

High grade cytomorphology

Increased mitotic activity

Increased mitotic activity

Positive stains
Electron microscopy description
Molecular / cytogenetics description
  • Acosta et al. described genomic alterations in a series of 8 cases using massively parallel DNA sequencing (Histopathology 2022;81:77)
  • Pathogenic variants were noted in 6 cases:
    • 3 cases harbored CDKN2A variants
    • Authors also described variants in AKT1, RB1, NF2, SETD2 and TP53 genes
Sample pathology report
  • Testicle, left, orchiectomy:
    • Adenocarcinoma, involving the rete testis, most consistent with a primary rete testis adenocarcinoma, 2.5 cm (see comment)
    • Surgical margins negative for carcinoma
    • Comment: The sections show a high grade cellular proliferation with an invasive growth pattern, centered in and around the rete testis. A mixture of architectural patterns is noted, including complex tubulopapillary, glandular, cribriform and solid areas. Focally, a transition between the surface epithelium of the rete testis and the invasive component is appreciable. The cytomorphology is notable for cuboidal to columnar cells with marked nuclear atypia, pleomorphism and prominent nucleoli. Immunohistochemically, the tumor cells are reactive for BerEP4, CK7 and EMA and negative for CDX2, inhibin, NKX3.1, OCT 3/4, PSA and TTF1. These findings argue against a germ cell tumor or mesothelioma and favor a primary tumor over a metastatic process. Based on this constellation of findings, the most likely possibility is that this is a primary rete testis adenocarcinoma; however, a metastatic process cannot be completely ruled out and thus another primary process should be clinically excluded.
Differential diagnosis
  • Transition between normal and neoplastic rete testis epithelium, if present, favors a diagnosis of primary rete testis adenocarcinoma over the following entities in the differential diagnosis:
Board review style question #1

A 68 year old man presents with a 4.7 cm nodule centered around the testicular hilum, with invasion into the testicular parenchyma. Cut surface is heterogeneous, with solid, cystic and hemorrhagic areas. Histologic examination reveals a mixture of architectural patterns, including glandular, cribriform and solid areas. The cells exhibit nuclear pleomorphism, prominent nucleoli and high mitotic activity (see figures above). Clinicoradiologic examination fails to identify any extrascrotal tumors. Based on the constellation of findings, the man is diagnosed with primary adenocarcinoma of rete testis. Regarding this entity, which of the following statements is true?

  1. CK7, CK AE1 / AE3 and EMA are positive in the majority of the cases
  2. It frequently shows immunoreactivity for CD30, OCT 3/4 and SALL4
  3. Many patients are asymptomatic and these tumors are usually detected incidentally on imaging studies
  4. These tumors typically follow an indolent course, without metastases
  5. This entity is typically associated with increased levels of alpha fetoprotein and βhCG
Board review style answer #1
A. CK7, CK AE1 / AE3 and EMA are positive in the majority of the cases

Comment Here

Reference: Adenocarcinoma of collecting ducts and rete testis

Adenocarcinoma of epididymis
Definition / general
  • Adenocarcinoma arising de novo in epididymis
Terminology
  • Also called papillary adenocarcinoma, cystadenocarcinoma, papillary cystadenocarcinoma
Epidemiology
  • Extremely rare
  • Age range: 27 - 82 years, mean age: 67 years
Sites
  • Tumors are centered on epididymis; may be confined to epididymis or may infiltrate adjacent tissue
Pathophysiology
  • Believed to arise from metaplastic epididymal epithelium
Clinical features
  • Usually presents as palpable scrotal mass with / without pain and hydrocele
Diagnosis
  • Confirmation and differentiation from other malignancies in this region is possible only on histology
Laboratory
  • No specific laboratory features
  • Negative serum markers for sex cord and germ cell tumors
Prognostic factors
  • Due to rarity of tumor, prognostic features are unknown
  • A large proportion of tumors reported in literature metastasize at presentation or later
Case reports
Treatment
  • High inguinal orchidectomy with retroperitoneal lymph node dissection
  • Adjuvant chemotherapy or radiation therapy
Gross description
  • 1 - 7 cm
  • Infiltrative tumor either confined to epididymis or infiltrating adjacent soft tissue
  • Tan to grey white cut surface with frequent foci of hemorrhage and necrosis
Microscopic (histologic) description
  • Tubular, tubulopapillary, papillary and cystic growth patterns, which often occur in combinations and varying degrees of complexity
  • Also an undifferentiated sheet like pattern
  • Tumor cells are cuboidal to columnar, predominantly clear, rich in glycogen
  • Often infiltrates into epididymal smooth muscle or adjacent soft tissue
  • Necrosis is usually present
  • Psammoma bodies are rare
Microscopic (histologic) images

Images hosted on other servers:
Missing Image

Adenocarcinoma of rete testis

Positive stains
Negative stains
Differential diagnosis
  • Other primary tumors of epididymis and paratesticular region
    • Adenomatoid tumor
      • Usually lack clear cells
      • Tubules lined by low cuboidal to flat cells which may be vacuolated but are not watery clear as seen in primary epididymal adenocarcinoma
      • Low grade nuclear features
      • Mesothelial origin
      • IHC for mesothelial markers usually positive
    • Carcinoma of rete testis
      • Centered on testicular hilum, associated with transition to normal rete testis
      • Elongated tubular pattern, unlike the round tubular pattern of epididymal adenocarcinomas
    • Mesothelioma
      • No clear cells
      • Calretinin, HBME1, WT1, mesothelin positive
    • Serous cystadenoma of epididymis
      • Lack of invasive features
      • Low grade cellular features
      • Associated with von Hippel-Lindau syndrome
    • Serous papillary carcinoma: infiltrates as single isolated papillae, no tubules
  • Metastatic prostatic carcinoma: PSA+
  • Metastatic GI adenocarcinoma: CEA+, CK20+; variable mucin

Adenomatoid tumor
Definition / general
Essential features
  • Most common benign paratesticular tumor
  • Usually based in the epididymis and well circumscribed
  • Expresses markers typical of mesothelial origin
ICD coding
  • ICD-10: D29.30 - benign neoplasm of unspecified epididymis
Sites
  • Testicular adnexa
Pathophysiology
  • Not known
Etiology
  • Not known
Clinical features
Diagnosis
  • Primarily made histologically
Laboratory
  • Testicular tumor markers: normal
Radiology description
  • Circumscribed, solid paratesticular mass
  • Unable to distinguish benign versus malignant on sonography (Radiology 1997;204:43)
  • Tumors of the tunica may be indistinguishable from a germ cell tumor by imaging
Radiology images

Images hosted on other servers:

Coronal MR

Ultrasound

Prognostic factors
  • Benign neoplasm
Case reports
Treatment
  • Surgical excision
  • Intraoperative frozen section may be helpful to indicate testis sparing surgery
Gross description
Gross images

Contributed by Debra L. Zynger, M.D.

Paratesticular tumor



Images hosted on other servers:

Well circumscribed tumor

Frozen section description
  • May be confused for malignant mesothelioma on biopsy; gross correlation often needed
  • Can have atrophic testes with fibrous stromal proliferation (Urology 2004;63:779)
Frozen section images

Contributed by Debra L. Zynger, M.D.

Distorted, compressed tubules

Spindle cell appearance Spindle cell appearance

Spindle cell appearance


Microscopic (histologic) description
Microscopic (histologic) images

Contributed by Sarah Findeis, M.D., Stephanie J. Conrad, M.D., Ming Zhou, M.D.,
the Genitourinary Pathology Society (GUPS) and @katcollmd on Twitter

Interface with testis

Cords and trabeculae pattern

Eosinophilic cells

Calretinin

CK AE1/3

Circumscribed tumor


Dilated cystic spaces

Vacuolated and attenuated epithelium

Bland nuclei

Calretinin

Adenomatoid tumor Adenomatoid tumor

Adenomatoid tumor


Adenomatoid tumor Adenomatoid tumor

Adenomatoid tumor

Adenomatoid tumor Adenomatoid tumor

Adenomatoid tumor

Adenomatoid tumor Adenomatoid tumor

Adenomatoid tumor

Virtual slides

Images hosted on other servers:

Adenomatoid tumor

Cytology description
  • Epithelioid, monolayered sheets or clusters of cells with eccentric, vesicular round to ovoid nuclei, granular chromatin and pale / vacuolated cytoplasm (J Cytol 2009;26:30, J Cytol 2013;30:65)
  • Can have a background of naked nuclei as well as stromal cells (Acta Cytol 1989;33:6)
Cytology images

Images hosted on other servers:

Moderate cellularity

Monolayered sheets of cells

Negative stains
Electron microscopy description
  • Rich in cytoplasmic organelles and has superficial desmosomes with microvilli (Cancer 1972;30:244)
Molecular / cytogenetics description
Sample pathology report
  • Testicle and spermatic cord, right, radical orchiectomy:
    • Paratesticular adenomatoid tumor, 1.5 cm
    • Margins widely free of tumor (> 5.0 cm)
    • Unremarkable background testis, epididymis and spermatic cord
    • Comment: The tumor is positive for AE1/3, WT1 and calretinin and is negative for SALL4 and inhibin.
Differential diagnosis
Board review style question #1

    Which is true regarding this paratesticular entity which does not involve the testicular parenchyma?

  1. Exclusively identified within men
  2. Malignant transformation occurs in 50% of cases
  3. Most common in the sixth to seventh decade
  4. Rarely identified within the testicular parenchyma
Board review style answer #1
D. These tumors are rarely identified within the testicular parenchyma. This is an adenomatoid tumor. They can be identified within both sexes. It occurs in the paratesticular region most commonly in the third to fourth decade. The tumor rarely involves the testicular parenchyma. If there is only intraparenchymal involvement, other primary parenchymal lesions need to be ruled out, such as a sex cord stromal tumors or germ cell tumors.

Comment Here

Reference: Adenomatoid tumor
Board review style question #2

    A 36 year old man presents to his physician with a painless testicular mass. Imaging showed a well circumscribed solid mass arising from the tunica albuginea and not involving the testicular parenchyma. An orchiectomy is performed which shows a 2.0 cm unencapsulated white-tan circumscribed nodule. Histologic examination shows cords and tubules of cuboidal eosinophilic cells with vacuolated cytoplasm in the background of fibrous stroma, which infiltrates into the surrounding testicular tubules. What is the most likely staining pattern for this lesion?

  1. Calretinin +, EMA +, BAP 1+
  2. CK AE1/3 -, CD31 +, CD15 +
  3. OCT3/4 +, CEA+, S100+
  4. Vimentin -, CD99 +, calretinin -
Board review style answer #2
A. Calretinin +, EMA +, BAP 1+. This is an adenomatoid tumor, which is of mesothelial origin as the staining pattern suggests. The patient presentation of a small painless mass in a patient in the third - fourth decade is typical. Adenomatoid tumors rarely involve the testicular parenchyma and are the most common paratesticular tumor. Microscopic infiltration may be misinterpreted as invasion but may occur and the tumor still has a benign course.

Comment Here

Reference: Adenomatoid tumor
Board review style question #3
    Which of the immunohistochemical stains, when positive, supports the diagnosis of adenomatoid tumor?

  1. AFP
  2. B72.3
  3. Calretinin
  4. CD34
  5. Inhibin
Board review style answer #3
C. Calretinin

Comment Here

Reference: Adenomatoid tumor 🧬

Adrenal cortical rests
Definition / general
  • Occur in 4 - 15% of testis, paratesticular tissue, spermatic cord or epididymis
  • For additional information, refer to ectopic adrenal tissue
Gross description
  • Small, round, yellow nodule
Microscopic (histologic) description
  • Circumscribed nodule of adrenal cortical tissue without medullary tissue
Microscopic (histologic) images

Contributed by Yuto Yamazaki, M.D., Ph.D., Hironobu Sasano, M.D., Ph.D., Debra L. Zynger, M.D. and Sean R. Williamson, M.D.
Adrenal rest Nests of cortical cells SF1

Spermatic cord adrenal rest and SF1


Radical orchiectomies / spermatic cords with adrenal rests Radical orchiectomies / spermatic cords with adrenal rests Radical orchiectomies / spermatic cords with adrenal rests

Spermatic cord adrenal rests


Radical orchiectomies / spermatic cords with adrenal rests Radical orchiectomies / spermatic cords with adrenal rests

Paratesticular adrenal cortical rest


Adult granulosa cell tumor
Definition / general
  • Sex cord stromal tumor resembling granulosa cells of the Graafian follicles of the ovary
Essential features
  • Rare, low grade, indolent sex cord stromal tumor; 10% have malignant potential
  • Most common architectural patterns are diffuse / solid, insular and spindled; pathognomonic Call-Exner bodies, which are a part of the microfollicular pattern, are less common
  • Morphologically, the tumor cells characteristically display uniformly round to ovoid nuclei with conspicuous, longitudinal nuclear grooves
  • FOXL2 gene mutations occur in a minor subset of cases, which is substantially less frequent than in its ovarian counterpart
ICD coding
  • ICD-O: 8620/1 - Granulosa cell, adult type
  • ICD-O: 8620/3 - Granulosa cell, malignant
  • ICD-10: D40.1 - Neoplasm of uncertain behavior of unspecified testis
  • ICD-10: C62.9 - Malignant neoplasm of testis, unspecified whether descended or undescended
Epidemiology
Sites
  • Testicular parenchyma
Pathophysiology
  • Derived from granulosa cells in which FOXL2, a granulosa cell expressed gene, plays a role in regulating cell fate
  • Missense mutation (402C → G, C134W) causes a negative effect by dysregulating gene expression, leading to increased cell proliferation and reducing apoptosis (Oncol Lett 2020;20:293)
Etiology
  • Unknown
Clinical features
Diagnosis
  • Imaging studies (e.g. ultrasound) may identify a testicular mass
  • Definitive diagnosis is made by histopathologic examination only
Laboratory
Radiology description
  • Nonspecific findings but typically shows a well demarcated, hypoechoic mass with hyperechoic foci and calcification (Radiol Case Rep 2015;3:188)
Radiology images

Images hosted on other servers:
Hypoechoic lesion

Hypoechoic lesion

Hypoechoic lesion and increased vascularity

Hypoechoic lesion /
increased vascularity

Increased vascularity

Increased vascularity

Prognostic factors
Case reports
Treatment
Gross description
  • Mean size: 2.8 cm (0.5 - 6.0 cm) (Am J Surg Pathol 2014;38:1242)
  • Well circumscribed, predominantly solid; occasionally solid and cystic
  • Lobulated, yellow-tan or tan-white cut surface
  • Infiltrative borders / extratesticular involvement, hemorrhage and necrosis are rarely seen and often associated with more aggressive behavior
Gross images

Contributed by Kristine M. Cornejo, M.D.
Tan-yellow solid mass

Tan-yellow solid mass

Lobulated tan-yellow mass

Lobulated tan-yellow mass

Microscopic (histologic) description
  • Low power: typically displays a nodular / lobulated appearance
  • High power: tumor cells contain uniformly round to ovoid nuclei with conspicuous longitudinal nuclear grooves, indistinct cell borders, variable amounts of eosinophilic cytoplasm
  • Most common patterns: diffuse / solid, insular and spindled (Am J Surg Pathol 2014;38:1242)
    • Diffuse / solid: sheets of round cells
    • Insular: nests of tumor cells surrounded by stroma
    • Spindled: sheets of fusiform cells
    • Cords / trabecular: regularly to irregularly arranged thin cords or thick columns of tumor cells
    • Water silk / gyriform: interweaving cords and undulating columns of cells
    • Microfollicular: numerous small spaces with hyalinized basement membrane material or eosinophilic fluid (Call-Exner bodies)
    • Herringbone / fascicular: spindled cells with fibrocollagenous stroma
    • Pseudopapillary: tumor cells surrounding fibrovascular cores
    • Palisading: tumor cells arranged parallel to each other, resembling a fence
    • Luteinized: more abundant clear to eosinophilic cytoplasm with rounded nuclei
  • Admixed with varying amounts of fibrocollagenous or edematous stroma
  • Mitotic count: mean 4.9/10 high power fields (0 - 18/10 high power fields); suggests the presence of mitoses are not of prognostic significance (Am J Surg Pathol 2014;38:1242)
  • May contain hemorrhage or hemosiderin deposition
  • Rarely displays necrosis and lymphovascular invasion; features of aggressive behavior
  • Tumor often infiltrates between seminiferous tubules and may involve rete testis and tunica albuginea
    • Rarely involves paratesticular or extratesticular tissue
Microscopic (histologic) images

Contributed by Kristine M. Cornejo, M.D.
Nodular appearance

Nodular appearance

Nuclear grooves

Nuclear grooves

Diffuse pattern

Diffuse pattern

Spindled pattern

Spindled pattern

Corded pattern

Corded pattern

Gyriform / watered-silk pattern

Gyriform / watered silk pattern


Microfollicular pattern (Call-Exner Bodies) Microfollicular pattern (Call-Exner Bodies) Microfollicular pattern (Call-Exner Bodies) Microfollicular pattern (Call-Exner Bodies)

Microfollicular pattern (Call-Exner bodies)

Insular pattern

Insular pattern

Palisading pattern

Palisading pattern


Pseudopapillary pattern

Pseudopapillary pattern

Herringbone / fascicular pattern

Herringbone / fascicular pattern

Leutinized cells

Luteinized cells

Inhibin A stain

Inhibin A stain

Positive stains
Negative stains
Molecular / cytogenetics description
Sample pathology report
  • Left testis, radical orchiectomy:
    • Adult granulosa cell tumor (3.0 cm), confined to the testis (see comment)
    • Comment: The tumor is small, well circumscribed and lacks features associated with a more aggressive behavior, such as significant cytologic atypia, conspicuous mitoses, presence of lymphovascular invasion or necrosis.
Differential diagnosis
  • Juvenile granulosa cell tumor:
    • Typically occurs in the first 6 months of life; rare after 1 year of age
    • Associated with ambiguous external genitalia and karyotypes
    • Most common patterns are solid and follicular, in which the follicles are irregular in size and shape lined by granulosa cells with an outer thecal layer; contains mucinous material that is not found in adult granulosa cell tumor and the follicles tend to be more uniform
    • Usually lacks Call-Exner bodies and nuclear grooves
  • Leydig cell tumor:
    • Presents at a similar age (mean: 46.5 years)
    • Similarly presents with sheets or lobular aggregates of cells with abundant eosinophilic cytoplasm arranged with variable amounts of fibrous stroma
    • Cells may be vacuolated and contain Reinke crystals, which are not seen in adult granulosa cell tumors
  • Sertoli cell tumor:
    • Presents at a similar age (mean: 45 years)
    • Displays a variety of patterns, such as trabecular, corded and diffuse, in addition to varying amounts of well formed tubules; the latter is typically not seen in adult granulosa cell tumors and lacks the typical microfollicular pattern or Call-Exner bodies
    • Composed of tumor cells with abundant pale to lightly eosinophilic cytoplasm, which may display nuclear grooves
    • Positive for nuclear beta catenin (Am J Surg Pathol 2015;39:1390)
  • Fibrothecoma:
    • Presents at a similar age (mean: 44 years)
    • Composed of ovoid or spindled cells in varying amounts of collagenous stroma
    • Lacks varied architectural patterns and nuclear grooves
    • Inhibin A positive (patchy to diffuse pattern) (Am J Surg Pathol 2013;37:1208)
  • Sex cord stromal tumor, unclassified:
    • Tumors that have features of indeterminate differentiation; lack findings characteristic of adult granulosa cell tumors or other sex cord stromal tumors
  • Sex cord stromal tumor, mixed:
    • Tumors that contain characteristics of 2 or more tumor subtypes
Board review style question #1

A 47 year old man presented with a right testicular swelling. Histologic examination revealed the tumor shown above. This tumor typically shows which of the following immunoprofiles?

  1. OCT4+ / SALL4+ / inhibin A- / FOX2L-
  2. OCT4- / SALL4+ / inhibin A+ / FOX2L-
  3. OCT4- / SALL4- / inhibin A+ / FOX2L+
  4. OCT4- / SALL4- / inhibin A- / FOX2L-
Board review style answer #1
C. OCT4- / SALL4- / inhibin A+ / FOX2L+. Histology shows Call-Exner bodies, which are pathognomonic for adult granulosa cell tumor. Adult granulosa cell tumor expresses general sex cord stromal tumor immunohistochemical markers, such as inhibin A. FOX2L has also been found to be a marker of sex cord stromal tumors and does not necessarily correlate with a FOX2L gene mutation, which has been identified in a small subset of testicular adult granulosa cell tumors. OCT4 and SALL4 are markers of germ cell tumors.

Comment Here

Reference: Adult granulosa cell

Anatomy & histology
Definition / general
  • Birth to age 4 (static phase): seminiferous tubules filled with small cuboidal cells with no definite lumen present, Leydig cells usually not visible
  • Age 4 - 10 (growth phase): tubules, tubular lumina and cells enlarge; tubules become tortuous
  • Age 10 to puberty (maturation phase): tubular cells have mitoses; Leydig cells prominent; spermatocyte differentiation visible
  • Adult (postpuberty): each testis weights 15 - 19g, measures 5 × 3 cm; takes 70 days for cells to mature from spermatogonium to primary spermatocyte to secondary spermatocyte to spermatid to spermatozoa; maturation is orderly along length of tubule but often not present in biopsy cross section
  • Normal spermatogenesis:
    • Not every tubule has complete spermatogenesis
    • Number of late spermatids correlates best with sperm counts
    • Some patients have normal sperm counts but low motility or duct obstruction

  • Testes is paired organ suspended in scrotum by spermatic cord
  • Each testis is attached to an epididymis, which connects rete testis to vas deferens
  • Testis is composed of convoluted seminiferous tubules in a stroma with Leydig cells
  • Three layers: outer serosa (tunica vaginalis, extension of peritoneal cavity) with mesothelial cells; tunica albuginea (tough fibrous septa that extends into testis and separates it into 250 lobules of 1 - 4 seminiferous tubules), inner tunica vasculosa
  • Seminiferous tubules converge into rete testis at hilum, anastomose into efferent tubules that penetrate tunica albuginea to form head of epididymis
Terminology
  • Efferent ducts: see micro images below
  • Epididymis:
    • Connects efferent ductules to vas deferens
    • Has head, body and tail
    • Composed of columnar cells (tall, ciliated with PAS+ nuclear inclusions), clear cells and basal contractile cells (actin positive)
    • May have "monster" cells similar to seminal vesicle (no significance, Am J Surg Pathol 1981;5:483)
    • Tubules have thick muscular coat
    • References: Am J Surg Pathol 2003;27:469
    • Nonpathologic morphologic variations:
      • Intranuclear eosinophilic inclusions: 72%, usually older patients
      • Lipofuscin pigment: 33% usually in efferent ducts and associated with obstructive changes
      • Cribriform hyperplasia: 42% usually NOT in normal testis
      • Paneth cell-like metaplasia: 8% with hyalin-like globules that are positive for PAS with and without diastase digestion, associated with obstructive changes
      • Nuclear atypia: 14% similar to that in seminal vesicles, associated with older age
      • Note: rarely present within hernia sacs (Am J Surg Pathol 1999;23:880)
      • References: Am J Surg Pathol 1998;22:990
  • Interstitium: contains Leydig cells and stromal elements (collagen and myoid cells that surround seminiferous tubules)
  • Leydig cells:
    • Single (20 microns) or in clusters between seminiferous tubules, produce testosterone in response to luteinizing hormone (LH)
    • Often associated with nerve fibers and blood vessels
    • Have abundant pink cytoplasm with lipid, lipochrome pigment, Reinke crystalloids (hexagonal prisms by EM), round nuclei with distinct nucleoli
    • Fewer Leydig cells in elderly
  • Mediastinum: posterior testicular capsule with vasculature, nerves, mediastinum of rete testis (where tubules converge)
  • Rete testis:
    • At testicular hilum
    • Complex tubular architecture may resemble teratoma
    • Connects testicular tubules to 12 - 15 ciliated efferent ducts, which merge into a single duct, the epididymis at its head
    • Rete lined by flattened to columnar epithelium with numerous microvilli
    • Efferent duct lumina are narrower than epididymis, lined by ciliated columnar cells with microvilli
  • Seminiferous tubules:
    • 150 - 250 microns in diameter, the average total length in each testes is 540 m (range 299 - 981 m)
    • Lined by multilayered epithelium with most mature cells towards lumina
    • Have basal lamina, outer myoid cells (positive for desmin, muscle specific actin, vimentin) and collagen
    • Contain Sertoli cells, spermatogonia (types A and B), primary spermatocytes, secondary spermatocytes, spermatids and spermatozoa
    • All except spermatozoa are held together by a narrow cytoplasmic bridge
    • Immature tubules are positive for alpha inhibin
  • Sertoli cells:
    • Columnar, on basement membrane, surround germ cell elements with cytoplasmic extensions, form blood - testis barrier
    • 7% of tubular cells
    • May contain Charcot-Bottcher crystalloids (bundles of microfilaments)
    • Have irregular, highly folded nuclei with prominent nucleoli
    • Produce anti-Müllerian hormone, which causes regression of Müllerian duct structures in utero
    • After birth, secrete androgen binding protein and are responsive to FSH
    • Also produce inhibin
  • Vas deferens:
    • Also called ductus deferens
    • 30 - 40 cm long tubular structure from tail of epididymis to prostatic urethra at level of verumontanum; distal vas deferens joins seminal vesicle to form ejaculatory duct
    • Rarely present within hernia sacs (Am J Surg Pathol 1999;23:880)
    • Should see complete transection in vasectomy specimens
    • References: Am J Surg Pathol 2003;27:469

Vestigial remnants:
  • Appendix epididymis: remnant of mesonephric duct
  • Appendix testis (hydatid of Morgagni):
    • Remnant of Müllerian duct; attached to tunica albuginea at upper testicular pole; present in 90% of males
    • May undergo hemorrhagic infarction from twisting on its pedicle
    • Gross: round / oval, 1 - 10 mm, pedunculated
    • Micro: columnar epithelium with vascular fibrous core with smooth muscle cells; may have glandular-like structures due to surface invaginations
  • Paradidymis (organ of Giraldes): remnant of mesonephric tubules
  • Vas aberrans (organ of Haller): remnant of mesonephric tubules
Embryology
  • Presence of Y chromosome (possibly sex determining region Y) determines formation of testis
  • Germ cells migrate to genital ridge, tubules formed by day 45; Wolffian ducts (form epididymis, vas deferens, seminal vesicles and ejaculatory ducts) develop by day 25, Müllerian ducts (form uterus and upper vagina) by day 43
  • If no Y, gonad becomes an ovary; if Y present becomes a testis
  • If Y present, Sertoli cells develop from genital ridge and secrete anti-Müllerian hormone / Müllerian inhibiting factor (AMH) by day 62, causing ipsilateral regression of Müllerian duct by day 75 - 80; lack of Sertoli cells means no AMH, no regression and presence of uterus and upper vagina
  • If Y present, Leydig cells arise by day 64 and produce testosterone, which causes development of Wolffian duct structures (epididymis, vas deferens, seminal vesicles and ejaculatory ducts) if functional androgen receptors are also present
  • If no testosterone is produced, Wolffian duct structures degenerate
  • Exogenous androgens or their production by maternal tumor or congenital adrenal hyperplasia cause development of Wolffian duct system regardless of presence of Y chromosome
  • Development of external genitalia requires testosterone plus androgen receptors plus 5 alpha reductase, which converts testosterone to dihydrotestosterone (DHT), which causes development of external genitalia between days 120 - 140, including elongation of phallus
  • If ovaries or no gonads present, internal ducts are female
  • If DHT not present, external genitalia is female
Diagrams / tables

Images hosted on other servers:

Scrotum

Transverse section

Right testis

Vertical section

Transverse section


Section of genital cord

Spermatic cord

Subcutaneous inguinal ring

Cremaster

Spermatic veins

Section of epididymis of guinea pig


Fundus of bladder

Gross images

Contributed by @Andrew_Fltv on Twitter
Testis anatomy Testis anatomy

Testis anatomy



Images hosted on other servers:

Normal testis

Microscopic (histologic) description
  • Spermatic cord:
    • Ciliated, pseudostratified epithelium with prominent nuclear inclusions resting on basal cell layer
    • Markedly thickened muscle coat of 3 layers (inner longitudinal, outer longitudinal, middle circular layer)
    • May contain psammoma bodies
Microscopic (histologic) images

Contributed by Asmaa Gaber Abdou, M.D. and @Andrew_Fltv on Twitter
Testis histology Testis histology

Testis histology



Images hosted on other servers:

Normal testis

Efferent ducts

Leydig cells


Seminiferous tubules


Various images of epididymis


Various images of spermatic cord

Positive stains
  • Epididymis and spermatic cord: CD10
Electron microscopy description
  • Spermatogenesis: EM may show round headed sperm or immotile cilia syndrome

Atrophy
Definition / general
  • Testicular atrophy is a nonneoplastic process characterized by the disappearance of tubular or germinal epithelium and replacement with variable degrees of fibrosis
  • Atrophy can present in the setting of testicular regression syndrome, postvasectomy, vascular accidents, testicular torsion, cryptorchidism, infectious process, external insults, iatrogenic and other settings
Essential features
  • Etiology of testicular atrophy is varied; it can be seen in a variety of clinical pathologic settings
  • Microscopic features include small tubules with few or no viable germ cells and a relative predominance of Leydig cells
  • Potential risk for germ cell neoplasia in testicular regression syndrome with retained viable germ cells
Terminology
ICD coding
  • ICD-10: N50.0 - atrophy of testis
  • ICD-11: GB03 - atrophy of testis
Epidemiology
  • Newborn / children for testicular regression syndrome
  • Any age for other etiologies
Sites
  • Testicle parenchyma
Pathophysiology
  • In testicular regression syndrome, absence of a testis in an otherwise normal 46,XY male is usually unilateral and is assumed to be a consequence of intrauterine or perinatal torsion or infarction (J Clin Res Pediatr Endocrinol 2012;4:116)
  • Hypothalamic - pituitary - gonadal axis dysregulation, leading to reduced steroidogenesis and spermatogenesis (Reprod Sci 2021;28:2735)
  • Compromised vascular supply after a vascular accident or testicular torsion, leading to atrophy
Etiology
Clinical features
  • Unilateral or bilateral nonpalpable testis
  • Normal external genitalia
  • Infertility
  • Presence of micropenis in a subset of patients with testicular regression syndrome (J Clin Res Pediatr Endocrinol 2012;4:116)
  • Variable degrees of sexual ambiguity in familial cases of testicular regression syndrome
Diagnosis
  • Measurement of gonadotropin and testosterone levels, human chorionic gonadotropin stimulation test, measurement of anti-Müllerian hormone, inhibin B levels, INSL3 levels and karyotype analysis
  • Scrotal and abdominal ultrasound
  • Laparoscopic exploration to visualize blind ending spermatic vessels within the retroperitoneum, or visualization of spermatic vessels and vas deferens exiting a closed internal inguinal ring (J Clin Res Pediatr Endocrinol 2012;4:116)
Laboratory
  • In bilateral testicular regression syndrome, testosterone, inhibin B and INSL3 are undetectable, gonadotropins are elevated and karyotype is 46,XY (Best Pract Res Clin Endocrinol Metab 2022;36:101619)
  • Hypothalamic - pituitary - gonadal axis dysregulation causes impaired LH:FSH ratio and decreased testosterone
  • Lower sperm concentration, lower total sperm count and motility
  • Azoospermia
Radiology description
  • Absence or decreased size of testicle in scrotum
Prognostic factors
  • Potential risk for germ cell neoplasia in testicular regression syndrome with retained viable germ cells
Case reports
Treatment
  • Surgical removal of the testicular remnant may or may not be necessary (Front Pediatr 2022;10:834083)
  • Procedure for removal of outflow obstruction, if fertility is desired
  • Hormonal supplementation in cases of hypothalamic - pituitary - gonadal axis dysregulation
  • Treatment of underlying infection
Gross description
  • Spermatic cord with a small mass of firm, fibrotic tissue at one end as well as elements of the vas deferens, spermatic artery and venous plexuses (J Clin Res Pediatr Endocrinol 2012;4:116)
  • Small, fibrotic testis
Gross images

Contributed by @Andrew_Fltv on Twitter
Testicular atrophy

Testicular atrophy



Images hosted on other servers:

Comparison to normal testis

Microscopic (histologic) description
  • Fibrovascular nodule with hemosiderin laden macrophages and dystrophic calcifications
  • Rare presence of seminiferous tubules (6.3% of cases) and viable germ cells (2.1% of cases) in testicular remnants in testicular regression syndrome (Front Pediatr 2022;10:834083)
  • Small tubules, thick basement membrane and few or no germ cells in postvasectomy specimens
  • Sclerotic tubules
  • Interstitial fibrosis
  • Increased Leydig cells
Microscopic (histologic) images

Contributed by Patricija Zot, M.D., Rafael E. Jimenez, M.D., @ThatGlassTho on Twitter and @Andrew_Fltv on Twitter

Seminiferous tubules replaced by collagen deposition

Retained germ cells

Atrophy Atrophy

Atrophy


Testicular atrophy Testicular atrophy Testicular atrophy

Testicular atrophy

Positive stains
  • Inhibin and SF1 may highlight residual Sertoli and Leydig cells
  • CD117 may highlight residual germ cells
Negative stains
Molecular / cytogenetics description
  • DHX37 pathogenic variants are a new cause of an autosomal dominant form of 46,XY disorders of sex development, including gonadal dysgenesis and testicular regression syndrome (Genet Med 2020;22:150)
  • Heterozygous partial loss of function mutations in SF1 may be associated with bilateral anorchia (vanishing testis syndrome) and micropenis in humans (Hum Reprod 2007;22:3255)
Sample pathology report
  • Left testicle, orchiectomy:
    • Testicular parenchyma with seminiferous tubule fibrosis
Differential diagnosis
  • Regressed germ cell tumor:
    • Discrete scar; not a diffuse process
    • Germ cell neoplasia frequently present in the background (positive for SALL4, OCT 3/4, CD117, PLAP, D2-40)
    • Abnormal serum tumor markers
    • May present with metastatic disease
  • Leydig cell tumor:
    • Unifocal
    • Well circumscribed mass
    • May have infiltrative borders in malignant variant
    • Reinke crystals may be present
Board review style question #1

A 35 year old man presents for evaluation of infertility. After several laboratory and radiologic tests, a biopsy of a testicle was performed. The biopsy is shown in the image above. Presence or absence of which of the following would support a diagnosis of testicular atrophy?

  1. Absence of germ cell neoplasia in situ
  2. A nodule composed of cells containing Reinke crystals
  3. Presence of isochromosome 12p
  4. Presence of necrosis
Board review style answer #1
A. Absence of germ cell neoplasia in situ

Comment Here

Reference: Testicular atrophy
Board review style question #2
Parents of a 6 month old baby boy bring the child in for evaluation due to a concern for a nonpalpable testicle. On physical exam, there is only one testicle that has descended and is palpable in the scrotum. Imaging studies show no evidence of the second testicle in the abdomen or any other location; however, spermatic cord with a small nodule is present. The nodule is biopsied and shows benign fibrovascular tissue with no evidence of germ cells. What is the most likely diagnosis?

  1. Cryptorchidism
  2. Regressed mixed germ cell tumor
  3. Teratoma
  4. Testicular regression syndrome
Board review style answer #2
D. Testicular regression syndrome

Comment Here

Reference: Testicular atrophy

Biopsy
Definition / general
  • Biopsy indicated for azoospermia (no sperm present) without endocrine abnormalities, since biopsies may show focal spermatogenesis
  • Most biopsies have 3 - 5 lobules plus septa of tunica albuginea
  • Usually see many spermatozoa, "perfect" tubules mixed with occasional tubules with scattered "disorganized" spermatogenesis
  • In young men, ratio of germ cells to Sertoli cells is usually 13:1
  • Multinucleated giant stromal cells are associated with testicular atrophy due to estrogens
  • Hyalinized tubules with elastic fibers indicate hyalinization developed postpuberty
  • Zenker's and Bouin's fixative are preferred, as formalin introduces significant artifact
Biopsy Results for Azoospermia (No Sperm Present, No Endocrine Abnormalities)
  1. Germ cell aplasia / Sertoli cell only syndrome (29%): tubular basement membrane thickening, no germ cells, usually normal number of Leydig cells
  2. Spermatocytic arrest (26%): usually early arrest at primary spermatocyte level (no spermatids, no spermatozoa), normal Leydig cells; may be late arrest (no spermatozoa only)
  3. Generalized fibrosis (18%)
  4. Normal (27%): usually associated with bilateral obstruction as seen in Young syndrome (also chronic sinopulmonary infections) or testicular blockage (50% of tubules lack lumina; disorderly maturation of germ cells); surgery (epididymovasotomy, vasovasotomy) is often successful

Azoospermia due to known obstruction:
  • Severity related to cause / span of obstruction, usually reduction in spermatids only due to increased hydrostatic pressure (47%)
  • Also can be normal testes (28%), reduction of primary spermatocytes and spermatids (9%), reduction in all germ cell types (13%) or hyalinization (2% [Am J Surg Pathol 1999;23:1546])
Biopsy Results for No Sperm Count Due to Endocrine Abnormalities (Testicular Failure)
  1. Hypogonadotrophic eunuchoidism (60%): low FSH and LH levels; small infantile tubules with few or no Leydig cells, scattered spermatogonia and Sertoli cells
  2. Klinefelter syndrome (30%): XXY karyotype, tubular fibrosis, prominent basement membrane thickening and Leydig cell hyperplasia; associated with increased incidence of breast carcinoma, possibly Leydig cell tumors, extragonadal (mainly mediastinal) germ cell tumors
  3. Testicular aplasia (10%): absence of testicular tissue, high urinary LH and FSH
Biopsy Results for Oligospermia
  1. Incomplete spermatocytic arrest: some tubules normal, some with arrest
  2. Regional or incomplete fibrosis
  3. Spermatogenic hypoplasia: tubules with reduced number of germ cells that are also disordered
  4. Tubular hyalinization: includes Kleinfelter syndrome (small diameter tubules with thickened basement membrane and increased Leydig cells)
  5. Mixed atrophy: synchronous occurrence of seminiferous tubules with germ cells and tubules with Sertoli cells only
  6. Normal spermatogenesis: associated with duct obstruction

  • Sloughing and disorganization: lumina contain desquamated immature cells, disordered spermatogenesis (nonspecific, can be seen in hypoplasia, duct obstruction or mechanical damage / artifact to specimen)
  • Other causes of oligospermia: varicocele, cystic fibrosis causing obstruction in epididymis and vas deferens
Fine Needle Aspiration (FNA)
  • Used to differentiate normal testis, hypospermatogenesis, early and late maturation arrest, Sertoli cell only patterns
  • Minimally invasive, may replace testis biopsy
  • 2 - 11 aspiration sites, 10 - 30 needle excursions / site; adequate if 100 clusters of 20 or more testis cells
  • May be more representative than biopsy since more sampling
  • Interpretation should focus on relative abundance of germ cells (primary spermatocytes, spermatids, mature sperm with tail) and Sertoli (support) cells
  • Frequency of clinical diagnoses: idiopathic nonobstructive (50%), varicocele (18%), genetic (8%), cryptorchidism (8%), cancer (8%), obstruction (7%), Kartageners syndrome (1%)
  • Hypospermatogenesis: presence but paucity of all germ cell types
  • Sertoli cell only: no primary spermatocytes, no spermatids, no spermatozoa
  • Early maturation arrest: primary spermatocytes but no spermatids, no spermatozoa
  • Late maturation arrest: all cells but spermatozoa
  • Note: FNA cannot assess basement membrane, which is important for neoplasm and CIS diagnoses
  • References: Am J Surg Pathol 2001;25:71

Brenner tumor
Definition / general
  • Extremely rare tumor of testis and paratesticular regions composed of transitional type epithelium; usually occurs in ovary
  • Also called transitional cell tumor
Epidemiology
  • Age range 37 - 70 years (mean 57.7)
Pathophysiology
  • Etiology unknown; may originate from Walthard rests within tunica vaginalis or transitional epithelial nests located in testicular / paratesticular locations
Clinical features
  • Testicular mass, testicular pain
  • Must rule out metastatic urothelial carcinoma
Radiology description
  • Ultrasound remains imaging modality of choice for evaluation of scrotal masses
Prognostic factors
Case reports
Treatment
  • Orchiectomy
Gross description
  • Solid and cystic masses, 1 to 5 cm
Microscopic (histologic) description
  • Nests of transitional cells (solid or cystic) with fibrous or cellular spindle cell stroma
  • Nests show mostly a pavement type arrangement with some focal columnar type cell appearance
  • Nuclei are round to oval, some with nucleoli and longitudinal nuclear grooves
  • Focal mucinous differentiation has been reported
Positive stains
  • CK7: cytoplasmic staining
  • Uroplakin III: cytoplasmic and membrane staining
  • WT1: focal; nuclear staining
Negative stains
Electron microscopy description
  • Cells with complex interdigitations interlocking their plasma membranes, which are otherwise joined by tight desmosomes
  • Small intercellular spaces containing elongated projections of cytoplasm
  • Cytoplasm contains microfilaments, poorly preserved mitochondria and ribosomes
  • Deep nuclear clefts (Cancer 1979;43:539)

Choriocarcinoma
Definition / general
  • Choriocarcinoma is a malignant nonseminomatous germ cell tumor (GCT) composed of trophoblastic cells with associated hemorrhage
Essential features
  • Tumor that arises from trophoblastic tissue and consists of syncytiotrophoblasts, cytotrophoblasts and intermediate trophoblasts with associated hemorrhage
  • Increased serum beta hCG
  • Early hematogenous metastasis
  • Mixed germ cell tumor with choriocarcinoma component has better outcome than pure choriocarcinoma
Terminology
  • Nonseminomatous germ cell tumor (NSGCT)
  • Trophoblastic malignant teratoma, chorionepithelioma (not recommended)
ICD coding
  • ICD-O:
    • 9100/3 - choriocarcinoma, NOS
    • 9101/3 - choriocarcinoma combined with other germ cell elements
  • ICD-10:
    • C62.90 - malignant neoplasm of unspecified testis, unspecified whether descended or undescended
  • ICD-11:
    • 2C80.2 - germ cell tumor of testis
    • XH3WM1 - choriocarcinoma combined with other germ cell elements
    • XH8PK7 - choriocarcinoma, NOS
Epidemiology
Sites
Pathophysiology
  • Unknown
Etiology
  • No specific known cause
  • Precursor lesion: germ cell neoplasia in situ (GCNIS)
  • Testicular GCT risk factors include cryptorchidism and a personal or family history of testicular cancer (Am Fam Physician 2018;97:261)
  • Testicular GCT are likely the result of genetic and environmental factors during fetal development and subsequent changes in premalignant germ cells following puberty (Expert Rev Anticancer Ther 2020;20:75)
Clinical features
  • Presentation with elevated serum beta hCG, scrotal mass with or without pain, possible hyperthyroidism and gynecomastia (Curr Oncol Rep 2015;17:2, Am Fam Physician 2018;97:261)
  • Can present with hemoptysis, seizures, confusion (Radiol Case Rep 2021;16:923, Curr Oncol Rep 2015;17:2)
  • Patients with pure or predominant choriocarcinoma have distant metastasis at presentation (100% in 1 study), usually to the lungs (67%), liver (60%) and less frequently brain (20%) (Am J Surg Pathol 2014;38:111)
  • Cases with minimal choriocarcinoma component (≤ 5%) have regional lymph node metastases in 10% and distant metastases in 7% at presentation (Am J Surg Pathol 2018;42:1113)
  • Choriocarcinoma syndrome is a rare, life threatening complication in patients with substantially increased serum beta hCG (usually > 50,000 IU/L) due to high choriocarcinoma tumor burden at metastatic sites, often following chemotherapy induction, yielding fatal bleeding and pulmonary hemorrhage leading to acute respiratory failure (Cureus 2021;13:e14286)
Diagnosis
Laboratory
Radiology description
Radiology images

Images hosted on other servers:

CT with testicular mass

Testis ultrasound with calcification

Chest Xray with nodules

Chest Xray in choriocarcinoma syndrome


Chest CT with cannonball metastasis

MRI of brain

Abdominal CT with retroperitoneal mass

Abdomen CT with liver metastasis

Prognostic factors
Case reports
Treatment
Gross description
  • Central hemorrhage and necrosis, surrounded by viable tumor
  • Tumor size has a wide range:
Gross images

Contributed by Debra L. Zynger, M.D. and Andres Matoso, M.D.
Mixed GCT with predominately choriocarcinoma Mixed GCT with predominately choriocarcinoma

Mixed GCT with predominate choriocarcinoma

Mixed GCT with minor component of choriocarcinoma Mixed GCT with minor component of choriocarcinoma Mixed GCT with minor component of choriocarcinoma

Mixed GCT with minor component of choriocarcinoma

Large hemorrhagic tumor

Frozen section description
  • Intraoperative definitive diagnosis of the type of testicular GCT is not needed because it does not impact intraoperative management
  • Determination of benign versus malignant testicular tumor for small masses via frozen section is highly accurate (Eur Urol 2003;44:458)
Microscopic (histologic) description
  • Solid nests and sheets of syncytial cells (syncytiotrophoblasts) and mononucleated trophoblasts (cytotrophoblasts and intermediate trophoblasts)
  • Syncytiotrophoblasts:
    • Large cells
    • Abundant, dense, eosinophilic cytoplasm
    • Indistinct cell border
    • Multinucleated
    • Pleomorphic, hyperchromatic nuclei which can have a degenerated, smudged appearance
    • Grow over mononucleated cells and can form a thin compressed rim or a thicker cap
  • Mononucleated trophoblasts (cytotrophoblasts and intermediate trophoblasts):
    • Medium sized cells
    • Pale, eosinophilic cytoplasm
    • Distinct cell border
    • Mitotically active
  • Hemorrhage, cyst formation and necrosis are common (Am J Surg Pathol 2014;38:111)
  • Lymphovascular invasion is often present in tumors with pure or predominant choriocarcinoma (Am J Surg Pathol 2014;38:111)
  • Germ cell neoplasia in situ may be present
  • Remaining nonneoplastic testicular parenchyma may show findings seen in other germ cell tumors, including atrophy, maturation arrest, Leydig cell hyperplasia, microlithiasis, angiopathy, Sertoli cell only tubules and Sertoli cell nodules (Am J Surg Pathol 2006;30:1260)
Microscopic (histologic) images

Contributed by Debra L. Zynger, M.D.
2 cell types 2 cell types

2 cell types

Hemorrhage

Hemorrhage

Syncytiotrophoblasts

Syncytiotrophoblasts

Syncytiotrophoblast rim Syncytiotrophoblast rim

Syncytiotrophoblast rim


Mononucleated trophoblasts Mononucleated trophoblasts Mononucleated trophoblasts

Mononucleated trophoblasts


Choriocarcinoma with IHC

Choriocarcinoma with IHC

bhCG

Beta hCG

Inhibin

Inhibin

CK7

CK7

GATA3

GATA3

OCT 3/4

OCT 3/4

Cytology description
  • Dual cell population composed of mononucleated trophoblasts and syncytiotrophoblasts (Ann Saudi Med 1997;17:432)
  • Mononucleated trophoblasts:
    • Intermediate size, round
    • Single round nuclei, coarse chromatin and visible single nucleoli
    • Clear to slightly basophilic cytoplasm with few small vacuoles (Ann Saudi Med 1997;17:432)
  • Syncytiotrophoblasts:
    • Large cells
    • Multiple, pleomorphic nuclei
    • Abundant cytoplasm with variable numbers of small vacuoles (Ann Saudi Med 1997;17:432)
  • Background of necrosis and hemorrhage which may limit cellularity of specimen (Ann Saudi Med 1997;17:432)
Positive stains
Molecular / cytogenetics description
Sample pathology report
  • Right testis, radical orchiectomy:
    • Mixed germ cell tumor, teratoma (35%), seminoma (20%), embryonal carcinoma (20%), yolk sac tumor (20%) and choriocarcinoma (5%) types (see synoptic report)
Differential diagnosis
Board review style question #1

Which testicular germ cell tumor component is classically associated with hemorrhage?

  1. Choriocarcinoma
  2. Embryonal carcinoma
  3. Seminoma
  4. Teratoma
  5. Yolk sac tumor
Board review style answer #1
A. Choriocarcinoma

Comment Here

Reference: Choriocarcinoma
Board review style question #2

Which testicular germ cell tumor component is shown in the image?

  1. Choriocarcinoma
  2. Embryonal carcinoma
  3. Seminoma
  4. Teratoma
  5. Yolk sac tumor
Board review style answer #2
A. Choriocarcinoma

Comment Here

Reference: Choriocarcinoma

Cryptorchidism
Definition / general
  • Absence of one or both testes in the scrotum
  • Most common congenital abnormality of the genitourinary tract (Transl Pediatr 2016;5:233)
  • Associated with infertility and subfertility, testicular germ cell tumor, testicular torsion and inguinal hernia
Essential features
  • Absence of one or both testes in the scrotum most commonly due to failure of descent into the scrotum during fetal development
  • Increased risk of reduced fertility and testicular germ cell tumor
Terminology
  • Cryptorchidism: absence of one or both testes in the scrotum
  • Anorchia or vanishing testis syndrome: absence of one or both testes
  • Orchiopexy or orchidopexy: procedure that tethers the testicle into the scrotum
  • Ectopic testis: testis is situated away from normal path of descent; differs from undescended testicle
ICD coding
  • ICD-10:
    • Q53.1 - undescended testicle, unilateral
    • Q53.2 - undescended testicle, bilateral
    • Q53.9 - undescended testicle, unspecified
Epidemiology
Sites
  • Undescended testicle may be located in the abdomen or inguinoscrotal region
Pathophysiology
  • Testicular descent from initial position on abdominal wall is divided into 2 stages with abnormalities of migration thought to be associated with undescended testicle (Endocr Rev 2008;29:560)
    • Transabdominal descent: the gubernaculum, a caudal ligament connecting the testicle to the internal ring, holds testicle near internal ring as abdominal cavity develops; insulin-like factor 3 (INSL3) and anti-Müllerian hormone dependent
    • Inguinoscrotal descent: the gubernaculum shortens, pulling testicle through inguinal canal to position within scrotum; androgen dependent
  • May be a component of testicular dysgenesis syndrome, which includes increased levels of testicular malignancy, poor semen quality and hypospadias
Etiology
Clinical features
  • 80% of undescended testicles palpated within inguinal canal or high scrotal area
  • 20% of undescended testicles not palpated
  • Associated with increased risk of testicular germ cell tumor
    • Increasing risk of malignancy with delayed treatment (J Urol 2009;181:452)
    • Relative risk of malignancy is 2 - 8x; higher risks associated with delayed repair, bilateral cryptorchidism, additional endocrinopathies, abnormal karyotype and intra-abdominal testes
    • Seminoma is most common malignancy
Diagnosis
Laboratory
  • In cases of bilateral undescended testicles, hormonal studies (LH, FSH, Müllerian inhibiting substance [MIS] and testosterone) may support diagnosis of gonadal dysgenesis with abnormal hCG stimulation test (Am Fam Physician 2000;62:2037)
Radiology description
Prognostic factors
  • Delayed repair reduces fertility, which is worse with bilateral cryptorchidism
  • Relative risk for testicular germ cell tumor is 2.2x with prepubertal repair versus 5.4x with postpubertal repair (N Engl J Med 2007;356:1835)
Case reports
Treatment
Clinical images

Images hosted on other servers:

Intra-abdominal mass

Abdominal testis

Mass with right gonadal vein

Scrotum with cryptorchidism

Abdominal wall ectopic testis

Gross description
  • Smaller and atrophic compared to descended testis
Gross images

Contributed by Debra L. Zynger, M.D.

Cryptorchid testicle

Microscopic (histologic) description
  • Histopathologic differences become much more pronounced after 2 years of life and are increased with delayed orchiopexy (J Urol 2009;182:704)
  • Peritubular fibrosis
  • Seminiferous tubule atrophy
  • Decreased / absent spermatogenesis
  • Sertoli cells may demonstrate granular cell change with eosinophilic granular cytoplasmic lysosomes
  • Sertoli cell only seminiferous tubules: tubules with only bland, monotonous pale cells with granular cytoplasm attached to the basement membrane; absent germ cells and no spermatogenesis
  • Sertoli cell nodule: nodules composed of immature elongated Sertoli cells (Pick adenoma) may be present; can have central microliths
  • Increased microliths within seminiferous tubules
  • Retained Leydig cells give appearance of hyperplasia
  • Evaluate for germ cell neoplasia in situ (GCNIS): large, pleomorphic, basally located cells within the seminiferous tubules may be present; use immunostains for confirmation
Microscopic (histologic) images

Contributed by Debra L. Zynger, M.D.

Sertoli cell only tubules

Absent spermatogenesis

Peritubular fibrosis

Sertoli cell nodule


Retained Leydig cells

Granular cell change within Sertoli cells

Microliths

Cytology description
  • Not used to evaluate a cryptorchid testis
Positive stains
Sample pathology report
  • Ectopic testis, removal:
    • Seminiferous tubules with Sertoli only tubules, peritubular fibrosis, no spermatogenesis and microliths consistent with cryptorchid testis
Differential diagnosis
  • Germ cell neoplasia in situ (GCNIS):
    • Atypical cells with large nuclei located next to the basement membrane of the seminiferous tubules
    • Cytoplasmic clearing around nuclei
    • Heterogeneity of cells within seminiferous tubules
    • Positive for CD117, PLAP, D2-40, OCT 3/4 (but can be falsely positive in boys < 2 years old with cryptorchidism)
Board review style question #1
A 24 year old man with a history of untreated cryptorchidism presents with a painful abdominal mass. Pathology is most likely to be consistent with which of the following?

  1. Dermoid cyst
  2. Germ cell neoplasia in situ
  3. Mixed germ cell tumor
  4. Seminoma
  5. Sertoli cell tumor
Board review style answer #1
D. Seminoma is the most common malignancy in the setting of cryptorchidism.

Comment Here

Reference: Cryptorchidism
Board review style question #2

What histological changes are most likely to be observed in a nonneoplastic cryptorchid testis?

  1. Atrophic seminiferous tubules, peritubular fibrosis and nodules of cells containing dark nucleoli and eosinophilic granular cytoplasm
  2. Bland appearing cysts lined with cuboidal epithelium and ciliated cells
  3. Cysts lined by squamous cells filled with keratin
  4. Glycogen rich cells with large, irregular nuclei arranged in sheets
  5. Rod-like cytoplasmic inclusions with a diffuse growth pattern and rare mitotic cells
Board review style answer #2
A. Atrophic seminiferous tubules, peritubular fibrosis and nodules of cells containing dark nucleoli and eosinophilic granular cytoplasm are findings consistent with an undescended testicle. B is incorrect because these might be the findings associated with benign glands, such as those seen in endometriosis or endosalpingiosis. C is incorrect because this histology may be associated with teratoma. D is incorrect because these might be the findings of a seminoma, which are more common in undescended testicles. E is incorrect because Leydig cell tumors often contain Reinke crystals, described in this answer choice.

Comment Here

Reference: Cryptorchidism

Cystic trophoblastic tumor
Definition / general
  • Distinctive cystic trophoblastic lesion arising after spontaneous or chemotherapy induced regression of choriocarcinoma
  • Described predominantly in postchemotherapy retroperitoneal lymph node dissections of patients with testicular germ cell tumors
  • Has also been described to involve extragonadal sites in both treated and untreated testes of patients with mixed germ cell tumors
Essential features
  • Cystic proliferation of trophoblasts in postchemotherapy resections from patients with metastatic, nonseminomatous germ cell tumors of the testis
  • Not aggressive, with behavior that is similar to postchemotherapy residual teratoma
Terminology
  • Cystic trophoblastic tumor (CTT)
  • Previously known as choriocarcinoma-like lesion (CCLL) with 2 subtypes: teratomatous CCLL and cystic atypical choriocarcinoma (Am J Surg Pathol 1988;12:531)
ICD coding
  • ICD-10: C63.9 - malignant neoplasm of male genital organ, unspecified
  • ICD-11: 2C80.Y - other specified malignant neoplasms of testis
Epidemiology
Sites
  • Primarily postchemotherapy retroperitoneal lymph nodes with mixed germ cell tumors
  • Treated or untreated testes with mixed germ cell tumors
  • Extragonadal sites (e.g., central nervous system) secondary to treated or untreated testicular mixed germ cell tumors
Etiology
  • May arise from regressing choriocarcinoma as the more aggressive cells are eliminated by chemotherapy or spontaneous regression, leading to the persistence of a slow growing, less aggressive component of intermediate type trophoblasts that develops into cystic trophoblastic tumor (Am J Surg Pathol 2017;41:788)
  • Cystic trophoblastic tumor may also represent an intermediate stage in the maturation of choriocarcinoma to teratoma (Am J Surg Pathol 2004;28:1212)
  • Recent research shows that the levels of miR-371a-3p are negligible in CTT; these findings are similar to those found in postpubertal type teratoma and to the metastatic mature teratoma masses; hence, it supports the hypothesis that CTT represents an advanced maturation step toward teratoma (Histopathology 2023;83:151)
Clinical features
  • Most commonly seen in patients with testicular germ cell tumors after cisplatin based chemotherapy
  • Can occur as part of primary testicular mixed germ cell tumors and are occasionally seen in visceral and nonregional lymph node germ cell tumor metastases (Am J Surg Pathol 2017;41:788)
  • Less commonly, these tumors have been described in germ cell tumor patients without chemotherapy (Virchows Arch 2023;482:581)
Diagnosis
  • Histologic examination of involved tissue
Laboratory
  • Normal to mildly elevated levels of the beta subunit of human chorionic gonadotropin (βhCG)
Radiology description
  • Enlarged retroperitoneal lymph nodes
Prognostic factors
  • Prognosis of CTT in retroperitoneal lymph node dissection specimens is similar to that of teratoma and complete surgical resection without additional therapy is therefore recommended (Am J Surg Pathol 2017;41:788)
Case reports
  • 12 year old boy with cystic trophoblastic tumor in a primary central nervous system postchemotherapy germ cell tumor (Int J Surg Pathol 2020;28:925)
  • 22 year old man with mixed germ cell tumor composed of postpubertal teratoma, embryonal carcinoma and cystic trophoblastic tumor (Cesk Patol 2022;67:212)
  • 31 year old man with metastatic cystic trophoblastic tumor without prior chemotherapy (Urol Case Rep 2017;13:154)
Treatment
  • Requires no further treatment, however, these patients should be carefully monitored on follow up
Gross description
  • CTT is admixed commonly with teratoma and less commonly with other germ cell tumors and it occurs in small proportions; for these reasons, characteristic gross features cannot be ascribed to it
Microscopic (histologic) description
  • Cysts lined by squamoid appearing, predominantly mononucleated intermediate trophoblasts
  • Cyst may vary in size and shape and often appears collapsed among other germ cell tumor components
  • Cells display smudged nuclei with abundant eosinophilic cytoplasm, suggesting a degenerative phenotype
  • Cytoplasmic lacunae (vacuoles) may be present
  • Small solid clusters of cells and single cells may be seen adjacent to the larger cysts
  • Cyst may contain eosinophilic secretion or fibrinoid material
  • Cytoplasmic lacunae may coalesce to create variably sized spaces
  • Mitoses are rarely seen
  • Reference: Am J Surg Pathol 2004;28:1212, Am J Surg Pathol 2017;41:788
Microscopic (histologic) images

Contributed by Vikas Mehta, M.D.
Cyst, fibrinoid material

Cyst, fibrinoid material

Intermediate type trophoblasts Intermediate type trophoblasts

Intermediate type trophoblasts

Intermediate trophoblasts, smudged chromatin

Intermediate trophoblasts, smudged chromatin

Solid nest, fibrinoid material

Solid nest, fibrinoid material

βhCG positivity

βhCG positivity

Positive stains
Sample pathology report
  • Retroperitoneal lymph node, lymph node dissection:
    • Metastatic teratoma in a background of extensive fibrosis and necrosis; microscopic foci of cystic trophoblastic tumor
Differential diagnosis
  • Choriocarcinoma:
    • Distinctly biphasic, hemorrhagic and pleomorphic; displays brisk mitotic activity
    • βhCG levels are expected to be highly elevated
  • Teratoma:
    • Demonstrates varying degrees of atypia
    • Any type of tissue may be present, such as gastrointestinal glands, respiratory epithelium, cartilage, squamous epithelium with keratinization, primitive undifferentiated spindle cells or neuroepithelium
  • Somatic malignancy, such as squamous cell carcinoma:
    • Hypercellular variants with foci of solid areas can mimic CTT
Additional references
Board review style question #1
Which of the following is a feature consistent with cystic trophoblastic tumor?

  1. Biphasic pattern of admixed cytotrophoblasts and syncytiotrophoblasts
  2. Infiltrative growth pattern
  3. Normal to mildly elevated serum levels of βhCG
  4. Variable SALL4 expression in mononuclear cells
Board review style answer #1
C. Normal to mildly elevated serum levels of βhCG is consistent with cystic trophoblastic tumors. Other features are characteristic of choriocarcinoma in which βhCG levels are expected to be highly elevated. Answer A is incorrect because choriocarcinoma is biphasic, hemorrhagic and pleomorphic and it displays brisk mitotic activity. Answer B is incorrect because choriocarcinoma exhibits an infiltrative and destructive pattern. Answer D is incorrect because SALL4 positivity is seen in cytotrophoblasts in choriocarcinoma.

Comment Here

Reference: Cystic trophoblastic tumor
Board review style question #2

Which of the following statements is true for cystic trophoblastic tumors (CTTs)?

  1. CTTs after chemotherapy show a high frequency of progressive disease and warrant additional chemotherapy
  2. CTTs are identified exclusively in the setting of postchemotherapy retroperitoneal lymph node dissections of patients with testicular germ cell tumors
  3. CTTs lack keratohyaline granules and central keratin and often have intracytoplasmic lacunae
  4. CTTs show goblet cells, ciliated cells and other specialized epithelial types
Board review style answer #2
C. CTTs lack keratohyaline granules and central keratin and often have intracytoplasmic lacunae. Answer A is incorrect because the prognosis of CTT is similar to teratoma and requires no further treatment but a close follow up. Answer B is incorrect because CTT was first described in retroperitoneal lymph node dissections after chemotherapy; however, it is now known that these tumors can occur as part of primary testicular mixed germ cell tumors and they are occasionally seen in visceral and nonregional lymph node germ cell tumor metastases. Less commonly, these tumors have been described in germ cell tumor patients without chemotherapy. Answer D is incorrect because cysts are lined by squamoid appearing cells with smudged irregular nuclei, abundant eosinophilic cytoplasm with intracytoplasmic lacunae and associated fibrinoid material.

Comment Here

Reference: Cystic trophoblastic tumor

Cysts
Definition / general
  • Benign, cystic lesions that may be confused with testicular tumors
  • Does not include cystic neoplasms (e.g., epidermoid cyst, dermoid cyst [teratoma], papillary cystadenoma of the epididymis)
Essential features
  • Mesothelial cysts and hydrocele constitute benign true cysts and fluid collections arising from the mesothelial lining of the visceral and parietal tunica vaginalis and are more common in children
  • Cysts and cystic dilatations of the epididymis and rete testis (or more rarely, the seminiferous tubule) arise from the tubal epithelium, frequently secondary to obstruction and are more common in adults
  • Frequently characterized by simple unilocular or multilocular cystic structures and an absence of complex growth / solid areas
Terminology
  • Includes true cysts and fluid collections (celes) of the spermatic cord and paratesticular structures
  • 2 main types
    • Extratesticular (arises from the mesothelial lining [tunica vaginalis] and does not communicate with tubular system)
      • Hydrocele
        • Encysted presentation does not communicate with peritoneal cavity
        • Funicular presentation does communicate with peritoneal cavity
      • Mesothelial cyst (also termed spermatic cord cyst)
    • Testicular (arises from the epithelial lining of the rete testis, efferent ducts and epididymis)
      • Epididymal cysts and cystic dilatation (tubular ectasia) of epididymis and rete testis are associated and likely exist on a continuum
      • Epididymal cysts > 2 cm within the vaginal cavity are termed spermatocele
      • Cystic dysplasia of rete testis is distinctive congenital malformation
ICD coding
  • ICD-10:
    • N50.3 - cyst of epididymis
    • N50.89 - other specified disorders of the male genital organs
    • N50.81 - testicular pain
    • N50.82 - scrotal pain
    • N50.9 - disorder of male genital organs, unspecified
Epidemiology
  • Hydrocele is one of the most common causes of painless scrotal swelling in infants (up to 10%); generally regresses spontaneously by 1 - 2 years old and rarely persists into adulthood
  • Mesothelial cysts are most common in men > 40 years old
  • Cysts and cystic dilatation of epididymis and rete testis constitute the most common paratesticular cystic lesions, frequently in men > 55 years old and seen in up to 30% of asymptomatic patients at ultrasound (AJR Am J Roentgenol 1984;143:161)
  • Cystic dysplasia of rete testis is rare, generally unilateral and most often seen in children (mean presentation of 5 years old), in association with urinary tract malformations (Front Pediatr 2022;10:898038)
Sites
  • Spermatic cord, testicular adnexa
Etiology
  • Hydrocele
    • Congenital form occurs due to patent processus vaginalis or abnormal closure of processus; most regress by adulthood
    • May be associated with trauma, testicular torsion, tumors, chronic inflammation (e.g., in setting of bacterial epidiymoorchitis, filarial disease) or congenital lymphatic malformations
  • Mesothelial cyst
  • Cysts (and cystic dilatation) of epididymis and rete testis
    • Acquired condition frequently associated with or resulting from mechanical compression of epididymis or spermatic cord by surgical changes (e.g., vasectomy), trauma, neoplastic or infectious process, dialysis (rete testis cystic dilation) or hormonally induced atrophy of epididymis (e.g., secondary to cirrhosis) (Hum Pathol 1996;27:336, Am J Surg Pathol 1996;20:1231, J Ultrasound 2011;14:208)
    • Rare intraparenchymal cystic dilatation of seminiferous tubules may result from focal malformation or inflammation
    • Epididymal cysts associated with in utero exposure to diethylstilbestrol
  • Cystic dysplasia of rete testis
    • Congenital malformation of rete testis, frequently associated with ipsilateral urogenital malformations (e.g., renal agenesis, multicystic dysplasia of the kidney)
    • Thought to arise due to disorders of communication between mesonephric duct and germinal epithelium during development (Front Pediatr 2022;10:898038)
Clinical features
  • Painful or painless cystic swelling of the scrotum
  • May be incidentally identified
Diagnosis
  • Transillumination
  • Ultrasonography (Doppler ultrasonography is important in a setting of painful swelling to rule out testicular torsion)
Radiology description
  • Varies by entity
  • Most frequently characterized by simple uninoculated or multiloculated fluid filled cystic structures
  • Complex loculations or areas of solid growth should raise concern for alternative etiology
Radiology images

Images hosted on other servers:
Cyst of testis (focal dilatation of seminiferous tubule)

Cyst of testis (focal dilatation of seminiferous tubule)

Cystic dilatation (tubular ectasia) of rete testis

Cystic dilatation (tubular ectasia) of rete testis

Epididymis cyst

Epididymis cyst

Cystic dilatation (tubular ectasia) of epididymis

Cystic dilatation (tubular ectasia) of epididymis


Mesothelial cyst in tunica albuginea

Mesothelial cyst in tunica albuginea

Hydrocele

Hydrocele

Chronic hydrocele with multiloculated appearance

Chronic hydrocele with multiloculated appearance

Spermatic cord cyst

Spermatic cord cyst

Case reports
Gross description
  • Hydrocele
    • Fluid filled cyst caused by accumulation of clear serous fluid between the visceral and parietal tunica vaginalis
    • Cyst fluid is generally clear but may be complicated by hemorrhage or infection
    • May appear multiloculated in the setting of chronic inflammation and progressive fibrosis resulting in internal septa (J Ultrasound 2011;14:208)
  • Mesothelial cyst
    • Simple or multilocular cystic dilation within the tunica vaginalis or tunica albuginea
    • Clear to bloody serous cyst fluid
  • Epididymal cyst (spermatocele) and simple cyst of rete testis
    • Epididymal cyst fluid contains abundant spermatozoa
    • Epididymal cysts > 2 cm present clinically as spermatocele
    • Rete testis cysts are typically unilocular and limited to hilum but may bulge into testis; a multilocular form is associated with epididymal cysts
Gross images

AFIP images
Cyst of tunica albuginea

Cyst of tunica albuginea



Images hosted on other servers:
Encysted hydrocele

Encysted hydrocele

61 year old man

Multiple paratesticular, unilocular, thin walled cysts

Microscopic (histologic) description
  • Hydrocele and mesothelial cysts
    • Single layer of cuboidal or flattened mesothelial cells
    • Prominent atypia, squamous metaplasia or inflammation may be present (Mod Pathol 2005;18:S131)
  • Cysts and cystic dilatation of epididymis and rete testis
    • Simple cyst or multicystic distension of the rete testis or epididymis
    • Single layer of normal tubal epithelium (cuboidal to flattened ciliated epithelial cells)
    • Lumen filled with spermatozoa
    • Chronic inflammation may be present
    • May show columnar cell change, fibrosis, giant cell reaction
  • Cystic dysplasia of rete testis
    • Multicystic lesion located in the mediastinum testis
    • Irregular cystic spaces lined by a single layer of cuboidal epithelium (Front Pediatr 2022;10:898038)
Microscopic (histologic) images

Contributed by Stephanie Siegmund, M.D., Ph.D. and AFIP
Mesothelium lined cystic cavity

Mesothelium lined cystic cavity

Mesothelial lining of hydrocele

Mesothelial lining of hydrocele

Cystic dilatation of epididymis Cystic dilatation of epididymis

Cystic dilatation of epididymis


Cystic dilatation of epididymis

Cystic dilatation of epididymis

Spermatocele

Spermatocele

Ruptured spermatocele

Ruptured spermatocele

Cyst of tunica albuginea

Cyst of tunica albuginea

Sample pathology report
  • Right testes:
    • Mesothelium lined cyst (3.0 cm), consistent with hydrocele sac

  • Spermatocele sac:
    • Ciliated columnar / cuboidal cell lined cyst with aggregates of spermatozoa, consistent with spermatocele
Differential diagnosis
  • Ovarian epithelial type tumors:
    • Believed to arise from Müllerian duct remnants or secondary to Müllerian metaplasia of mesothelium
    • Entire spectrum of serous or mucinous cystadenomas, borderline tumors and carcinomas can be seen, with serous borderline tumors most frequent (Diagn Pathol 2015;10:118)
    • Histologically and immunophenotypically identical to ovarian counterparts
    • Can be distinguished by their complex papillary structures and expression of PAX8, WT1, variable ER and PR, EMA and CA-125
  • Cystadenoma of rete testis:
    • Irregular proliferation of cystic structures lined by cytologically bland low cuboidal epithelium and separated by fibrous septa (AJR Am J Roentgenol 2007;189:W67)
    • Can be distinguished grossly from rete testis cysts and cystic dilatation by admixed irregular cystic and solid areas and histologically by areas of complex growth (e.g., sertoliform cystadenoma), budding and tufting
  • Papillary cystadenoma of epididymis:
    • Associated with von Hippel-Lindau syndrome
    • Dilated ducts lined by a single or double layer of cuboidal to low columnar epididymis epithelium, characterized by cilia and secretory droplets / variable cytoplasmic vacuolization; rarely, mucinous features may be present mimicking intestinal type goblet cells
    • Can be distinguished from rete testis cysts and cystic dilatation by presence of papillae
  • Teratoma, prepubertal type:
    • Epidermoid cysts are unilocular and can be distinguished grossly and histologically by their simple stratified squamous epithelium lining (lacking skin appendages) and keratinaceous contents
    • Dermoid cysts can be distinguished grossly and histologically by the complex structure frequently containing multiple somatic tissue elements (e.g., bone, hair)
  • Cysts of appendix epididymis and appendix testis:
    • These 2 embryonic remnants may become enlarged and cystically dilated by serous fluid, thus mimicking other testicular cysts (Surg Radiol Anat 2005;27:557)
Board review style question #1

A 67 year old patient with a history of cirrhosis presents with painless scrotal swelling. An orchiectomy reveals the paratesticular lesion pictured above. What is the most likely diagnosis?

  1. Cystic dilatation of epididymis
  2. Hydrocele
  3. Mesothelial cyst
  4. Spermatocele
  5. Teratoma
Board review style answer #1
A. Cystic dilation of epididymis. The image shows a cystically dilated epididymis with proteinaceous luminal accumulation in a cystic space lined by tubal epithelium. Answer E is incorrect because the structure is consistent with normal histology in this location rather than teratomatous elements and teratoma would be less common in an elderly patient with a history of hormonal abnormality (i.e., cirrhosis). Answers C and B are incorrect because a mesothelial cyst or hydrocele would be lined by mesothelium. Answer D is incorrect because while spermatocele is also an epididymal structure lined by tubal epithelium, there is no evidence of spermatozoa in this image.

Comment Here

Reference: Cysts
Board review style question #2
Which of the following clinical scenarios is most likely to be associated with cystic dysplasia of the rete testis?

  1. 4 day old neonate with persistent bilateral scrotal swelling
  2. 5 year old boy with a complex solid and cystic scrotal mass
  3. 6 year old boy with right renal agenesis and unilateral scrotal swelling
  4. 34 year old man with 2 months of painful unilateral scrotal mass and recent diagnosis of tuberculosis
  5. 67 year old man with metastatic castration resistant prostatic adenocarcinoma
Board review style answer #2
C. 6 year old boy with right renal agenesis and unilateral scrotal swelling. Cystic dysplasia of the rete testis is most commonly seen in children (median age is 5 years old) and is frequently associated with ipsilateral urogenital malformations, such as renal agenesis. Answer A is incorrect because by contrast, unilateral or bilateral neonatal scrotal swelling is most commonly due to persistent patency of the processus vaginalis with spontaneous resolution by 1 - 2 years of age. Answers E and D are incorrect because scrotal swelling in adults is more frequently due to cysts or cystic dilatation of the epididymis or rete testis secondary to obstruction in the setting of hormonal changes, neoplastic processes, surgery or infection. Answer B is incorrect because the presence of a complex scrotal mass rather than uniloculated or multiloculated fluid filled mass raises the possibility of prepubertal teratoma.

Comment Here

Reference: Cysts

Disorders of sexual differentiation
Androgen insensitivity
Definition / general
  • Either XY or XXY
  • Called testicular feminization if complete
  • Most frequent cause of male pseudohermaphroditism
  • Lack of androgen receptor due to mutations in gene on X chromosome

Clinical features
  • Phenotypic female, tall with well formed breasts, absent / scanty pubic and axillary hair, shallow vagina and lack of upper vagina because anti-Müllerian hormone (AMH) causes Müllerian duct regression
  • Patients also have bilateral cryptorchidism with intraabdominal, inguinal or labial testes; usually no Wolffian or Müllerian derivatives
  • Recommend gonadectomy by puberty since associated with germ cell tumors (30% by age 50)

Case reports

Gross description
  • Tanish brown testes with multiple white nodules of Sertoli cells and Wolffian / Müllerian duct cysts at lateral pole of testis

Microscopic (histologic) description
  • Small seminiferous tubules without lumina composed of Sertoli cells only, usually immature, with sparse spermatogonia, marked Leydig cell hyperplasia (often without Reinke crystals), ovarian type stroma; nodules are probably hamartomas of Sertoli cells
Gonadal dysgenesis-mixed
Definition / general
  • Testis plus contralateral streak gonad, testis and contralateral gonadal agenesis, hypoplastic gonads with tubules in one gonad or streak gonad with
  • Müllerian structures present since no / minimal AMH produced
  • Usually bilateral fallopian tubes
  • Usually incomplete masculinization of external genitalia, poor development of ipsilateral Wolffian duct structures
  • External genitalia are male, female or ambiguous (ambiguous in most patients, two - thirds raised as female)
  • Phenotypic females may develop signs of virilization at puberty
  • Karyotypes: 45 X0 / 46 XY, 46 XY most common
  • Associated with low immunoglobulin levels, aberrant bony development of inner ear structures, cardiovascular and renal anomalies

Treatment
  • Early bilateral gonadectomy advocated if Y chromosome material is present to prevent gonadoblastoma (one - third) or other germ cell tumors; also to prevent virilization if patient is raised as female (Arch Pathol Lab Med 1990;114:679)

Microscopic (histologic) description
  • Tubules with mild hypospermatogenesis to total sclerosis (Mod Pathol 2002;15:1013, Hum Pathol 1982;13:700)
  • Streak gonad has ovarian stroma without primordial ovarian follicles
  • Streak ovary has primitive sex cord-like structures within ovarian type stroma, variable germ cell components, resembles either gonadoblastoma, granulosa cell or Sertoli cell tumors

Differential diagnosis
Gonadal dysgenesis-pure
Definition / general
  • Also called Swyer syndrome
  • Female phenotype and female genital appearance, normal Müllerian structures
  • Bilateral streak gonads, so no hormonal or reproductive potential
  • 46 XY but no testicular tissue; no evidence of Turner syndrome
  • Early bilateral gonadectomy is recommended due to high incidence of gonadoblastoma and dysgerminoma (J Pediatr Urol 2011;7:72)

Case reports

Gross images

Images hosted on other servers:

Woman with dysgerminoma and streak gonad



Microscopic (histologic) images

Images hosted on other servers:

Dysgerminoma

Streak gonad

Klinefelter syndrome
Definition / general
  • 47XXY, reduced body and pubic hair, gynecomastia in 40 - 80%, high FSH, variable LH
  • Increased risk of breast cancer and extragonadal germ cell tumor

Gross description
  • Small, firm testes

Microscopic (histologic) description
  • Reduced number of intratubular germ cells, some tubules are Sertoli cell only
  • Also tubular sclerosis, Leydig cell nodules (appear hyperplastic due to tubular atrophy), focal spermatogenesis rarely seen

Electron microscopy description
Male pseudohermaphroditism
Definition / general
  • XY, testes present, phenotype ambiguous or female

Congenital adrenal hyperplasia
  • Autosomal recessive, due to various enzymatic defects that cause different patterns of synthesis of glucocortical, mineralocorticoid and sex hormone synthesis
  • Genetic males have cryptorchidism, viable Wolffian duct structures, female or ambiguous genitalia, no Müllerian duct structures
  • Testes resemble cryptorchid testes
  • May have bilateral Leydig cell hyperplasia with deficiencies of 21-hydroxylase, 11β hydroxylase, 17α hydroxylase, 3β hydroxysteroid dehydrogenase
  • Treat with corticosteroids or surgical excision of testes

Dysgenetic male pseudohermaphroditism
  • Bilateral dysgenetic testis, Müllerian structures, cryptorchidism, inadequate virilization
  • May be XO / XY mosaics
  • Infertile, no spermatogenesis
  • 30% incidence of testicular tumors (J Urol 1978;119:525)
Persistent Müllerian duct
Definition / general
  • Rare form of male pseudohermaphroditism, Müllerian duct structures persist due to lack of AMH effect due to either mutation in AMH gene on chromosome 19p13.3 or abnormality of receptor gene on chromosome 12q13
  • X linked or autosomal recessive
  • Phenotypic male, normal external genitalia, unilateral or bilateral cryptorchidism, may have empty hemiscrotum, normal Wolffian duct derivatives; however also have Müllerian duct derivatives (uterus and usually 2 fallopian tubes) within an inguinal hernia
  • 2 forms:
    • Unilateral cryptorchidism and contralateral hernia
    • Bilateral cryptorchidism, uterus in pelvis, both testes embedded in broad ligament
  • 15% risk of germ cell tumors, including intratubular germ cell neoplasia

Case reports

Differential diagnosis
True hermaphroditism
Definition / general
  • Unequivocal ovarian and testicular tissue in same patient, occuring as either bilateral or unilateral ovatestes or as a testis opposite an ovary, regardless of karyotype
  • Ovary is usually normal; testis usually lacks spermatogonia
  • No streak gonads present; usually no associated gonadoblastomas
  • Usually no other developmental malformations and patients may have normal sexual and reproductive functions

Treatment
  • After assigning gender, remove inappropriate gonad and biopsy remaining tissue

Microscopic (histologic) description
  • Ovarian compartment has numerous primordial follicles with primary oocytes and a few primary or antral follicles (Mod Pathol 2002;15:1013)

Differential diagnosis

Embryonal carcinoma
Definition / general
  • Pluripotent and malignant germ cell tumor (GCT) resembling undifferentiated stem cells during embryonic development
Essential features
  • Pleomorphic, high grade appearing type of GCT
  • Usually occurs mixed in combination with other types of testicular GCT
  • Predominance in a mixed GCT is associated with a higher risk of metastases and relapse (BJU Int 2020;125:355, BMC Cancer 2020;20:728)
  • Positive: OCT 3/4, CD30; negative / weak: D2-40, CD117, glypican 3
ICD coding
  • ICD-O: 9070/3 - embryonal carcinoma, NOS
Epidemiology
Sites
  • Testis
  • Anterior mediastinum
  • Retroperitoneum
Pathophysiology
Etiology
Clinical features
Diagnosis
  • Ultrasound
  • Serum tumor markers
  • CT imaging with contrast of chest, abdomen and pelvis to evaluate for metastasis
  • Radical inguinal orchiectomy with histopathologic evaluation is necessary for definitive diagnosis
Laboratory
  • Serum tumor markers should not be used as a screening tool
  • Elevated serum human chorionic gonadotropin (hCG), alpha fetoprotein (AFP) and lactate dehydrogenase (LDH) may be present (J Clin Oncol 2010;28:3388)
  • Current guidelines recommend measuring hCG, AFP and LDH before and after initial treatment (J Clin Oncol 2010;28:3388)
  • Serum tumor markers can be used to monitor for recurrence
Radiology description
  • Heterogenous and hypoechoic appearance on ultrasound; microlithiasis may be present (J Clin Ultrasound 2010;38:21)
  • Testicular non(pure)seminomatous GCTs appear heterogenous on MRI, with enhancement in areas of necrosis and hemorrhage (AJR Am J Roentgenol 2007;189:W331)
  • CT imaging is recommended to identify presence of metastases
  • Lymph node metastases from testicular nonseminomatous GCTs appear heterogenous or cystic on CT imaging (Radiol Clin North Am 2012;50:1111)
Radiology images

Images hosted on other servers:

Calcified mass

Prognostic factors
Case reports
Treatment
  • Initial management is radical inguinal orchiectomy
  • No further treatment necessary in 75% of clinical stage I nonseminomatous GCT (Ann Oncol 2010;21:1296)
  • If pT2 or higher, retroperitoneal lymph node dissection or chemotherapy
  • If pN2 - 3, chemotherapy
  • Resistant to radiation therapy (Arch Pathol Lab Med 2012;136:435)
Clinical images

Images hosted on other servers:

Cryptorchid testis with embryonal carcinoma

Gross description
  • Gray-tan mass with hemorrhage and necrosis
Gross images

Contributed by Debra L. Zynger, M.D.

Mixed GCT with predominance of embryonal carcinoma

Mixed GCT with minor component of embryonal carcinoma

Microscopic (histologic) description
  • Multiple growth patterns usually present (Am J Surg Pathol 2014;38:689)
  • 3 most common growth patterns: solid (55%), glandular (17%) and papillary (11%) (Am J Surg Pathol 2014;38:689)
  • Rare patterns: nested (3%), micropapillary (2%), anastomosing glandular (1%), sieve-like glandular (< 1%), pseudopapillary (< 1%), and blastocyst-like (< 1%) (Am J Surg Pathol 2014;38:689)
  • Polygonal cells (Arch Pathol Lab Med 2007;131:1267)
  • Cells are crowded, have indistinct distinct cell borders and appear to have overlapping nuclei
  • Moderate amount of amphophilic and granular cytoplasm
  • Pleomorphic, high grade nuclear features
  • Mitotic figures are common
  • Smudgy degenerative appearing nuclei are often seen
  • Necrosis is common, both as single cell necrosis and larger foci
  • Often grows admixed with yolk sac tumor and can rarely form polyembryoma-like structures, called embryoid bodies
  • Residual seminiferous tubules may contain germ cell neoplasia in situ (GCNIS) or intratubular embryonal carcinoma, which entirely fills the tubule
  • Lymphovascular invasion is common within embryonal carcinoma predominant GCTs, is usually best visualized at the periphery testicle and may entirely occlude vessels, mimicking nodules of tumor
Microscopic (histologic) images

Contributed by Debra L. Zynger, M.D.

Solid growth

Glandular growth

Papillary growth


Cellular overlap

Pleomorphic

Admixed with yolk sac tumor

Intratubular

Lymphovascular invasion

Spermatic cord invasion


OCT 3/4

CD30

PLAP

CD117

D2-40

AE1 / AE3

Virtual slides

Images hosted on other servers:

Mixed GCT with predominance of embryonal carcinoma

CD30

Cytology description
  • No role in diagnosis of primary testicular GCT
Negative stains
Electron microscopy description
Molecular / cytogenetics description
Sample pathology report
  • Left testis, radical orchiectomy:
    • Mixed germ cell tumor, teratoma (45%), embryonal carcinoma (25%), yolk sac tumor (15%), seminoma (10%) and choriocarcinoma (5%) types (see synoptic report)
  • See staging information
Differential diagnosis
Board review style question #1

A 26 year old man presented with a left testicle mass. A radical orchiectomy was performed. Which is the correct diagnosis for the testicular germ cell tumor component shown in the above image?

  1. Choriocarcinoma
  2. Embryonal carcinoma
  3. Seminoma
  4. Teratoma
  5. Yolk sac tumor
Board review style answer #1
B. Embryonal carcinoma

Comment Here

Reference: Embryonal carcinoma
Board review style question #2


The H&E image above is a brain metastasis of testicular germ cell tumor with choriocarcinoma and embryonal carcinoma. Which of the following 2 immunostains will be positive in choriocarcinoma but weak or negative in embryonal carcinoma (one of which is shown above)?

  1. AFP, PLAP
  2. CD30, OCT 3/4
  3. CK7, p63
  4. D2-40, CD117
  5. SALL4, AE1 / AE3
Board review style answer #2
C. CK7, p63. CK7 (shown in the immunostain image above) is strongly and diffusely positive choriocarcinoma. CK7 is weak or negative in embryonal carcinoma. Expression for p63 is seen in mononucleated trophoblast cells in choriocarcinoma. p63 is negative in embryonal carcinoma. AFP is positive in yolk sac tumor and weak to negative in embryonal carcinoma and choriocarcinoma. PLAP is variably positive in embryonal carcinoma and negative in choriocarcinoma. CD30 and OCT 3/4 are positive in embryonal carcinoma and negative in choriocarcinoma. D2-40 and CD117 are positive in seminoma but negative in both choriocarcinoma and embryonal carcinoma. SALL4 and AE1 / AE3 are positive in both choriocarcinoma and embryonal carcinoma.

Comment Here

Reference: Embryonal carcinoma
Board review style question #3

In a testicular mixed germ cell tumor, increased percentage of which component correlates with a worse prognosis?

  1. Embryonal carcinoma
  2. Seminoma
  3. Spermatocytic tumor
  4. Teratoma
  5. Yolk sac tumor
Board review style answer #3
A. Embryonal carcinoma. In a testicular germ cell tumor, higher percentage of embryonal carcinoma and choriocarcinoma are associated with negative outcomes, such as local and distant metastases. Increased proportions of teratoma and yolk sac tumor are associated with better outcomes. In a mixed germ cell tumor, percentage of seminoma usually does not impact prognosis or management. Spermatocytic tumor is a germ cell tumor unrelated to germ cell neoplasia in situ.

Comment Here

Reference: Embryonal carcinoma

Epidermoid cyst
Definition / general
Case reports
Gross description
  • Intraparenchymal lesion, usually adjacent to tunica albuginea, mean 2 cm, contains white grumous keratin debris
Microscopic (histologic) description
  • Cyst with keratinized squamous epithelial lining containing a granular cell layer, cyst filled with laminated keratin
  • Cyst rupture may cause granulomatous reaction
  • No adnexal structures, no other tissue types
  • Pilomatrixoma-like variant:
    • Adjacent testis does not show intratubular germ cell neoplasia, unclassified type (IGCNU)
    • Eosinophilic islands of "shadow" squamous epithelium with focal, peripheral calcification and ossification
    • The ghost-like cells are similar to the shadow cells of cutaneous pilomatrixoma (Am J Surg Pathol 2001;25:788)
Microscopic (histologic) images

Contributed by Debra L. Zynger, M.D. and @ThatGlassTho on Twitter

Testicular epidermoid cyst

Cyst wall with adjacent normal seminiferous tubule

Lamellar keratin within the cyst

Stratified squamous epithelium with adjacent keratin

Teratoma-epidermoid cyst Teratoma-epidermoid cyst

Teratoma - epidermoid cyst

Molecular / cytogenetics description
  • Isochromosome 12p and other overrepresentation of 12p are present in usual teratoma but not in epidermoid cyst (Clin Cancer Res 2006;12:5668)
Molecular / cytogenetics images

Images hosted on other servers:

Normal FISH for 12p

Differential diagnosis
  • Teratoma: usually postpubertal testis, IGCNU present, often atypia in teratomatous elements

Epididymitis
Definition / general
  • Primary cause of epididymal obstruction
  • Usually related to cystitis, prostatitis or urethritis that spreads through vas deferens or lymphatics
  • May cause testicular ischemia and necrosis, later scarring and infertility with preservation of Leydig cells and preserved sexual activity
  • Acute disease: epididymis enlarged, covered with fibrin, may contain pus and rupture
  • Brucellosis: affects testis and epididymis in 20% of cases; has granulomatous appearance
  • Gonorrhea: affects epididymis before testis
  • Tuberculosis: may cause confluent caseation that spreads into testis and simulates malignancy; may cause scrotal fistula; should culture to rule out M. kansasii and M. avium-intracellulare
  • Children: usually gram negative rods, congenital genitourinary abnormality
  • Sexually active men age 35 or less: Chlamydia trachomatis, Neisseria gonorrhoeae
  • Men age 35+: E. coli, Pseudomonas and other urinary tract infection organisms
Microscopic (histologic) images

Contributed by Veena Maheshwar, M.D., Kiran Alam, M.D., Anshu Jain, M.D.

25 year old man with epididymal mass: filiarial epididymitis


Epithelioid trophoblastic tumor
Definition / general
Essential features
  • Unusual trophoblastic tumor, distinct from choriocarcinoma and placental site trophoblastic tumor (PSTT) with morphologic features similar to a carcinoma
  • Both morphologic and immunohistochemical studies showed that the ETT is mainly composed of chorionic type intermediate trophoblastic cells
Terminology
  • Atypical choriocarcinoma
Epidemiology
  • Rare gestational trophoblastic neoplasm with only 52 cases documented in literature
  • Typically occurs in young men between 19 and 43 years
Sites
  • Testis, metastasis to lymph nodes, lung
Etiology
  • Seems to develop from neoplastic transformation of cytotrophoblastic cells that assume a differentiation toward the chorionic type intermediate trophoblastic cells that are normally found in chorion laeve (fetal membrane)
Clinical features
  • Typically seen in young men between 19 and 43 years
  • Serum βhCG may show mildly increased levels
Diagnosis
  • Based on characteristic morphology and immunoprofile
Laboratory
  • May be associated with mild elevation of βhCG
Radiology description
  • No characteristic features
Prognostic factors
  • Not an aggressive tumor
  • Requires no additional treatment after initial treatment for germ cell tumor
Case reports
Microscopic (histologic) description
  • Can assume several growth patterns including cohesive nests of squamoid cells with abundant pink cytoplasm and lacking associated hemorrhage
  • Tumor cells have single pleomorphic and hyperchromatic nuclei with prominent nucleoli and occasional multinucleation along with well defined cytoplasmic membranes without discernible intercellular bridges
  • Intracytoplasmic vacuoles containing fibrinoid cellular debris, sometimes with pyknotic nuclear fragments can be identified
  • Extracellular hyaline material and lymphocytes at periphery Infiltrating tumor nests and single squamoid cells can also be recognized
  • Mitotic count ranges from few to several in different fields
  • Occasional central cystic degeneration containing fibrinoid material is noted
  • Most striking histologic feature is absent biphasic pattern of cytotrophoblasts and syncytiotrophoblasts
Microscopic (histologic) images

Contributed by Debra L. Zynger, M.D.

Testicular epithelial trophoblastic tumor

Positive stains
Negative stains
Differential diagnosis
  • Choriocarcinoma: shows a biphasic pattern of syncytiotrophoblastic and mononucleated trophoblast cells and significant fresh hemorrhage, often with extensive necrosis; βhCG reactive syncytiotrophoblast cells which also usually shows only limited reactivity for HPL
  • Regressing choriocarcinoma: large mononucleated trophoblast cells with pale to eosinophilic cytoplasm; absent biphasic pattern of cytotrophoblasts and syncytiotrophoblasts with necrosis and fresh hemorrhage hyalinized fibrous background with numerous hemosiderin laden macrophages
    • Few mitotic figures
    • Prominent Leydig cell hyperplasia in the surrounding testis
    • AE1 / AE3 positive
  • Placental site trophoblastic tumor: sheets of large discohesive cells with focal hemorrhage; HPL (focal), p63 positive
Board review style question #1
Epithelial trophoblastic tumor can be differentiated from a choriocarcinoma by all the following except:

  1. Biphasic pattern of syncytiotrophoblastic and mononucleated trophoblast cells
  2. HSD3B1 Positivity
  3. Ki67 index of 10 - 25%
  4. Prominent fibrinoid material around vessels
Board review style answer #1
A. Biphasic pattern of syncytiotrophoblastic and mononucleated trophoblast cells

Comment Here

Reference: Epithelioid trophoblastic tumor

Fibroma thecoma group
Definition / general
  • Sex cord stromal tumors composed of spindle, oval or round cells with varying amounts of collagen present
Essential features
  • Hemorrhage and necrosis absent
  • Variable cell density, variable collagen
  • Usually no Sertoli or granulosa elements present but may have minor aggregates of other sex cord cells (Arch Pathol Lab Med 1999;123:391)
  • Usually scanty mitoses (≤ 5 mitoses/10 high power fields), can be as high as 10 mitoses/10 high power fields
Terminology
  • Synonyms:
    • Fibrothecoma
    • Thecofibroma
    • Thecoma / fibroma group
    • Testicular fibroma
ICD coding
  • ICD-O:
  • ICD-10: D29.20 - benign neoplasm of unspecified testis
  • ICD-11: 2F34 & XH34A0 - benign neoplasm of skin of male genital organs & thecoma, NOS
Epidemiology
Sites
  • Testis
  • Paratestis
Pathophysiology
  • Unknown
  • May be associated with Gorlin syndrome (nevoid basal cell carcinoma syndrome), abnormalities of PTCH gene (J Pediatr Surg 2010;45:E1)
Etiology
  • Unknown
Clinical features
  • Commonly presents as slowly increasing unilateral testicular mass (Urol Ann 2021;13:308)
  • Hormonal changes are extremely uncommon
  • Clinically benign behavior with no reported recurrences or metastases (Urol Ann 2021;13:308)
Diagnosis
  • Ultrasound
  • Orchiectomy
Laboratory
  • Hormone levels are typically within normal range
  • Serum testosterone and estrogen may be elevated in some secreting tumors
  • Tumor markers (beta human chorionic gonadotropin [hCG], alpha fetoprotein [AFP], lactate dehydrogenase [LDH]) are within normal range (Urol Case Rep 2020;33:101368, Urol Ann 2021;13:308)
Radiology description
  • Well defined, hypoechoic, solid mass
  • Can appear as a heterogeneous mass
Prognostic factors
Case reports
Treatment
  • Diagnostic orchidectomy is curative; observation after orchidectomy is the only necessary treatment
Gross description
  • Well defined, firm tumors ranging from 0.5 to 7.6 cm in greatest dimension (Am J Surg Pathol 2013;37:1208)
  • Cut surface is tan, yellow or white and solid to cystic; can have focally hemorrhagic areas
Gross images

Images hosted on other servers:

Well circumscribed intratesticular tumor

Microscopic (histologic) description
  • Circumscribed but unencapsulated; encapsulated forms are rare
  • Arranged in fascicles, storiform or a combination pattern in collagenized stroma with small blood vessels
  • Herringbone pattern may be seen
  • Variable cell density, variable collagen
  • Usually scanty mitoses (≤ 5 mitoses/10 high power fields), can be as high as 10 mitoses/10 high power fields
  • May be infiltrative and entrap seminiferous tubules; this feature has no impact on tumor behavior
  • Usually no Sertoli or granulosa components are present but may have minor aggregates of other sex cord cells (Arch Pathol Lab Med 1999;123:391)
  • If > 1 microscopic focus of other sex cord stromal cells, classify as mixed or unclassified since these tumors have metastatic potential unlike the classical fibroma
Microscopic (histologic) images

Contributed by Rafael E. Jimenez, M.D., M.H.A.
Bland nuclei

Bland nuclei

Fascicular pattern

Fascicular pattern

Encapsulated tumor

Encapsulated tumor

Collagenized stroma

Collagenized stroma

Inhibin

Inhibin

SF1

SF1

Negative stains
Electron microscopy description
  • Fibroma cells resemble both fibroblasts and myofibroblasts
Molecular / cytogenetics description
  • Although loss of heterozygosity at 9q22.3 (PTCH locus) is observed in 40% of sporadic ovarian thecoma fibromas, only one case in testis has been reported (J Pediatr Surg 2010;45:E1)
Sample pathology report
  • Right testis, orchiectomy:
    • Fibrothecoma (see comment)
    • Comment: The neoplastic cells showed positivity for inhibin, smooth muscle actin and SOX9 while MelanA, KIT and desmin were negative. This is consistent with the diagnosis above.
Differential diagnosis
  • Fibrosarcoma:
    • Malignant tumor with infiltration, cytological atypia, frequent mitoses, necrosis
  • Leiomyoma:
    • Positive for muscle markers; negative for inhibin and other sex cord stromal markers
  • Neurofibroma:
    • Patchy S100 positive; negative for sex cord stromal markers
  • Fibrous pseudotumors:
    • Less cellular, more inflammatory cells
  • Solitary fibrous tumor:
    • CD34 positive; tends to arise from extratesticular tissues like tunica albuginea, tunica vaginalis, gubernaculum, not testicular stromal cells
  • Unclassified sex cord stromal tumors:
    • Show more than focal incomplete Sertoli cell differentiation; strong positive S100 is useful stain
  • Myoid gonadal stromal tumor:
    • Rare, usually well circumscribed tumor composed of densely packed spindle cells with variable intervening collagen deposits; the nuclei are tapered with inconspicuous nucleoli
    • Positive for SMA, S100, FOXL2 and SF1 and negative for SOX9
  • Granulosa cell tumor:
    • Nodular growth pattern with cells showing nuclear grooves; luteinization of cells and mitotic figures are rare
    • FOXL2 mutations can be seen
    • Reticulin staining shows fibers surrounding aggregates of cells instead of individual cells as in fibrothecoma
Board review style question #1

A 65 year old man presented with a painless mass in his right testis. An ultrasound demonstrated a solid mass, measuring 5.4 cm in greatest dimension. He was scheduled for a radical orchiectomy. The resected tumor's histology is depicted in the image above. What is the best course of action after this diagnosis?

  1. Chemotherapy
  2. Observation
  3. Orchiectomy
  4. Radiation
Board review style answer #1
B. Observation

Comment Here

Reference: Fibroma thecoma group
Board review style question #2
Which of the following immunohistochemical profiles best supports the diagnosis of a fibrothecoma of the testis?

  1. Calretinin positive, S100 negative, CD34 positive
  2. CD56 negative, inhibin negative, smooth muscle actin negative
  3. Inhibin positive, FOXL2 positive, MelanA negative
  4. MIC2 positive, FOXL2 negative, S100 negative
Board review style answer #2
C. Inhibin positive, FOXL2 positive, MelanA negative

Comment Here

Reference: Fibroma thecoma group

Fibrous pseudotumor
Definition / general
  • Diffuse or localized, reactive fibromatous proliferation involving epididymis, tunica or spermatic cord
  • May simulate a testicular tumor (Arch Surg 1978;113:814)
  • May cause bowel wall perforation in infants with meconium periorchitis
Terminology
  • Other terms used:
    • Fibroma
    • Nonspecific peritesticular fibrosis
    • Nodular fibrous periorchitis, chronic proliferative periorchitis, reactive periorchitis or pseudofibrous periorchitis
    • Fibrous periorchitis (for diffuse lesions)
    • Proliferative funiculitis
    • Nodular pseudotumor (if forms a mass)
    • Inflammatory fibropseudotumor or pseudotumor (if forms a mass)
    • Peritesticular fibromatosis
    • Fibrous mesothelioma
Sites
  • Typically involves tunica albuginea
Case reports
Radiology images

Images hosted on other servers:

MRI: polycyclic mass

Clinical images

Images hosted on other servers:

Intrascrotal mass

Gross description
  • Can show diffuse band-like thickening, often encasing the testis
  • Localized forms show single or multiple nodules, 2 mm to 9 cm
  • Cut surface is usually firm, whitish
Gross images

Images hosted on other servers:

Pedunculated mass

Well demarcated margins

Microscopic (histologic) description
  • Dense fibrosis, focal edema, lymphangiectasis and perivascular round cell infiltration
  • Active epididymitis and chronic interstitial orchitis present in some cases
  • Associated with fibrinous or hyalinized loose bodies in cavity
  • Composed primarily of spindle cells with variable hyalinization and collagenization
  • Variable component of lymphocytes, plasma cells, histiocytes and scattered eosinophils
  • Nuclear atypia is usually absent, and if present is usually focal and restricted to the more cellular areas with more florid granulation tissue-like proliferation
  • May have focal whorled arrangement, focal calcification and ossification
  • Usually no mitoses
  • No atypical mitoses
Microscopic (histologic) images

Images hosted on other servers:

Dense fibrotic tissue

Collagen rich hyalinized fibrotic tissue

CD3

IgG4

CD31

Differential diagnosis
  • Sarcomas: generally are more cellular, with atypical mitoses and pleomorphism

Germ cell neoplasia in situ
Definition / general
  • Common precursor of type II germ cell tumors (i.e., seminomas and most postpubertal nonseminomatous germ cell tumors of the testis) (Nat Rev Cancer 2019;19:522)
  • Neoplastic gonocyte-like cells with latent totipotent (naive) developmental potential, located in the spermatogonial niche of seminiferous tubules
Essential features
  • Precursor for a subset of adult germ cell tumors (in WHO classification, referred to as germ cell neoplasia in situ [GCNIS] associated)
  • Associated with uncorrected cryptorchidism, ambiguous genitalia, infertility and a prior history of contralateral postpubertal germ cell tumor or GCNIS
  • Presence in testicle from a patient with extragonadal germ cell tumors may represent a burnt out germ cell tumor
  • Difficult to diagnose in infantile / prepubertal testis due to morphologic and immunohistochemical overlap with normal or delayed maturation
  • Frequently demonstrate aneuploidy but lack isochromosome 12p seen in invasive adult germ cell tumors
Terminology
  • Germ cell neoplasia in situ (GCNIS) was introduced in the 2016 edition of WHO's Tumors of the Urinary System and Male Genital Organs
  • Intratubular germ cell neoplasia (ITGCN) (not recommended / obsolete)
  • Intratubular germ cell neoplasia of unclassified type (not recommended / obsolete)
  • Carcinoma in situ of the testis (not recommended)
ICD coding
  • ICD-O: 9064/2 - intratubular malignant germ cells
  • ICD-10: C62 - malignant neoplasm of testis
    • With relevant subcodes (e.g., C62.9 - malignant neoplasm of testis, unspecified whether descended or undescended)
  • ICD-11: 2C80.2 & XH8AD3 - germ cell tumor of testis & intratubular malignant germ cells
Epidemiology
  • Found at higher rates in patients with uncorrected cryptorchidism (2 - 4%), ambiguous genitalia (25%) or infertility (1%) (Int J Cancer 1999;83:815)
Sites
Pathophysiology
  • Thought to arise from incompletely differentiated primordial germ cells, which undergo whole genome duplication events followed by recurrent losses of chromosome arms and whole chromosomes, thus creating an aneuploid state with subsequent additional mutations (e.g., KIT, KRAS) in a subset (Br J Cancer 2001;85:213, Cell Rep 2018;23:3392)
    • KIT mutations that occur early (i.e., after genome duplication) are thought to result in a subset of seminomas with global genomic hypomethylation that do not differentiate to other histologic subtypes
  • Believed to represent the universal precursor of type II germ cell tumors, which represent ~95% of germ cell tumors in postpubertal males (e.g., seminoma, embryonal carcinoma, choriocarcinoma, some teratomas and yolk sac tumors)
  • Show overexpression of embryonic transcription factors that increase proliferation and suppress apoptosis
Etiology
Diagrams / tables

Images hosted on other servers:
GCNIS derived adult germ cell tumors

GCNIS derived adult germ cell tumors

Clinical features
  • Microlithiasis detected by ultrasonography in patients undergoing workup for infertility may be associated with a higher risk of GCNIS (especially if bilateral) (J Urol 2004;171:158)
  • Testicular biopsies can be offered to patients with microlithiasis and 1 additional risk factor for GCNIS (Andrology 2020;8:1736)
Diagnosis
  • Diagnosis requires histologic examination (testicular biopsy, orchiectomy)
  • Biopsy of contralateral testis in patients with adult germ cell tumor is more common in Europe than North America
Laboratory
Radiology description
Prognostic factors
Case reports
Treatment
  • Orchiectomy if not already performed or close surveillance with routine monitoring for progression
  • Chemotherapy does not reduce the risk of progression
  • Low dose radiotherapy may be considered for contralateral GCNIS in patients with localized adult germ cell tumor (Ann Oncol 1998;9:657, BMC Urol 2013;13:71, Oncology 2009;77:33)
Microscopic (histologic) description
  • Neoplastic cells located along the basement membrane of seminiferous tubules (spermatogonial niche)
  • Cells are large atypical gonocyte-like with abundant clear cytoplasm and large (10 - 11 μm) hyperchromatic nuclei with coarse chromatin, angulated borders and prominent nucleoli; these cells are similar in appearance as seminoma (Histopathology 1978;2:157)
  • Affected seminiferous tubules frequently have a thickened basement membrane / peritubular hyalinization and lack spermatogenic maturation
  • Can spread in pagetoid fashion into rete testis, along the plane between the rete epithelium and the basement membrane
Microscopic (histologic) images

Contributed by Stephanie Siegmund, M.D., Ph.D., Maria Tretiakova, M.D., Ph.D., Alexander Subtelny, M.D., Ph.D. and Michelle Hirsch, M.D., Ph.D.
Prominent, large atypical gonocytes Prominent, large atypical gonocytes Prominent, large atypical gonocytes

Prominent, large atypical gonocytes

Pagetoid GCNIS into rete testis Pagetoid GCNIS into rete testis

Pagetoid GCNIS into rete testis

Normal prepubertal testis

Normal prepubertal testis


GCNIS with adjacent microinvasion GCNIS with adjacent microinvasion

GCNIS with adjacent microinvasion

Podoplanin / D2-40

Podoplanin /
D2-40

Membranous D2-40 immunohistochemistry

Membranous
D2-40 IHC

Membranous KIT immunohistochemistry

Membranous KIT IHC

Nuclear OCT 3/4 immunohistochemistry

Nuclear OCT 3/4 IHC

Virtual slides

Images hosted on other servers:
Peripheral residual seminiferous tubules

Peripheral residual seminiferous tubules

Positive stains
Molecular / cytogenetics description
  • Aneuploid (polyploid) with gain of additional findings at the time of invasion (e.g., isochromosome 12p is seen in invasive disease but not in GCNIS) (Genes Chromosomes Cancer 2003;38:117)
  • Aneuploidy, absence of isochromosome 12p can be demonstrated by FISH, single nucleotide polymorphism (SNP) array or karyotype (Am J Surg Pathol 2020;44:e66)
Molecular / cytogenetics images

Images hosted on other servers:
Representative aneuploidy and chromosomal abnormalities

Representative aneuploidy and chromosomal abnormalities

Sample pathology report
  • Right testicle and spermatic cord, radical orchiectomy:
    • Testis with fibrosis, seminiferous tubular atrophy and scattered foci of intratubular germ cell atypia, consistent with germ cell neoplasia in situ (GCNIS)
    • No invasive tumor is identified
    • Spermatic cord, epididymis and rete testis are negative for tumor
    • No lymphovascular invasion is identified
    • AJCC classification (8th edition): pTis N0
Differential diagnosis
  • Infantile or prepubertal testis with gonocytes with normal or delayed maturation:
  • Intratubular seminoma:
    • Large atypical gonocyte-like cells filling the seminiferous tubules without invasion
  • Seminoma with microinvasion:
    • Subtle single cells and small clusters of large atypical gonocyte-like cells with foci of microinvasion beyond the basement membrane into testicular parenchyma
    • Immunostains (OCT 3/4, KIT / c-KIT / CD117, D2-40, PLAP) can highlight cells that may be overlooked on H&E
Board review style question #1

Which of the following tumors of the testicle is associated with the finding shown in the image above?

  1. Embryonal carcinoma
  2. Juvenile granulosa cell tumor
  3. Spermatocytic tumor
  4. Teratoma, prepubertal type
  5. Yolk sac tumor, prepubertal type
Board review style answer #1
A. Embryonal carcinoma. Germ cell neoplasia in situ (GCNIS) is associated with type II germ cell tumors (e.g., seminoma, embryonal carcinoma) and of the options listed embryonal carcinoma is the only option that fits this designation. Answers B, C, D and E are incorrect because GCNIS is not known to be associated with other testicular neoplasms such as sex cord stromal tumors (e.g., juvenile granulosa cell tumor), spermatocytic tumor or prepubertal germ cell tumors.

Comment Here

Reference: Germ cell neoplasia in situ
Board review style question #2

Which of the following histologic features is associated with germ cell neoplasia in situ (GCNIS)?

  1. Atypical appearing gonocytes filling seminiferous tubules and extending into stroma as single cells
  2. Presence of i(12p)
  3. Seminiferous tubules lined with germinal epithelium undergoing maturation from spermatogonia to mature spermatids towards the lumen
  4. Sertoli only pattern
  5. Thickened seminiferous tubule basement membrane
Board review style answer #2
E. Thickened seminiferous tubule basement membrane. Of the listed features, only a thickened basement membrane is associated with germ cell neoplasia in situ (GCNIS). Answer D is incorrect because a Sertoli only pattern would be found in testicles lacking germ cells and possessing only Sertoli cells (as in some types of infertility). Answers A and C are incorrect because atypical gonocytes invading the interstitium between seminiferous tubules would be diagnostic of invasive disease, while a population of maturing germinal epithelium ranging from spermatogonia to mature spermatids in the lumen of the tubule would be consistent with nonneoplastic, maturing spermatogenesis and unlikely in the context of GCNIS. Answer B is incorrect because the presence of i(12p) indicates molecular progression beyond in situ disease (e.g., seminoma) and is therefore absent in the precursor GCNIS lesion.

Comment Here

Reference: Germ cell neoplasia in situ

Gonadoblastoma
Definition / general
  • Rare benign gonadal tumor composed of germ cells and sex cord stromal derivatives resembling immature granulosa cells and Sertoli cells
Terminology
Epidemiology
  • Most cases occur within first 2 decades; nearly 94% by age 30
  • Those with with partial dysgenesis / mosaicism are diagnosed prior to puberty; those with complete androgen insensitivity are diagnosed after puberty
  • Almost always associated with disorders of sex development
  • ~10% with dysgenetic gonads develop gonadoblastoma / seminoma by age 10 (Obstet Gynecol Surv 1986;41:74)
  • ~25% with mixed gonadal dysgenesis and Y component develop gonadoblastoma and germ cell tumor by age 40 (Am J Obstet Gynecol 1976;124:293)
Sites
  • Usually abdominal / inguinal undescended gonads / testis
  • May occur in ovaries of phenotypically and genetically normal females (see ovary tumor chapter)
Pathophysiology and etiology
Disorders of sexual development (DSD) clearly associated with increased risk of gonadoblastoma include:
  • Complete androgen insensitivity (46,XY) (30 - 66% incidence)
  • Pure gonadal dysgenesis / Swyer syndrome (46,XY)
  • Mixed gonadal dysgenesis (45,X / 46,XY) (55% incidence)
  • Turner syndrome (45,XO) with XY mosaicism
    • Molecular presence of Y chromosome (6 - 9% incidence) results in 43% incidence of gonadoblastoma
  • Frasier syndrome (rare 46,XY DSD caused by mutation in WT1 gene)
  • Maternal exposure to androgens, drugs, alcohol or illness during first trimester is associated with gonadal dysgenesis and intersex disorders; however, association with gonadoblastoma development is not established
Clinical features
  • ~80% are phenotypic females with subtle abnormalities in sexual development; cases with pure gonadal dysgenesis can be completely normal females
    • Patients with clitoromegally, abnormal hirsutism, etc. are candidates for karyotype analysis to exclude intersex disorders
  • ~20% are phenotypic males with gynecomastia, hypospadias, cryptorchidism; usually 46XY or 45X / 46XY
  • 1/3 have bilateral gonadoblastoma
  • Commonly associated with primary amenorrhea, developmental delay of genitalia and secondary sexual characters
  • 50% have abdominal mass with malignant transformation into seminoma or with metastatic disease
  • Gonadoblastoma not identified during neonatal period may go undetected until puberty when patients present with primary amenorrhea
  • Features of intersex disorders:
    • Presence of inguinal hernia in phenotypically female neonates associated with male pseudohermaphrodites in 1.6%
  • Bimanual abdominal palpation can be used to confirm presence of uterus in initial few days of neonatal life (due to effects of maternal hCG)
  • Patients with complete androgen insensitivity / male pseudohermaphroditism not diagnosed during neonatal period usually go unnoticed until puberty
    • Normal breast development but secondary sexual characters like pubic and axillary hair growth are rudimentary; vagina may be hypoplastic
  • Patients with mixed gonadal dysgenesis (45XO / 46XY) have
    • Ambiguous genitalia with varying degrees of phallic enlargement
    • Undescended testis
    • Urogenital sinus with labioscrotal fusion
    • Nearly all have a uterus, vagina and fallopian tubes in addition to an ovary / streak and a contralateral testicle
    • 50% are short and ~33% appear similar to individuals with Turner syndrome
Laboratory
  • Karyotyping is most important investigation for identifying those with intersex disorders who are at increased risk of developing gonadoblastoma
  • May have abnormalities in serum electrolytes, may be life threatening
  • Hormonal evaluation is necessary to rule out other causes of abnormal sexual development and to plan hormonal replacement
Prognostic factors
  • Excellent prognosis prior to malignant transformation, with no deaths reported
  • 30% of patients have malignant germ cell tumor in dysgenetic or contralateral gonad
  • Cure rates > 80% have been reported, including those with seminomatous overgrowth or metastases
Case reports
  • 10 year old girl, Turner mosaic with Y chromosome material identified by restriction fragment analysis and unilateral microscopic gonadoblastoma (Am J Clin Pathol 1988;90:622)
  • 12 year old girl with minute Y chromosome derived marker (J Med Genet 1992;29:542)
Treatment
  • Some cases undergo spontaneous involution
  • Treatment of choice is excision of gonadoblastoma and dysgenetic gonads prior to neoplastic transformation; however, timing of surgery varies with underlying disorder of sexual development
    • For mixed gonadal dysgenesis and mosaicism, surgery recommended before puberty, as early as possible
    • For complete androgen insensitivity (pure gonadal dysgenesis), surgery recommended after completion of puberty but before age 20:
      • Allows development of female breast by peripheral conversion of testosterone by aromatase into estrogens
      • Incidence of gonadoblastoma or other germ cell tumors before age 20 is negligible in these patients
      • Patients are usually diagnosed at puberty when they present with primary amenorrhea
Gross description
  • Tumor size varies from microscopic to 8 cm
  • Yellow to tan nodules with gritty cut surface
  • No hemorrhage or necrosis unless transformation to malignant germ cell tumor
Gross images

Images hosted on other servers:

Gonadoblastoma and dysgerminoma in gonadal dysgenesis

Microscopic (histologic) description
  • Three growth patterns
    1. Most common: round / irregular clusters of immature Sertoli cells and germ cells surrounded by basement membranes, with Sertoli cells encircling rounded hyaline nodules
    2. Sertoli cells surround large germ cells
    3. Germ cells occupy center of nests with peripheral ring of Sertoli cells
  • Stroma may contain large polygonal cells indistinguishable from Leydig cells: common in postpubertal patients
  • Calcification common: may be focal involving hyaline bodies or extensive
  • 50% have coexisting or subsequent seminoma
  • 8 - 10% develop other germ cell malignancies (Am J Obstet Gynecol 1972;113:410)
Microscopic (histologic) images

Images hosted on other servers:

Gonadoblastoma

Female patient

Positive stains
Electron microscopy description
  • Some cells show Charcot-Böttcher filaments of Sertoli cells
Molecular / cytogenetics description
  • Germ cells are aneuploid (Histopathology 1997;30:177)
  • Tsuchiya identified gonadoblastoma locus near Y centromere; TSPY gene is most likely candidate
    • TSPY is expressed in adult spermatogonia and fetal gonocytes and has vital functions in male stem / germ cell proliferation
    • It is ectopically expressed in gonadoblastoma, seminoma, ITGCN, some nonseminomatous germ cell tumors
    • TSPY expression correlates with PLAP, c-kit, OCT3 / 4
  • Seminomas and nonseminomatous tumors arising within dysgenetic gonad are usually diploid, unlike those arising from normal gonad (Genes Chromosomes Cancer 1999;25:134, Cancer Genet Cytogenet 1991;57:219, Cancer Res 1998;58:3105)
Differential diagnosis
  • Sertoli cell tumor with nodules with entrapped germ cells in a may be mistaken for a gonadoblastoma

Granulomatous lesions of testis and paratestis
Definition / general
  • Rare; usually men 40 - 59 years with sudden onset of tender testicular mass, variable fever
  • May be a response to acid fast products of disintegrated sperm, postinfectious or due to trauma or sarcoidosis
  • Resembles pyogenic epididymo-orchitis
  • Benign, although granulomatous inflammation may be associated with seminoma
  • Recommend cultures to rule out infectious process (brucellosis, leprosy, sarcoidosis, syphilis, TB)
  • Granulomatous ischemic lesion
    • Usually affects head of epididymis
    • May be due to ischemia with secondary granulomatous reaction and scarring
  • Gross description
    • Solid, unilateral nodular enlargement of testis; resembles lymphoma
  • Microscopic (histologic) description
    • Lymphocytes and plasma cells infiltrate interstitium and surround seminiferous tubules
    • Giant cells and histiocytes that resemble (but are not) actual granulomas
    • Granulomatous ischemic lesion
      • Zone of necrosis involving efferent ducts and interstitial connective tissue, with adjacent lymphocytes and macrophages
      • Macrophages form large clusters with cholesterol crystals and foreign body type giant cells in duct lumen
      • Also intratubular epithelial regeneration and proliferation of small ducts showing epithelial regeneration and numerous spermatozoa in their lumen
      • Associated with ceroid granuloma, spermatic granuloma and epidermoid metaplasia of the efferent ducts
  • Reference: Am J Surg Pathol 1997;21:951
Infectious causes
AIDS
  • Associated with markedly reduced spermatogenesis, arrested maturation, germ cell aplasia, tubular hyalinization / thickening of basement membranes, interstitial inflammation and fibrosis, reduction in Leydig cells, Sertoli cell only pattern (J Pathol 1991;163:47, Urology 1999;53:203, Mod Pathol 1989;2:233, Hum Pathol 1989;20:210)
  • Often other infections in testis or epididymis (Candida, CMV, Histoplasma, mycobacteria, toxoplasmosis)
  • Testicular atrophy related findings do not appear to be immune mediated (Hum Pathol 1989;20:572)
  • Testis is an uncommon location for Kaposi sarcoma in AIDS patients

Brucellosis
  • Zoonotic infection acquired from sheep, camels, cattle, dogs, goats, reindeer, swine via skin / mucous membrane contact or contaminated animal products
  • Affects testis and epididymis (epididymo-orchitis) in 2 - 20% of cases, causing scrotal pain, swelling, fever
  • Often diagnosed by laboratory studies
  • Case report: 32 year old man with painless testicular mass-brucellosis (Int J Urol 2004;11:683)
  • Micro description: granulomatous or testicular abscess
  • Treatment: antibiotics; orchiectomy if resembles a neoplasm or refractory to therapy
  • Additional references: Urol Int 2009;82:158, BMC Res Notes 2011;4:286, Clin Infect Dis 2001;33:2017

E. coli related pyogenic epididymo-orchitis
  • Usually due to E. coli
  • Resembles granulomatous orchitis
  • Complications: venous thrombosis, septic testicular infarct

Gonorrhea
  • Usually spreads from posterior urethra to prostate, seminal vesicles and epididymis
  • Testis involved only if untreated

Histoplasma capsulatum
  • Rarely presents as testicular mass
  • May resemble sperm granuloma (J Clin Pathol 1974;27:929)
  • Caseating granulomatous inflammation with giant cells
  • Small yeast forms (2 - 5 micrometers) are identifiable by silver stain (J Urol 2000;164:1652)

Leprosy
  • Does not occur in U.S.
  • Rarely presents with orchitis (Am J Clin Pathol 1980;73:712)
  • Testicular involvement thought to be facilitated by lower temperature of scrotum
  • 3 phases of testicular involvement
    1. Vascular phase: blood vessels show perivascular lymphocytic inflammation and interstitium is filled with macrophages containing mycobacteria
    2. Interstitial phase: endarteritis, Leydig cell clusters, interstitial fibrosis, histiocytes containing acid fast bacteria and reduced spermatogenesis
    3. Obliterative phase: dense fibrosis, no detectable tubules, reduced vessels, rare acid fast bacteria; associated with gynecomastia and infertility

Mumps
  • Testicular infections rare in infected children (prepubertal) but occur in 15 - 40% of postpubertal men one week after parotiditis
  • Usually unilateral (bilateral in 15 - 30%); epididymitis is also common (85%) and often precedes orchitis
  • 33% of infected postpubertal men develop testicular atrophy, 2 - 10% become infertile
  • Incidence increasing, due to reduced use of vaccine (BJU Int 2010;105:1060)

Syphilis
  • Testis usually involved first
  • Discrete gummas contribute to enlarged, irregular testis
  • Gummas: diffuse interstitial inflammation with edema, lymphocytes and plasma cells, with obliterative endarteritis and perivascular cuffing
  • Spirochetes usually identified in gummatous but not fibromatous stages

Tuberculosis
  • Usually begins in epididymis and spreads to testis
  • Prostate and seminal vesicles are usually also infected
BCG therapy
[Pending]
Sperm granuloma
[Pending]
Malakoplakia
[Pending]
Sarcoidosis
[Pending]
Iatrogenic (injectables)
[Pending]
Nonspecific / unknown cause
[Pending]
Gross images

Contributed by Yale Rosen, M.D. and @SueEPig on Twitter

TB orchitis

Granulomatous orchitis

Granulomatous orchitis



Images hosted on other servers:

Brucellosis

Microscopic (histologic) images

Contributed by Sean R. Williamson, M.D. and @SueEPig on Twitter

Tuberculosis involving testis and paratestis

Granulomatous orchitis Granulomatous orchitis Granulomatous orchitis

Granulomatous orchitis

Differential diagnosis

Grossing and features to report
Definition / general
  • Spermatic cord margin (sample before cut into tumor since contamination is a common problem) (Mod Pathol 1996;9:762)
  • Tumor (1 - 2 section per cm, representing all grossly different features)
  • Tumor and tunica
  • Tumor and hilum (site of extratesticular extension in > 90%) (Am J Clin Pathol 1999;111:534)
  • Tumor and normal testis
  • Normal testis
  • Epididymis

Hernia sac with mesothelial entrapment
Definition / general
  • Entrapment of mesothelial cells within hernia sac fibroadipose tissue
Essential features
Epidemiology
Pathophysiology
  • Persistent serosal injury causes reactive hyperplasia of the mesothelial lining and submesothelial fibrosis, entrapping mesothelial cells beneath the mesothelial lined surface (Mod Pathol 2005;18:S131, Diagn Pathol 2011;6:78)
Clinical features
  • Similar clinical presentation as a patient with a hernia sac without mesothelial entrapment (usually hydrocele, also hematocele, cyst adjacent to epididymis, hemorrhagic epididymal cyst, pyocele) (Am J Surg Pathol 2014;38:54)
Prognostic factors
Treatment
  • Similar treatment as a hernia without mesothelial entrapment, i.e. excision of hernia sac (J Urol 2011;186:1620)
Gross description
Microscopic (histologic) description
Microscopic (histologic) images

Contributed by Debra L. Zynger, M.D.

Entrapped glands

Cords, glands and small nests

Gland formation

Entrapped mesothelium

WT1

Calretinin

Negative stains
Molecular / cytogenetics description
Differential diagnosis
Board review style question #1

    Which of the following is a feature of a hernia sac with mesothelial entrapment?

  1. Glands parallel to surface
  2. GLUT1 and p53 positivity
  3. Homozygous for 9p21 deletion
  4. Invasion into muscle
  5. Prominent mitotic activity
Board review style answer #1
A. Glands parallel to surface

Comment Here

Reference: Hernia sac with mesothelial entrapment
Board review style question #2

    A 48 year old man presents with a hydrocele with the findings in the above picture. What is your diagnosis?

  1. Adenomatoid tumor
  2. Diffuse malignant mesothelioma
  3. Hernia sac with mesothelial entrapment
  4. Histiocytic inflammation
  5. Metastatic adenocarcinoma
Board review style answer #2
C. Hernia sac with mesothelial entrapment

Comment Here

Reference: Hernia sac with mesothelial entrapment

Hydrocele
Definition / general
  • Accumulation of serous fluid between visceral and parietal layers of tunica vaginalis
Essential features
  • Accumulation of serous fluid between visceral and parietal layers of tunica vaginalis
  • Idiopathic or associated with nonneoplastic or neoplastic lesions
  • Note: thorough macroscopic examination and extensive sampling are needed to rule out mesothelioma (Singapore Med J 2015;56:e53)
ICD coding
  • ICD-10: N43.3 - hydrocele, unspecified
  • ICD-11: GB00 - hydrocele or spermatocele
Epidemiology
Sites
  • Scrotum
  • Rarely, the hydrocele sac may extend beyond the scrotum to the abdomen via the inguinal canal (abdominoscrotal hydrocele) (Urol Case Rep 2020;32:101254)
Pathophysiology

Encysted variant
Funicular variant
Persistent communication
with the peritoneal cavity
No Yes
Etiology Defective closure at both
proximal and distal ends of
processus vaginalis
Defective closure of the distal
end of tunica vaginalis
Macroscopic differential
diagnosis
Inguinal lymphadenopathy,
hernia, tumor of the
spermatic cord
Reference: Acta Radiol 2007;48:1138
Etiology
  • Mostly idiopathic, putative causes: excessive secretion or decreased reabsorption of fluid by parietal mesothelial cells, congenital lack of efferent lymphatics
  • Association with inguinal hernia, scrotal trauma, inflammation (epididymoorchitis) or tumors of the testis / paratestis (BJU Int 2011;107:1852)
  • Filarial hydrocele (StatPearls: Filarial Hydrocele [Accessed 2 February 2023]):
    • Late and chronic manifestation of filariasis
    • Very common in endemic areas (tropical and subtropical countries)
    • Due to parasite induced blockage and dysfunction of the lymphatic vessels
Clinical features
Diagnosis
  • Usually diagnosed by physical examination and transillumination
Radiology description
  • Simple fluid collection at ultrasound; may contain septations, calcifications or cholesterol (Radiographics 2009;29:2017)
  • Avascular at Doppler evaluation
  • MRI: low signal on T1, high signal on T2 weighted images
Radiology images

Images hosted on other servers:

Scrotal MRI: cystic lesion

Encysted hydrocele

Case reports
Treatment
Gross images

Images hosted on other servers:

Abdominoscrotal hydrocele

Microscopic (histologic) description
  • Loose connective tissue lined by a single layer of cuboidal or flattened mesothelial cells
  • Lining may show mesothelial hyperplasia (both solid and papillary), squamous metaplasia or prominent atypia
  • Usually clear luminal fluid
  • Fibrinous exudate, chronic inflammatory infiltrate and fibrosis in longstanding cases, due to infection or hemorrhage (Cheng: Urologic Surgical Pathology, 4th Edition, 2019)
  • Occasional presence of florid nodular collections of histiocytes and aggregates of incidental benign small blue cells of possible rete epithelial origin (Hum Pathol 2010;41:88, Am J Surg Pathol 2016;40:1507)
Microscopic (histologic) images

Contributed by Francesca Sanguedolce, M.D., Ph.D.
Connective tissue, mild inflammation

Connective tissue, mild inflammation

Fibrosis, moderate inflammation

Fibrosis, moderate inflammation

Mesothelial lining

Mesothelial lining

Calretinin

Calretinin

Sample pathology report
  • Left testis, hydrocelectomy:
    • Hydrocele
Differential diagnosis
  • Testis tumor:
    • Solid mass at transillumination
  • Mesothelioma:
  • Spermatocele:
    • Cystic dilatation of the epididymis, efferent ductule or proximal rete testis
    • Negative for mesothelial markers
    • Spermatozoa and proteinaceous fluid in the lumen
    • Often ciliated epithelial lining
Board review style question #1

Which cell type lines a hydrocele?

  1. Endothelial cells
  2. Germinal cells
  3. Mesothelial cells
  4. Urothelial cells
Board review style answer #1
C. Mesothelial cells. Since hydrocele is defined as an accumulation of serous fluid between visceral and parietal layers of tunica vaginalis, its lining is provided by mesothelial cells.

Comment Here

Reference: Hydrocele
Board review style question #2
Which microscopic findings can occur in a hydrocele?

  1. Acute inflammation, mucinous metaplasia
  2. Chronic inflammation, fibrosis, squamous metaplasia
  3. Clear cell hyperplasia
  4. Psammoma bodies
Board review style answer #2
B. Chronic inflammation, fibrosis, squamous metaplasia. Longstanding hydrocele may be complicated by inflammation and hemorrhage, resulting in abnormal findings involving both the lining cells and the connective tissue.

Comment Here

Reference: Hydrocele

Infertility
Hypospermatogenesis
Definition / general
  • All stages of spermatogenesis present but reduced to a varying degree
  • Includes the mixed pattern with some tubules showing Sertoli cells only or hyaline sclerosis with others tubules containing complete spermatogenesis
  • Mild if changes in occasional tubules; severe if significant reductions in all tubules, moderate in between
  • Often is thickening of tunica propria, interstitial fibrosis, tubular sclerosis, germ cell disorganization and sloughing into lumina
  • Many causes of infertility induce hypospermatogenesis, such as diabetes mellitus, toxins, excess heat, varicocele, hypothyroidism, irradiation and others

Additional references
Infertility
Definition / general
  • Causes: pretesticular, testicular, posttesticular
  • Pretesticular: extragonadal endocrine disorders (hypothalamic, pituitary, adrenal); includes elevated prolactin levels (Arch Pathol Lab Med 1984;108:35)
  • Testicular: little treatment currently available
  • Posttesticular: duct obstruction (congenital, inflammatory, postsurgical); surgical treatment often successful since spermatogenesis is normal
    • Impaired sperm motility due to epididymal or immunologic factors is considered posttesticular
  • Evaluation: history and physical examination, semen analysis, white blood cell count in semen, detection of antisperm antibodies, sperm function tests (cervical mucus interaction, ova penetration, hemizonal assay)
  • Testicular biopsy is helpful for azoospermia without endocrine abnormalities
  • RNA binding motif: nuclear immunostain identifies spermatogenesis in biopsies that appear to be Sertoli cell only (Hum Pathol 2001;32:36)

Additional references
Maturation arrest
Definition / general
  • Complete maturation arrest: germ cell maturity ceases at a specific point frequently at primary spermatocyte level; sperm counts usually zero
  • Incomplete maturation arrest: similar to complete but a few late spermatids are present in a few seminiferous tubules, some prefer the term hypospermatogenesis instead of incomplete maturation arrest; patients are usually oligospermic (Arch Pathol Lab Med 2010;134:1197)
  • Same etiologies as hypospermatogenesis (diabetes mellitus, toxins, excess heat, varicocele, hypothyroidism, irradiation); also postpubertal gonadotropin deficiency, alkylating agents
  • Nonzero sperm counts indicate late spermatids are present somewhere in testis, although perhaps not in area biopsied
  • Patients with early maturation arrest have a greater incidence of genetic anomalies and are more likely to have worse reproductive outcomes than are patients with late maturation arrest (Urology 2012;80:826)

Microscopic (histologic) description
  • Numerous spermatogonia, few spermatocytes, no mature spermatozoa
  • Sertoli cells prominent since reduced germ cells
  • Tubules often contain degenerated cells with irregular dense nuclei

Microscopic (histologic) images

Contributed by Asmaa Gaber Abdou, M.D.

Various images


Inflammatory pseudotumor
Definition / general
  • Synonyms: chronic proliferative periorchitis, inflammatory pseudotumor, nodular and diffuse fibrous proliferation, paratesticular fibrous pseudotumor
  • Ages 7 - 95 years, peaks in 20's
  • Reactive fibrous mass, usually between testicular tunica layers; also involves epididymis and spermatic cord, associated with hydrocele, infection, trauma (Am J Surg Pathol 2010;34:569)
  • Some cases may be type of IgG4 associated sclerosing disease (Virchows Arch 2011;458:109, Hum Pathol 2012;43:2084)
Case reports
  • 22 year old man with collagen rich stromal tumor attached to testicular tunica albuginea (Hum Pathol 1990;21:866)
Treatment
  • Excision
Gross description
  • Multinodular thickening of peritesticular tissue or discrete mass of firm white tissue up to 15 cm
  • May be an associated hydrocele or hematocele
Microscopic (histologic) description
  • Dense fibrous tissue, fibroblasts, inflammatory cells, dystrophic calcification
  • With time, cellularity decreases and fibrosis increases
  • Usually no necrosis, no pleomorphism, no / rare mitotic activity (Am J Surg Pathol 2010;34:569)
Positive stains
Negative stains
Differential diagnosis

Intratubular large cell hyalinizing Sertoli cell neoplasia
Definition / general
Essential features
  • Benign, multifocal and often bilateral testicular lesion
  • Found in prepubertal males, usually associated with gynecomastia
  • Associated with Peutz-Jeghers syndrome
Terminology
  • Accepted: intratubular large cell hyalinizing Sertoli cell neoplasia or intratubular large cell hyalinizing Sertoli cell tumor
  • Historically: sex cord tumor with annular tubules (not recommended)
ICD coding
  • ICD-O: 8640/1 - Sertoli cell tumor, NOS
  • ICD-10: D29.20 - benign Sertoli cell tumor of testis
  • ICD-11: unknown
Sites
  • Seminiferous tubules
Pathophysiology
  • Unknown
Etiology
Clinical features
Diagnosis
Laboratory
  • Increased estradiol level
Radiology description
Prognostic factors
  • Considered a benign condition, unknown what makes rare cases invasive
Case reports
Treatment
  • Aromatase inhibitor therapy initial treatment
  • Orchiectomy for nonresponders with continued / progressive clinical estrogen symptoms (Am J Surg Pathol 2007;31:827)
Gross description
Microscopic (histologic) description
Microscopic (histologic) images

Contributed by Vikas Mehta, M.D. and @katcollmd on Twitter

Seminiferous tubules

Basement membrane deposition

Calretinin

Inhibin A

Collagen IV

PAS


Intratubular large cell hyalinizing Sertoli cell neoplasia Intratubular large cell hyalinizing Sertoli cell neoplasia Intratubular large cell hyalinizing Sertoli cell neoplasia Intratubular large cell hyalinizing Sertoli cell neoplasia

Intratubular large cell hyalinizing Sertoli cell neoplasia

Negative stains
  • Not well studied in literature
Molecular / cytogenetics description
Sample pathology report
  • Testis, right, core needle biopsy:
    • Intratubular large cell hyalinizing Sertoli cell neoplasia
Differential diagnosis
Board review style question #1

What diagnosis is associated with a 4 year old patient who presents with pigmented macules around the lips and oral mucosa, gynecomastia and the following findings on testis biopsy?

  1. Gonadoblastoma
  2. Intratubular large cell hyalinizing Sertoli cell neoplasia
  3. Large cell calcifying Sertoli cell neoplasm
  4. Sertoli cell nodule
Board review style answer #1
B. Intratubular large cell hyalinizing Sertoli cell neoplasia. The patient has Peutz-Jeghers syndrome and the testicle shows intratubular large cell hyalinizing Sertoli cell neoplasia. The image shows a tubule composed of large Sertoli cells without normal spermatogonia and spherules of basement membrane-like material characteristic of this entity.

Comment Here

Reference: Intratubular large cell hyalinizing Sertoli cell tumor
Board review style question #2
Which of the following symptoms is most associated with intratubular large cell hyalinizing Sertoli cell neoplasia?

  1. Gynecomastia
  2. Mucosal ulcers
  3. Normal age of puberty
  4. Short stature
Board review style answer #2
A. Gynecomastia. This is a symptom seen due to the elevated serum estradiol level in these patients.

Comment Here

Reference: Intratubular large cell hyalinizing Sertoli cell tumor

Juvenile granulosa cell tumor
Definition / general
  • Neoplasm composed of primitive appearing granulosa cells growing in solid and follicular patterns
  • Most common neoplasm of the testis during the first 6 months; almost all tumors are seen in first decade of life (J Cancer Res Clin Oncol 2020;146:2829)
  • No malignant cases reported
Essential features
  • Juvenile granulosa cell tumor (JGCT) is a rare neoplasm with a wide morphologic spectrum
  • Solid and reticular patterns may pose diagnostic challenges but the lobular appearance and follicular differentiation are characteristic
  • Immunostains aid in diagnosis with positive expression of SF1, calretinin, vimentin and inhibin and negative expression of alpha fetoprotein (AFP)
ICD coding
  • ICD-O: 8622/1 - granulosa cell tumor, juvenile
  • ICD-11: 2F34 & XH2U25 - benign neoplasm of male genital organs & granulosa cell tumor of the testis, juvenile
Epidemiology
Sites
  • Almost invariably unilateral
Pathophysiology
  • Thought to arise from a specialized gonadal stromal cell of the testicle, which is histologically very similar to JGCT of the ovary
Clinical features
  • Most commonly seen as testicular mass (65%) or enlarging testis (25%)
  • Testicular masses are asymptomatic and can be present within the abdomen or inguinal region or within the scrotal sac
  • Can also present with lack of endocrine features
  • Some cases present with the contralateral testis as clinically normal but undescended
Diagnosis
  • Imaging: testicular ultrasonography usually reveals a well defined large intratesticular multicystic and solid mass
  • Pathological findings after orchiectomy are conclusive
Laboratory
  • Serum alpha fetoprotein (AFP) and beta human chorionic gonadotropin (HCG) are within normal range and do not aid in diagnosis
Radiology description
  • Multiseptated, cystic intratesticular / intra-abdominal mass observed on ultrasonography
Radiology images

Images hosted on other servers:
Well encapsulated cystic mass

Well encapsulated cystic mass

Prognostic factors
  • Favorable prognosis, with no documented metastasis or relapses
  • Lymphovascular invasion has been documented in only 2 cases and involvement of rete testis in 4 cases (Am J Surg Pathol 2015;39:1159)
Case reports
Treatment
  • In circumstances where organ preservation is not possible, total orchiectomy is performed
Gross description
  • Tumors measure 0.5 - 5 cm (mean: 2 cm; median: 1.5 cm)
  • Well circumscribed and yellow-orange or tan-white on cut surface
  • Can have a solid (33% cases) or cystic component (67% cases) or both (Am J Surg Pathol 2015;39:1159)
  • Watery to mucoid cystic fluid
Gross images

Images hosted on other servers:
Testicle after surgery

Testicle after surgery

Microscopic (histologic) description
  • Solid, nodular or follicular patterns are the most common and sometimes alternate
  • Follicles vary from large and round / oval to small and irregular containing basophilic or eosinophilic secretion (mucicarmine+)
  • In solid or nodular areas, the cells grow in sheets or irregular clusters
  • Cells have moderate to large amounts of pale to eosinophilic cytoplasm, round to oval hyperchromatic nuclei, some of which contain nucleoli
  • Mitoses are common, in contrast to adult granulosa cell tumor
  • Extensive hyalinization (forming nodules) and prominent myxoid background may be seen
  • Call-Exner bodies are uncommon
  • Apoptosis, entrapped seminiferous tubules, rete testis involvement and lymphovascular invasion could be seen in rare cases (Am J Surg Pathol 2015;39:1159)
Microscopic (histologic) images

Contributed by Vikas Mehta, M.D.
Variable sized follicles

Variable sized follicles

Follicles with bland cells

Follicles with bland cells

Tumor cells with eosinophilic cytoplasm

Tumor cells with eosinophilic cytoplasm

Round to oval hyperchromatic nuclei

Round to oval hyperchromatic nuclei

Positive stains
Molecular / cytogenetics description
Sample pathology report
  • Left testis, radical orchiectomy:
    • Juvenile granulosa cell tumor (1.5 cm), confined to the testis (see comment)
    • Comment: The tumor is small and well circumscribed and shows variable sized follicles lined by bland appearing, oval, round cells arranged in single or multiple layers with outer layers resembling theca cells. Many mitotic figures are identified. No nuclear grooves or Call-Exner bodies identified.
Differential diagnosis
  • Yolk sac tumor:
    • High elevations of AFP
    • Presence of Schiller-Duval bodies
    • True follicles are diagnostic of JGCT and are not a feature of yolk sac tumor
    • Negative for SF1, FOXL2, inhibin A, calretinin
  • Teratoma:
    • Numerous ectodermal, mesenchymal and endodermal tissues occur
    • Virtually all elements lack cellular atypia
  • Cystic dysplasia of the testicle (Front Pediatr 2022;10:898038):
    • Rare and benign cause of testicular swelling in pediatric population
    • Irregular cystic spaces in the mediastinum of the testis, displacing the testicular parenchyma
    • Cysts lined with flattened cuboidal epithelium and separated by fibrous septa
    • Frequently associated with genitourinary anomalies, ipsilateral renal agenesia and multicystic dysplasia of the kidney
  • Sertoli cell tumor:
    • Small size, multifocal, monotonous tubular / cord architecture, associated with androgen insensitivity syndrome and cryptorchidism
  • Gonadoblastoma:
    • Almost always associated with disorders of sex development
    • Round nests and cords composed of germ cells, small sex cord cells and globoid deposits of hyaline basement membrane material
    • May also show Charcot-Böttcher crystals
  • Unclassified sex cord stromal tumors:
  • Rhabdomyosarcoma:
    • More typically in paratesticular location
    • Lacks follicle formation; small round blue cells with rhabdomyoblasts
    • Positive for desmin, MyoD1 and myogenin
Board review style question #1

A 5 month old boy presented with a left testicular mass. An ultrasound of the testis showed a variably solid cystic mass with internal septations. An image from the resection specimen is shown above. Which of the following features will point toward the diagnosis of juvenile granulosa cell tumor?

  1. Presence of intracellular or extracellular eosinophilic hyaline globules, basement membrane material and Schiller-Duval bodies
  2. Round nests and cords composed of germ cells, small sex cord cells and globoid deposits of hyaline basement membrane material
  3. Variable morphologic patterns with biphasic epithelioid and spindle cell components
  4. Variable sized true follicles lined by bland appearing, oval, round cells arranged in single or multiple layers with outer layers resembling theca cells
Board review style answer #1
D. Variable sized true follicles lined by bland appearing, oval, round cells arranged in single or multiple layers with outer layers resembling theca cells. Answer A is incorrect because the features described are found in yolk sac tumors. Answer C is incorrect because variable morphologic patterns with biphasic epithelioid and spindle cell components are found in a subset of sertoli cell tumors, NOS. Answer B is incorrect because the features described are found in gonadoblastoma.

Comment Here

Reference: Juvenile granulosa cell tumor
Board review style question #2
Which of the following choices represents the immunoprofile of juvenile granulosa cell tumor of testis?

  1. Desmin+, MyoD1+, myogenin+
  2. Glypican3+, SALL4+, OCT3/4+
  3. S100+, SMA+, SF1+
  4. SF1+, FOXL2+, inhibin+
Board review style answer #2
D. SF1+, FOXL2+, inhibin+. Answer B is incorrect because glypican3+, SALL4+, OCT3/4+ can be seen in immature teratoma. Answer C is incorrect because the features listed are consistent with gonadal stromal tumor, myoid. Answer A is incorrect because the features listed are consistent with rhabdomyosarcoma.

Comment Here

Reference: Juvenile granulosa cell tumor

Large cell calcifying Sertoli cell tumor
Definition / general
Essential features
  • Generally presents as a benign sex cord stromal tumor found predominantly in patients < 20 years old; a subset presents in older patients and may behave aggressively
  • Associated with Carney complex in 20 - 40% of cases due to alterations in the gene PRKAR1A
  • Characterized by intratubular and sheet-like growth of large eosinophilic cells with laminated psammomatous, mulberry-like or dystrophic calcifications, a prominent neutrophilic infiltrate and lymphocytic rim
ICD coding
  • ICD-10: D29.20 - Sertoli cell tumor, male
Epidemiology
  • Male patients, generally < 20 years old
  • 20 - 40% of cases associated with Carney complex due to mutations in PRKAR1A gene (part of Carney complex with spotty cutaneous pigmentation, primary pigmented nodular adrenocortical disease, atrial myxomas, superficial angiomyxomas, malignant melanocytic nerve sheath tumors, blue nevus and thyroid carcinoma)
  • Debatable association with Peutz-Jeghers syndrome (rare reports, may represent intratubular large cell hyalinizing Sertoli cell neoplasia) and neurofibromatosis (Am J Surg Pathol 2021 Dec 16 [Epub ahead of print])
Sites
  • Testis
  • Multifocal and bilateral testicular involvement in > 50% of patients with Carney complex (J Urol 2002;167:1299)
Clinical features
Diagnosis
  • Definitive diagnosis by histopathological examination
Prognostic factors
  • Kratzer et al. proposed malignant prognostic features that include age > 25 years old and 2 or more of the following adverse features (Am J Surg Pathol 1997;21:1271):
    • Size > 4 cm
    • Extratesticular extension
    • Mitotic index > 3/10 high power fields (HPFs)
    • Coagulative tumor necrosis
    • Vascular invasion
    • High grade cytologic atypia
  • Prepubertal status may provide a protective effect (Am J Surg Pathol 2021 Dec 16 [Epub ahead of print])
  • Tumors associated with Carney complex may behave in biologically indolent fashion compared with sporadic tumors (Eur Urol 2001;40:699)
Case reports
Treatment
  • Partial or radical orchiectomy for organ confined disease
  • Retroperitoneal lymph node dissection with possible adjuvant chemotherapy or radiation therapy for metastatic disease (benefit unknown)
Gross description
Gross images

Contributed by Sara Vargas, M.D.
Ill defined testicular nodules

Ill defined testicular nodules



Images hosted on other servers:

Firm, white tan nodules

Tumor

Microscopic (histologic) description
  • Laminated psammomatous and mulberry-like calcifications
  • Prominent neutrophilic infiltrate and lymphocytic rim
  • Large epithelioid cells with eosinophilic cytoplasm, prominent nucleoli and minimal cytologic atypia
  • Cords, trabeculae, nests or sheets in a variably myxoid or fibrous stroma
  • Generally limited to testis with occasional extension into paratesticular tissue, including rete testis and hilar soft tissue
  • Malignant cases more likely to exhibit solid sheet-like growth and frequent nuclear pleomorphism ranging from mild to prominent
Microscopic (histologic) images

Contributed by Stephanie Siegmund, M.D., Ph.D.
Calcifications, neutrophils, myxoid stroma

Calcifications, neutrophils, myxoid stroma

Focus of intratubular tumor

Focus of intratubular tumor

Tumor involving rete testis

Tumor involving rete testis

Mulberry calcifications

Mulberry calcifications

Trabecular architecture

Trabecular architecture


Focal nuclear pleomorphism

Focal nuclear pleomorphism

Neutrophilic infiltrate

Neutrophilic infiltrate

Minimal cytologic atypia

Minimal cytologic atypia

Mulberry calcifications

Mulberry calcifications

PRKAR1A loss by IHC

PRKAR1A loss

Virtual slides

Images hosted on other servers:
Multifocal mass in pediatric testis

Multifocal mass in pediatric testis

Dystrophic calcifications

Dystrophic calcifications

Negative stains
Molecular / cytogenetics description
Sample pathology report
  • Testis and spermatic cord, radical orchiectomy:
    • Large cell calcifying Sertoli cell tumor (1.0 cm) (see comment)
    • Tumor is limited to the testicular parenchyma
    • No necrosis is identified
    • Mitoses number < 1 per 10 HPFs
    • The spermatic cord and cord resection margin are negative for tumor
    • No lymphovascular invasion is identified
    • Background testis with maturing spermatogenesis
    • Comment: The slides demonstrate sheets and cords of large epithelioid cells with eosinophilic cytoplasm and prominent nucleoli. Immunohistochemistry demonstrates the following staining profile in lesional cells: positive staining for inhibin, calretinin, S100, MelanA and beta catenin (cytoplasmic, nonnuclear) and negative staining for PRKAR1A (loss), OCT 3/4 and SALL4. Overall, the findings are consistent with large cell calcifying Sertoli cell tumor. Adverse pathologic features are not identified (e.g., > 3 mitoses per 10 HPF, coagulative necrosis, vascular invasion, size > 4.0 cm, high grade cytologic atypia or extratesticular extension) (Am J Surg Pathol 1997;21:1271). Approximately 20 - 40% of cases are associated with Carney complex and genetic counseling is recommended in the appropriate clinical context.
Differential diagnosis
  • Leydig cell tumor (LCT):
    • Usually solitary and unilateral
    • Frequent Reinke crystals (diagnostic of LCT)
    • Lacks calcifications and neutrophilic infiltrate
    • Retains IHC expression of PRKAR1A
  • Sertoli cell tumor, not otherwise specified (NOS):
    • Similar population of large, polygonal Sertoli cells with eosinophilic cytoplasm
    • Usually lacks calcifications and neutrophilic infiltrate
    • Nuclear localization of beta catenin by IHC
    • Retains IHC expression of PRKAR1A
  • Intratubular large cell hyalinizing Sertoli cell neoplasia:
    • Predominantly intratubular process with expanded tubules and thickened basement membrane
    • Tubules filled by globular eosinophilic basement membrane-like material
    • Generally lacks calcifications
    • Retains IHC expression of PRKAR1A
Board review style question #1

Which of the following features is most consistent with sporadic presentation of large cell calcifying Sertoli cell tumor (LCCSCT), shown in the image, versus a syndromic association (e.g., Carney complex)?

  1. Bilateral disease
  2. Loss of PRKAR1A by IHC
  3. Multifocal disease
  4. Pediatric presentation
  5. Sibling(s) with similar presentation
Board review style answer #1
B. Loss of PRKAR1A by IHC. While PRKAR1A alterations are commonly found in patients with Carney complex, the loss of PRKAR1A is seen in most sporadic as well as syndromic LCCSCT and can frequently be demonstrated by as loss of PRKAR1A by IHC in sporadic tumors. All other listed answers (pediatric onset, bilateral and multifocal disease, sibling(s) with similar presentations) are more commonly associated with LCCSCT occurring in Carney complex patients and would suggest a syndromic association.

Comment Here

Reference: Large cell calcifying Sertoli cell tumor

Leiomyosarcoma
Definition / general
  • Malignant mesenchymal tumor with smooth muscle differentiation
Epidemiology
Sites
Pathophysiology
  • Hypothesized to arise from cremasteric muscle, vas deferens or contractile cells of other structures such as tunica or blood vessels (Urology 2002;60:1112)
  • Paratesticular tumors belong to "deep type" of leiomyosarcoma
  • No recurrent genetic abnormality has been documented
  • Tumor spreads via lymphatic and hematogenous metastases or by local extension
  • Lymphatic spread is to regional lymph nodes: external iliac, hypogastric, common iliac, retroperitoneal (J Urol 1970;103:628)
  • Lung is most common site for hematogenous spread
Clinical features
  • Typically presents with painless, slow growing mass (at least a few months) in scrotal sac
  • Rarely presents with sudden increase in size of long standing mass
  • Aggressive tumors may invade dartos muscle and overlying skin, causing a fungating mass
Laboratory
  • Normal serum tumor markers (LDH, β-HCG, α-fetoprotein) helps exclude a germ cell tumor
Radiology description
  • Sonography shows a mass with mixed echogenicity and increased vascularity (World J Radiol 2011;3:114)
  • Color Doppler ultrasonography reveals irregular vascularity, more dominant in periphery, similar to malignant tumors
  • CT usually shows absence of fat (absence of foci with negative Hounsfield units)
  • Variable degree of hydrocele may be present
Radiology images

Images hosted on other servers:

Mass posterior superior to right testis

Huge heterogeneous enhancing mass

Heterogenous echogenicity

Prognostic factors
  • Similar to sarcoma: tumor size, tumor grade, presence of necrosis, mitotic count, completion of resection, local recurrence, distant metastases
  • Specific prognostic criteria not well established due to rarity of tumor
  • FNCLCC grading (recommended over NIH system) gives points for:
    • Differentiation: 1, well differentiated leiomyosarcoma; 2, conventional leiomyosarcoma; 3, poorly differentiated / pleomorphic / epithelioid leiomyosarcoma
    • Mitotic count: 1, 0 - 9 per 10 high powered fields; 2, 10 - 19 per 10 high powered fields; 3, 20 or more per 10 high powered fields
    • Necrosis: 0, none; 1, less than or equal to 50%; 2, greater than 50%
    • Grade 1 is 2 - 3 points, grade 2 is 4 - 5 points and grade 3 is 6 - 8 points
Case reports
Treatment
  • Radical orchidectomy for resectable tumors
  • Simple excision is insufficient as residual disease was found in 27% of cases that underwent repeat wide excision (J Urol 1981;126:611)
  • Uncertain role of adjuvant radiation or chemotherapy; some authors recommend adjuvant therapy in high grade / high risk tumors and others recommend adjuvant radiotherapy for all grades to reduce local recurrence (Cancer 1996;77:1873, J Urol 1991;146:342)
  • Prophylactic retroperitoneal lymph node dissection may be performed but no survival benefit has been documented, so dissection recommended only if nodes are suspicious for involvement (J Urol 1970;103:628)
  • Single or multiagent chemotherapy with anthracyclines and if osfamides or gemcitabine for nonoperable or metastatic tumors
Clinical images

Images hosted on other servers:

Ulcerated and
fungating tumor
involving scrotum

Gross description
  • Tumor typically located outside tunica albuginea, may be centered on epididymis or spermatic cord
  • Cut surface is usually firm, gray white
  • Grossly identifiable necrotic and hemorrhagic areas may be seen
Gross images

Images hosted on other servers:

Well defined huge solid mass

Microscopic (histologic) description
  • Interlacing fascicles of spindle shaped cells with eosinophilic cytoplasm and cigar shaped nuclei
  • Mitotic activity is usually seen; necrosis may be seen
  • At least mild atypia; more undifferentiated cases have marked cellular atypia
  • Variably scattered mast cells, inflammatory cells, hyalinization
  • Histological variants:
    • Epithelioid
    • Dedifferentiated / pleomorphic
      • Discrete transition to more pleomorphic tumor (reminiscent of malignant fibrous histiocytoma) without any smooth muscle differentiation
      • Dedifferentiated areas are usually negative for all myogenic markers
      • Heterologous osseous or chondro-osseous elements may be seen rarely
    • With osteoclast-like giant cells (Urol Int 1996;56:259)
    • Myxoid (Am J Surg Pathol 2000;24:927)
    • Inflammatory
Microscopic (histologic) images

Case #254

Various images

Vimentin


Desmin

Alpha smooth muscle actin

PAS

PLAP



Images hosted on other servers:

Pattern of spindle cells - H&E

Mitotic activity and atypical nuclei


Desmin+, SMA+

Desmin+

SMA+

Immunohistochemistry & special stains
Electron microscopy description
Differential diagnosis

Leydig cell tumor
Definition / general
Essential features
  • Most common sex cord stromal tumor of the testis
  • Histology: diffuse / nodular growth of polygonal cells with abundant eosinophilic cytoplasm, uniform round nuclei and prominent central nucleoli; Reinke crystals may be present
  • Immunohistochemistry: inhibin A+, calretinin+, MelanA+, SF1+, AR+
  • Features associated with malignant potential include: size > 5 cm, infiltrative borders, cytological atypia, frequent mitoses (> 3/10 high power fields), vascular invasion and necrosis
  • Treatment: surgical resection, curative for nonmetastasizing tumors
  • Prognosis: overall 5 year survival after orchiectomy > 90%
Terminology
  • Interstitial cell tumor - obsolete term
ICD coding
  • ICD-10: D40.10 - Leydig cell tumor of testis
Epidemiology
Sites
Pathophysiology
  • Produces androgen, mainly testosterone, which can cause symptoms described below
  • Can also produce estrogen by either direct production of estradiol or by peripheral aromatization of testosterone (Arch Pathol Lab Med 2007;131:311)
Etiology
  • Little is known
  • Rare association with germline fumarate hydratase mutations in patients with hereditary leiomyomatosis and renal cell carcinoma syndrome and activating mutations of the luteinizing hormone receptor (N Engl J Med 1999;341:1731)
Clinical features
Diagnosis
  • Tumor histology and immunohistochemistry
Laboratory
  • Serum testosterone and estrogen levels may be elevated
  • Lower sperm concentration, lower total sperm count and motility (Hum Reprod 2019;34:1389)
Radiology description
  • Nonspecific
  • On ultrasound, tumors are generally well defined, homogeneous hypoechoic, small solid masses with hypervascularity (Arch Pathol Lab Med 2007;131:311)
  • May show cystic areas
Radiology images

Images hosted on other servers:
Missing Image

CT scan: circumscribed mass

Missing Image

MRI: solid enhancing mass

Missing Image

Ultrasound: mixed echogenic mass

Prognostic factors
  • Benign Leydig cell tumors: excellent prognosis, curative by surgery
  • Malignant Leydig cell tumors: poor survival, most develop metastatic disease leading to death (J Urol 2020;203:949)
Case reports
Treatment
Gross description
  • Well circumscribed, solid homogeneous mass
  • Usually < 5 cm in size
  • Golden brown or brownish green cut surface
  • Hyalinization and calcification may be present
  • Gross features suggestive of malignancy (Arch Pathol Lab Med 2007;131:311)
    • Large size: > 5 cm
    • Infiltrative margins
    • Hemorrhage and necrosis
    • Extratesticular extension
Gross images

Contributed by Debra L. Zynger, M.D.
Small tumor

Small tumor

Nodular tumor

Nodular tumor

High risk tumor

High risk tumor



Images hosted on other servers:
Missing Image

Circumscribed brown tumor

Frozen section description
  • Diffuse sheets of uniform polygonal cells with round nuclei, central nucleoli, abundant granular, eosinophilic cytoplasm and rectangular to club shaped Reinke crystals (Hum Pathol 2015;46:600)
  • Touch imprint and scrape smear preparations are better to highlight Reinke crystals (Arch Pathol Lab Med 2005;129:e65)
Frozen section images

Images hosted on other servers:
Missing Image

Reinke crystals

Microscopic (histologic) description
  • Architecture:
    • Diffuse or nodular with fibrous bands
    • Uncommon patterns: insular, trabecular, pseudotubular, ribbon-like, trabecular, spindled and microcystic (Surg Pathol Clin 2018;11:739)
  • Cytologic features:
    • Polygonal cells with abundant eosinophilic granular cytoplasm, uniform round nuclei and prominent central nucleoli; rarely, nuclei may have a ground glass appearance
    • Uncommon cell types: scant cytoplasm, foamy cytoplasm and spindling (Am J Surg Pathol 2002;26:1424)
    • Lipofuscin pigment maybe present: golden yellow on H&E stain, red-purple granular appearance on PAS stain
    • Binucleated and multinucleated cells may be present
    • Reinke crystals: pathognomonic; identified in only up to 30% (degradation / dissolution by formalin fixation); intracytoplasmic, rarely extracellular
    • Mitosis: rare
    • Mild nuclear atypia permissible
    • Occasionally, psammoma bodies, calcification, osseous and adipocytic metaplasia may be identified (Am J Surg Pathol 2002;26:1424)
  • Microscopic features suggestive of malignancy (most malignant tumors will have more than 2 of these features) (Am J Surg Pathol 1985;9:177):
    • > 5 cm
    • Infiltrative borders
    • Cytological atypia
    • Frequent mitoses (> 3/10 high power fields)
    • Vascular invasion
    • Necrosis
Microscopic (histologic) images

Contributed by Manju Aron, M.D. and Kristine Cornejo, M.D.

Polygonal cells

Eosinophilic cells

Reinke crystals

Inhibin positive

SF1 positive

Cytology description
  • Fine needle aspiration is rarely performed unless in a metastatic lymph node
  • Cellular smears with discohesive cells having eccentric round nuclei, evenly distributed chromatin, prominent nucleoli and abundant eosinophilic granular cytoplasm
  • Naked nuclei are common
  • Cytoplasm may be vacuolated due to lipid accumulation
  • Nuclear grooves, binucleation and multinucleation may be identified
  • Nuclear pseudoinclusions and Reinke crystals can be seen
  • No cytological features to differentiate Leydig cell tumors from nodular Leydig cell hyperplasia (J Am Soc Cytopathol 2019;8:220)
Positive stains
Negative stains
Electron microscopy description
  • Reinke crystals are diagnostic
    • Appearance depends on plane of sectioning: prismatics, hexagonal lattices or hexagonal microtubules with parallel lines
    • Located in cytoplasm but can be seen in nucleus or interstitium
  • Abundant smooth endoplasmic reticulum, mitochondria with tubulovesicular cristae, numerous lipid droplets and lipofuscin granules (Case Reports Histol Histopathol 1989;4:247)
Molecular / cytogenetics description
  • DNA aneuploidy is associated with malignant Leydig cell tumors, benign Leydig cell tumors are diploid
  • Gain of chromosome X, 19 or 19p and loss on chromosome 8 and 16 are most frequent findings (Oncol Rep 2007;17:585)
  • Somatic GNAS (guanine nucleotide binding protein, alpha stimulating activity polypeptide 1)
Sample pathology report
  • Right testicle, radical orchiectomy:
    • Leydig cell tumor, 2 cm in greatest dimension (see synoptic report)
    • Tumor limited to testis
    • Resection margins uninvolved by tumor
  • Note that benign tumors are not staged
Differential diagnosis
Board review style question #1

A 21 year old man had a 2 cm painless mass in the left testicle. Histological features are shown in the image above. Which marker will be positive in this tumor?

  1. SALL4
  2. PLAP
  3. OCT4
  4. Inhibin A
  5. Cytokeratin
Board review style answer #1
D. Inhibin A. The diagnosis is Leydig cell tumor.

Comment Here

Reference: Leydig cell tumor
Board review style question #2
Malignant potential in Leydig cell tumor is associated with which of the following factors?

  1. Younger patient age
  2. Diffuse growth pattern
  3. Large tumor size (> 5 cm)
  4. Calcification
  5. Reinke crystals
Board review style answer #2
C. Large tumor size (> 5 cm)

Comment Here

Reference: Leydig cell tumor
Board review style question #3

A 41 year old man was hit in the groin area by a baseball. An ultrasound showed a 3 cm tumor of the testis. On histology, the tumor showed diffuse sheet-like growth of cells with minimal stroma, shown above. The tumor cells were large and polygonal with abundant, slightly granular, eosinophilic cytoplasm. The tumor cell nuclei were round and contained a single prominent nucleolus. Rare intracytoplasmic eosinophilic crystals were identified. The most likely diagnosis of this tumor is which of the following?

  1. Adult granulosa cell tumor
  2. Juvenile granulosa cell tumor
  3. Leydig cell tumor
  4. Sertoli cell tumor
Board review style answer #3
C. Leydig cell tumor

Comment Here

Reference: Leydig cell tumor

Liposarcoma
Epidemiology
  • Rare; mean 63 years, range 41 to 87 years but younger patients have been reported
  • Most common malignant tumor involving the spermatic cord (46%) (Urol Oncol 2014;32:52)
  • Paratesticular liposarcomas account for ~12% of all liposarcomas
  • Well differentiated tumors tend to recur, often late yet have a good outcome; dedifferentiated tumors have metastatic potential
Sites
  • Usually arise from spermatic cord; also testicular tunica; rarely epididymis
Pathophysiology / etiology
  • Recurrent molecular abnormalities have been detected in liposarcomas, which differ between each subtype
  • Well differentiated liposarcoma is characterized by the presence of amplification involving 12q13-q15, which can be identified using FISH or CGH
  • Myxoid liposarcoma is characterized by the presence of t(12;16)(q13;p11), which results in the FUS-CHOP gene fusion that is present in over 95% of cases
    • Most often, the amino terminal domain of FUS (also known as TLS) is fused to C/EBP homologous protein (CHOP, also known as DDIT3 or GADD153)
    • In rare cases, an alternative translocation event is found, t(12;22)(q13;q12), that results in formation of the novel fusion oncogene where EWS takes the place of FUS
    • RET, IGF1R and IGF2 are highly expressed in MLPS and promote cell survival through both the PI3K/Akt and Ras-Raf-ERK/MAPK pathways (Hum Pathol 2009;40:1244, Nat Clin Pract Oncol 2007;4:591) and represent potential therapeutic targets
  • Pleomorphic liposarcoma shows complex chromosomal abnormalities with gains in 1p, 1q21-q32,2q, 3p, 3q, 5p12-p15, 5q, 6p21, 7p, 7q22 and losses involving 1q, 2q, 3p, 4q, 10q, 11q, 12p13, 13q14, 13q21-qter, 13q23-24
Clinical features
  • Painless scrotal mass of longstanding duration is the most common presentation
  • Rarely is of recent duration
  • May present as an inguinal hernia, hydrocele, hematocoele or other testicular tumor
Diagnosis
  • Diagnosis requires histological examination
  • Clinical differentiation from other testicular and paratesticular sarcomas is not possible
Laboratory
  • No specific laboratory abnormality is known
  • Serum markers for germ cell tumors and sex cord tumors are negative
Radiology images

Images hosted on other servers:

Nonhomogeneous right scrotal mass

No evidence of tissue infiltration

Large heterogenous extratesticular mass

CT of paratesticular liposarcoma presenting as hernia

Prognostic factors
  • Recurrence of well differentiated paratesticular liposarcoma after complete resection is extremely rare (Malays J Med Sci 2013;20:95)
  • Recurrence rate is < 10% when resection margin is 10 mm or greater
  • Risk factors for local recurrence / progression:
    • High grade tumor morphology
    • Large tumor size ( > 5 cm)
    • Inadequate / suboptimal resection
Case reports
Treatment
  • Usual treatment is orchidectomy through inguinal approach, with adequate margins
  • Elective inguinal node dissection is not indicated
  • Cases with a margin < 10 mm or with residual tumor may benefit from radiotherapy; liposarcoma is radiosensitive and the well differentiated subtype is most sensitive
  • Radiotherapy also useful for unresectable cases
  • Role of chemotherapy (ex: doxorubicin) is debated; no clear benefit has been shown
  • Surgical excision is also the cornerstone of management of recurrent cases
Clinical images

Images hosted on other servers:

Scrotal mass at presentation

Tumor after chemotherapy

Gross description
  • Tumors are usually large, appear to be relatively circumscribed
  • Cut surface may be yellow, fatty with interspersed fibrous septae or may be uniformly firm and white
  • Areas of necrosis and hemorrhage may be seen in high grade tumors
Gross images

Images hosted on other servers:

Tumor mass

Yellow and myxoid areas

White yellow to red brown cut surface, marked necrosis

Right paratesticular mass with solid, yellow white, fatty, myxoid areas

Grossly firm tumor

Microscopic (histologic) description
  • Mature adipocytes, atypical spindle cells and multivacuolated lipoblasts embedded in a loose myxoid to dense fibrous stroma
  • All variants of liposarcoma may be seen; well differentiated or dedifferentiated liposarcomas are most common
  • Heterologous leiomyosarcomatous or osteosarcomatous differentiation (Am J Surg Pathol 2002;26:742) and osseous metaplasia (Case Rep Urol 2015;2015:965876) have been reported but their presence does not appear to affect prognosis
  • Pleomorphic liposarcoma accounts for < 5% of cases and is characterized by MFH-like histology with a disorderly pattern, pleomorphic cells, multinucleated bizarre giant cells and lipoblasts
  • Giant lipoblasts have enlarged globular or angular hyperchromatic nuclei
  • Other variants have also been reported
Microscopic (histologic) images

Images hosted on other servers:

Well differentiated liposarcoma:

Atypical spindle cells and lipoblasts

Mature adipocytes and lipoblasts



Well differentiated with dedifferentiated component:

Hypercellular stroma with atypical adipocytes

Atypical adipocytes with enlarged nuclei



Dedifferentiated with leiomyosarcomatous component:

H&E

MDM2+, CDK4+, alpha smooth muscle actin+, desmin+



Myxoid liposarcoma:

Lipoblasts contain lipid vacuoles

Plexiform arrangement of capillaries

Positive stains
  • MDM2, CDK4 (usually negative in myxoid / round cell and spindle subtypes)
  • S100
  • Heterologous elements may show positivity for desmin, actin, etc., based on the differentiation and dedifferetiated tumors have variable reactivity
  • STAT6 may rarely be positive (reported in dedifferentiated liposarcoma)
Negative stains
Differential diagnosis

Lymphoma
Definition / general
  • Either primary or secondary involvement of testis by malignant / clonal lymphocytes
  • Almost always diffuse large B cell lymphoma
  • Also anaplastic lymphoma, Burkitt lymphoma, Hodgkin lymphoma, peripheral T cell lymphoma, follicular lymphoma
Epidemiology
  • ~5% of testicular malignancies
  • Most common malignant testicular neoplasm in men over 60 years although can occur at any age (range 16 to 91 years, median age 67) (Clin Oncol (R Coll Radiol) 2012;24:358)
Clinical features
  • Painless testicular mass
  • Bilateral testicular involvement in up to 35% of cases; often spreads to CNS
Radiology description
  • Gray scale and color Doppler ultrasound findings include diffuse testicular infiltration / enlargement with hypervascularity or multifocal areas of hypoechoic, solid and hypervascular nodules within testes (Ultrasound Q 2013;29:247)
Prognostic factors
  • Better prognosis if testis involvement is primary site (60% 5 year survival vs. 17% for disseminated disease / other stages) and unilateral
Case reports
Treatment
  • Orchiectomy and chemotherapy
Gross description
  • White tan pink, fleshy, resembles seminoma, often extratesticular involvement
Gross images

Images hosted on other servers:

Diffuse large B cell lymphoma

Microscopic (histologic) description
  • Splaying apart but relative sparing of tubules by lymphoma cells
  • Malignant cells are pleomorphic and noncohesive with large irregular nuclei and prominent nucleoli
  • May have vascular invasion and sclerosis of seminiferous tubules
  • No intratubular germ cell neoplasia
Microscopic (histologic) images

Images hosted on other servers:

Diffuse large B cell lymphoma


Peripheral T cell lymphoma

Positive stains
  • LCA / CD45
  • Other results dependent on lymphoma subtype
Negative stains
Differential diagnosis

Melanotic neuroectodermal tumor
Definition / general
  • A rare benign tumor of neuroectodermal origin with biphasic population of neuroblastic cells and pigmented epithelial cells
Terminology
  • First described by Krompecher in 1918 as congenital melanocarcinoma of alveolar process of maxilla (BMJ Case Rep 2010 Nov 23;2010)
  • Previously known as: melanotic adamantinoma, melanotic hamartoma, melanotic progonoma, pigmented congenital epulis, pigmented neuroectodermal tumor, retinal anlage tumor, retinoblastic teratoma
Epidemiology
  • Rare
  • Usually young infants (mean 7 months, range 3 months to 8 years) but also reported in adults
  • Most common in maxilla of infants with 2:1 female preponderance (see Mandible - Maxilla Chapter)
  • Also reported in long bones (Cancer 1983;52:661), uterus, skin and brain
  • Epididymis is uncommon site
Etiology
  • Precise origin is unknown but suggested to be of neural crest origin, due to a developmental error in evolution of mesonephric ducts, where neural crest cells migrate (J Pathol Bacteriol 1967;93:549)
  • A dysembryogenetic neoplasm that recapitulates embryonic retinal development (Am J Surg Pathol 1991;15:233)
  • Relationship with other neural crest tumors such as PNET, neuroblastoma, etc. is not clear
Clinical features
  • Firm, rapidly enlarging mass, sometimes associated with hydrocele
  • Usually noticed at birth or within the first year of life
  • Lymphatic space invasion is associated with hydrocele formation
Diagnosis
  • Suspected clinically and confirmed histologically
Laboratory
  • Urinary vanillylmandelic acid (VMA) / homovanillic acid (HVA) elevated in some cases
Prognostic factors
Case reports
Treatment
Gross description
  • Round to oval, circumscribed, firm epididymal mass, usually < 4 cm
  • Cut surface is grey white with areas of pigmentation; color depends on amount of pigmentation
Gross images

Images hosted on other servers:

Maxillary tumors

Microscopic (histologic) description
  • Appears to recapitulate retina at 5 weeks of gestation
  • Contains dual population of cells: neuroblastic cells with high nuclear cytoplasmic ratio and epithelioid cells containing melanin
  • Mitoses may be present, usually in neuroblastic component
Microscopic (histologic) images

Images hosted on other servers:

Maxillary tumor: H&E and stains

Cytology description
  • Dual cell population with dyscohesive primitive neuroblast like cells in fibrillary background, admixed with clusters of larger cells having melanin pigment (Acta Cytol 2006;50:460)
Positive stains
Negative stains
Electron microscopy description
  • Large cells contain melanosomes and premelanosomes with varied maturation while the neuroblastic cells contain neurosecretory granules and neurofilaments
Differential diagnosis

Mesothelioma
Definition / general
  • Malignant tumor of mesenchymal origin arising from the serosal membrane of the tunica vaginalis
  • Usually aggressive, similar to peritoneal cavity tumor (Am J SurgPathol 1995;19:815)
Epidemiology
Sites
  • Rarely arises from the tunica vaginalis (0.3 - 5% of all tumors), often in relation to the epididymis, testis or spermatic cord
  • More commonly arises in pleura (68 - 85%) and peritoneum (9 - 24%) (Arch Pathol Lab Med 2012;136:113)
Pathophysiology
  • Originates from tunica vaginalis, which derives from invagination of the peritoneum into the scrotum
Etiology
Clinical features
  • Usually present with hydrocele - insidious, painless swelling of scrotum
  • Most cases are unilateral; bilateral tumors are rare, ~4% of cases (Cancer 1998;83:2437)
  • Other presentations include inguinal hernia, epididymitis or testicular mass
  • Poor prognosis, even with negative resection margins
  • Mean disease specific survival of 23 months, range of 2 to 64 months (Arch Pathol Lab Med 2012;136:113)
Radiology description
  • Scrotal ultrasonography usually reveals hydrocele with thickened wall and hypoechoic nodule over epididymis / tunica vaginalis, or a solid paratesticular mass (Urology 2012;80:e3) Ultrasound reveals hydrocele with thickened wall with hypoechoic nodule over epididymis / tunica vaginalis
Radiology images

Images hosted on other servers:

Increased thickness of the left tunica vaginalis

Small nodules

Border indistinctness in the right scrotum

Prognostic factors
  • Extent of disease at presentation is important prognostic factor (Cancer 1998;83:2437); metastases at presentation is associated with worse prognosis
  • Complete / adequate excision improves disease free survival; lower recurrence rate after radical orchiectomy (~10%) versus simple hydrocele sac excision (~30%) (Cancer 1998;83:2437)
  • Better outcome in younger patients
  • Well differentiated papillary mesothelioma has better prognosis than undifferentiated spindle cell mesothelioma
  • Other prognostic factors include tumor size, lymph node metastases, invasion into adjacent structures, degree of differentiation
Case reports
Treatment
  • Radical orchiectomy is standard treatment
  • If treated with transcrotal surgery, subsequent hemiscrotectomy or hemiscrotal irradiation is often recommended
  • Retroperitoneal lymph node dissection is recommended only if pre-op workup reveals suspicious nodes
  • Well differentiated papillary mesothelioma may be treated with only surgery
Clinical images

Case #261

Gross description
  • Multiple nodules within hydrocele sac, frequently associated with mass infiltrating spermatic cord, epididymis or testis
  • May be a solitary nodule
  • Hydrocele sac is thick walled, often containing hemorrhagic fluid and papillary excrescences
  • Less often, mesothelioma creeps along tunica without forming a mass-like lesion
Gross images

AFIP images

Tunica vaginalis



Images hosted on other servers:

Radical orchiectomy specimen

Nodular mass at spermatic cord

Extracted tumor adhered strongly to scrotum skin

Microscopic (histologic) description
  • Same as mesothelioma at other sites
  • Either epithelial (60 - 70%), spindle cell (least common) or biphasic type (30 - 40%)
  • Epithelial type has epithelioid cells arising from the tunica vaginalis with papillary, tubular, adenomatoid or solid architectural patterns
  • Biphasic type has fascicles of spindle cells with scanty stroma, often merging with epithelial cells
  • In well differentiated tumors, neoplastic cells are typically cuboidal with scant to moderate amounts of eosinophilic cytoplasm and bland cytologic features
  • Tumors may also appear poorly differentiated
  • Papillae have thick, hyalinized fibrovascular cores lined by a single layer of atypical mesothelium
  • Stroma may be desmoplastic or show necrosis
  • Variable psammoma bodies
Microscopic (histologic) images

Contributed by Jennifer Gordetsky, M.D.
Malignant mesothelioma Mesothelioma

Mesothelioma



Case #261



Images hosted on other servers:

Epithelial tumor cells

Glandular pattern in desmoplastic stroma

Malignant mesothelial tubular structures

Biphasic exophytic nodule


Papillary structures

Mesothelioma of uncertain malignant potential

Cells are rounded with nuclear atypia

Cytology description
  • Moderately cellular smears; well differentiated tumors have papillary clusters of epithelial cells with minimal atypia
Negative stains
Electron microscopy description
  • Cells show epithelial and mesenchymal differentiation
  • Epithelial cells joined by intercellular junctions, including desmosomes and junctional complexes, with lumen formation
  • Numerous cytoplasmic filaments including tonofibrils; cytoplasmic glycogen is also present
  • Long, slender microvilli are typically seen on the surface of the tumor cells as well as in intracellular and intercellular lumina
  • Microvilli length to diameter ratio > 10 (Ulbright: AFIP Series 3, Vol 25)
Molecular / cytogenetics description
  • Poorly characterized - mesotheliomas harbor multiple cytogenetic abnormalities with no specific diagnostic characteristics
  • Losses of chromosomal regions in 1p, 3p, 6q, 9q, 8p, 14q and 22q and gains in 5p, 6p, 8q, 15q, 17q, 20 and monosomy 22 have been reported (Arch Pathol Lab Med 2012;136:113)

Metastases
Definition / general
  • Tumors secondarily involving testis and paratesticular structures from distant sites
  • Excludes hematopoietic tumors
Essential features
  • Most have known history of primary but rarely the initial presentation
  • Histologic features recapitulate their site of origin
  • Should be differentiated from germ cell tumor, sex cord stromal tumor, malignant mesothelioma and adenocarcinoma of the rete testis or epididymis
Terminology
  • Also called secondary tumors of the testis
Epidemiology
Sites
Clinical features
  • Most nonsymptomatic or incidental finding
  • Most unilateral and solitary
  • Rarely present as carcinoma of unknown primary
Diagnosis
  • Clinical information and ancillary investigations can be more helpful than special histological techniques in the differential diagnosis
Radiology images

Images hosted on other servers:

Renal cell carcinoma metastasis to testis

Lung adenocarcinoma metastasis to testis

Colorectal adenocarcinoma metastasis to testis

Case reports
Gross description
  • Localized mass, multiple nodules
Gross images

Contributed by Debra L. Zynger, M.D.
Metastatic prostatic adenocarcinoma

Metastatic prostatic adenocarcinoma



Images hosted on other servers:

Metastatic colorectal adenocarcinoma

Metastatic renal cell carcinoma

Microscopic (histologic) description
  • Histologic features recapitulate their site of origin
  • Metastatic prostatic adenocarcinoma tends to involve the tubular system
  • Other metastatic carcinomas tend to infiltrate the testicular interstitium while sparing the seminiferous tubules
  • Prominent lymphovascular invasion
Microscopic (histologic) images

Contributed by Gang Wang, M.D., Ph.D.

Metastatic prostatic adenocarcinoma
Missing Image

Involvement of the tubular system

Missing Image

Cribriform pattern
most frequent,
epididymis
involvement

Missing Image

Solid growth pattern

Missing Image

Metastatic ductal adenocarcinoma

Missing Image

Positive for PSA

Missing Image

Positive for ERG



Metastatic renal cell carcinoma, clear cell type
Missing Image

Broad pushing borders

Missing Image

Mixed patterns

Missing Image

Positive for PAX8

Missing Image

Positive for CAIX

Metastatic colorectal
adenocarcinoma

Missing Image

Cystic spaces containing mucin

Metastatic cutaneous squamous
cell carcinoma

Missing Image

Infiltrating and anastomosing nests

Missing Image

Positive for p40

Positive stains
  • Mucin (teratomatous elements and yolk sac tumors may also be positive)
  • EMA
Negative stains
Molecular / cytogenetics description
  • Detection of isochromosome 12 by fluorescence in situ hybridization would suggest a primary germ cell tumor rather than a metastases
Sample pathology report
  • Right testis and spermatic cord, radical orchiectomy:
    • Metastatic renal cell carcinoma, clear cell type, 2 cm, confined to the testicular parenchyma
    • Margins of resection, negative for malignancy
Differential diagnosis
Board review style question #1

Which of the following is the most common primary site of metastatic carcinoma in the testis?

  1. Bladder
  2. Kidney
  3. Prostate
  4. Lung
  5. Colon
Board review style answer #1
C. The most common primary site of metastatic carcinoma in the testis is the prostate

Comment Here

Reference: Metastases to testes
Board review style question #2

    As shown in the figure, there is an adenocarcinoma in the testis. Which of the following markers is the best for differentiation between this metastatic adenocarcinoma in the testis and primary adenocarcinoma of the rete testis or epididymis?

  1. Calretinin
  2. CDX2
  3. CK7
  4. CK20
  5. PAX8
Board review style answer #2
E. Primary adenocarcinomas of the rete testis or epididymis are positive for PAX8 while metastatic adenocarcinomas from other sites are usually negative for PAX8 (except kidney and thyroid)

Comment Here

Reference: Metastases to testes

Mixed and other sex cord stromal tumors
Definition / general
  • A sex cord stromal tumor of the testis with mixed sex cord stromal elements or undifferentiated features
  • May contain any combination of cell types, including Sertoli cells, Leydig cells, granulosa cells
Terminology
  • Also includes undifferentiated stromal tumors with no clearly demonstrable differentiation
Epidemiology
  • Rare tumor of descended testis; < 1% of testicular tumors (J Urol 2004;172:2370)
  • Can occur in any age group; 1/3 of cases occur in children
  • Account for 2/3 of sex cord tumors occurring in children
Etiology
  • No specific etiology has been reported for the mixed / undifferentiated sex cord stromal tumors
Clinical features
  • Usually present with a painless testicular swelling
  • May be painful (Ulster Med J 1997;66:54)
  • 15% present with gynecomastia
Comparison of mixed sex cord stromal tumors with and without gonadoblastoma
  • Mixed sex cord stromal tumor with gonadoblastoma:
    • Associated with disorders of sexual development (gonadal dysgenesis [pure or mixed], altered karyotype, pseudohermaphrodites with streak gonads)
    • Leydig-like cells and lutein cells
    • Virilization
    • Proliferative activity only in germ cells
  • Mixed sex cord stromal tumor without gonadoblastoma:
    • Genetically and phenotypically normal individuals
    • No Leydig-like cells and lutein cells
    • No endocrine effects usually; if present, usually feminization
    • Proliferative activity in sex cord stromal and germ cell components
Laboratory
  • Germ cell markers beta-hCG, LDH, alpha fetoprotein are within normal limits
Radiology description
  • Solid tumors but may have cystic areas
Prognostic factors
  • Similar to other sex cord stromal tumors: poor factors are large size, invasive growth pattern, angiolymphatic invasion, nuclear atypia, mitotic activity, necrosis
  • These tumors are usually benign in children < age 10 years
  • ~20% in older patients are malignant
Case reports
Treatment
  • Similar to other sex cord stromal tumors
    • High inguinal orchidectomy routinely done
    • Testicular sparing surgery may be attempted if tumor is small, well circumscribed without any invasive features, normal tumor markers and frozen section facilities are available for intraoperative diagnosis
    • No proven indication for lymph node dissection if nodes are not enlarged
Gross description
  • Gray, tan or yellow solid nodules
  • Presence of invasion into tunica, rete testis or adjacent structures predicts malignant / aggressive behavior
Gross images

Images hosted on other servers:

Well circumscribed lower pole testicular tumor

Microscopic (histologic) description
Microscopic (histologic) images

Images hosted on other servers:

Tumor is compressing adjacent tunica

Fascicular and storiform spindle cell arrangement

Insular arrangement of cells in hyaline stroma

Irregular ovoid nuclei with longitudinal grooves

Positive stains
Negative stains
Molecular / cytogenetics description
  • Ovarian tumors have c-kit mutations (seen in seminomas) and excess genetic material on #12p (seen in all germ cell tumors) but no genetic aberrations identified in testicular tumors (Virchows Arch 2006;448:612)
Differential diagnosis
  • Pure sex cord stromal tumors: show only one cell type and not more than an occasional microscopic focus of other elements; presence of more than occasional occurrence of two or more sex cord stromal cell types warrants a diagnosis of mixed sex cord stromal tumor
  • Sex cord stromal tumor with entrapped germ cells (Am J Surg Pathol 2000;24:535) and mixed germ cell sex cord stromal tumors: show clusters of germ cells in the center of the tumor; only one sex cord stromal cell type present

Mucinous borderline tumor
Definition / general
  • Primary mucinous tumor of ovarian type surface epithelium with borderline malignant potential
Epidemiology
Sites
  • Intratesticular or paratesticular
Pathophysiology
  • The histogenesis of the ovarian type surface epithelial tumors of the testis and paratestis remains unclear
  • These tumors are possibly derived from Müllerian remnants or by metaplasia of the mesothelium of the tunica vaginalis (Hum Pathol 2011;42:1343)
Diagnosis
  • Clinical evaluation is necessary to rule out metastatic spread from a primary adenocarcinoma of the appendix, colon, stomach, pancreas, lung or prostate
Case reports
Treatment
  • Radical orchiectomy
Clinical images

Images hosted on other servers:

Area of curvilinear calcification

Gross description
  • Paratesticular or intratesticular mucinous cystic mass, most often unilocular
  • Generally not as large as the ovarian tumor counterpart
Microscopic (histologic) description
  • Unilocular or multilocular cystic spaces lined by either endocervical or intestinal type mucinous epithelium
  • Tufting and pseudostratification of cells with mild to severe nuclear atypia
  • Occasional papillae
  • Occasional mitotic figures
  • Goblet cells are common
  • Mucin extravasation and an associated fibrotic, calcific reaction with foci of dystrophic calcification and ossification
  • Intracystic complex architecture diagnostic of intraepithelial carcinoma as well as foci of microinvasion have been reported
  • No intratubular germ cell neoplasia (Am J Surg Pathol 2003;27:1221, Can Urol Assoc J 2010;4:E112)
Microscopic (histologic) images

Images hosted on other servers:

Various images

Positive stains
Differential diagnosis

Mucinous cystadenocarcinoma
Definition / general
  • Ovarian type surface epithelial carcinoma of the testis with mucinous differentiation
Epidemiology
  • Extremely rare testicular tumor reported in middle age and elderly men (mean age 62)
Pathophysiology
  • The histogenesis of the ovarian type surface epithelial tumors of the testis and paratestis remains unclear
  • These tumors are possibly derived from Müllerian remnants or by metaplasia of the mesothelium of the tunica vaginalis (Hum Pathol 2011;42:1343)
Diagnosis
  • Clinical evaluation is necessary to rule out metastatic spread from a primary adenocarcinoma of the appendix, colon, stomach, pancreas, lung or prostate
Case reports
Treatment
  • Radical orchiectomy
Clinical images

Images hosted on other servers:

Right testicular borderline mucinous tumor

Gross description
  • Mucinous, cystic mass replacing the testicular parenchyma
Microscopic (histologic) description
  • Well to moderately differentiated malignant cystic neoplasm with mucinous differentiation
  • Back to back irregular glands, elongated fine complex papillary structures lined with alternating goblet cells and ciliated cells, with little intervening stroma
  • Papillary structures have atypical cells with hyperchromatic overlapping elongated nuclei with coarse chromatin and increased nuclear to cytoplasmic ratio
  • Cystic spaces filled with mucin, fibrinopurulent material, degenerative cellular debris
  • Psammomatous calcifications can be present as well as mucin extravasation
Microscopic (histologic) images

Images hosted on other servers:

Cyst wall irregularly thickened and fibrotic

Mucinous epithelium
resembling endocervical
type cells

Positive stains
Negative stains
Differential diagnosis

Myoid gonadal stromal tumor
Definition / general
  • A rare testicular neoplasm composed of spindle shaped cells that share features of smooth muscle and gonadal stroma (Virchows Arch 2021;478:727)
  • Hypothesized to arise from peritubular myoid cells or intertubular primitive mesenchymal cells (Am J Clin Pathol 2014;142:675)
  • Proposed as an emerging entity in the 2016 WHO classification because of the limited number of cases and the overlapping morphology and immunoprofile with the fibroma thecoma group; distinct entity in the WHO 5th edition
Essential features
  • A pure spindle cell tumor that lacks sex cord differentiation; positive for both actin and S100
  • Rare testicular tumor that may originate from intertubular primitive mesenchymal cells that undergo myogenic differentiation
  • Benign tumor with favorable prognosis
  • Small, circumscribed and nonencapsulated tumors that contain medium to large sized ectatic blood vessels with spindled tumor cells arranged in short intersecting fascicles
  • Differential diagnosis includes leiomyoma, testicular fibrothecoma and unclassified spindle cell predominant sex cord stromal tumor
Terminology
  • Historically, nomenclature includes
    • Unusual gonadal stromal tumor
    • Testicular stromal tumor with myofilaments
    • Unclassified sex cord stromal tumor with predominance of spindle cells
ICD coding
  • ICD-O: 8590/0 - myoid gonadal stromal tumor
  • ICD-11: 2F77 & XH9G57 - neoplasms of uncertain behavior of male genital organs & sex cord - gonadal stromal tumor, NOS
Epidemiology
  • Can occur in any age but typically seen in young to middle aged men between 4 and 59 years (median is 37 years, mean is 43 years) (Am J Clin Pathol 2014;142:675)
Sites
  • Affects the testis unilaterally
Pathophysiology
  • Hypothesized to arise from peritubular myoid cells which originate from intertubular primitive mesenchymal cells that undergo myogenic differentiation
  • Peritubular myoid cells play key roles in the development of the fetal testis and maintenance of testicular function (Hum Pathol 2012;43:144, Arch Histol Cytol 1996;59:1)
Etiology
  • Unknown
Clinical features
Diagnosis
  • Surgical specimen evaluated by H&E and immunohistochemical staining
Laboratory
  • No abnormalities seen
Radiology description
Radiology images

Images hosted on other servers:

Ultrasound features

Prognostic factors
  • Small tumors (< 4 cm) with low mitotic activity have an extremely favorable outcome
  • Recurrence or metastasis is rarely seen
  • Reference: Am J Clin Pathol 2014;142:675
Case reports
Treatment
  • Orchiectomy
  • Adjuvant therapy generally not required
Gross description
  • Unifocal and well circumscribed with a yellow / tan appearance
  • Small sized tumors that range from < 1 cm to 4 cm (Am J Clin Pathol 2014;142:675)
Frozen section description
  • Spindle cell neoplasm, low grade
Microscopic (histologic) description
  • Circumscribed and nonencapsulated
  • Focally contain medium to large sized ectatic blood vessels and can be seen adjacent to the rete testis
  • Tumor cells are spindled and arranged in short intersecting fascicles, sometimes with a storiform pattern
  • Scant collagen with thin bands in the background
  • Nuclei range from elongated to fusiform with inconspicuous to small nucleoli and occasional grooves
  • Cytoplasm is scant to moderate and ill defined, ranging from pale to lightly eosinophilic
  • No significant cytologic atypia is seen
  • Mitotic figures vary from 0 to 5 per 10 high power fields
  • No necrosis or lymphovascular invasion
  • No differentiated gonadal stromal cells (Sertoli, Leydig or granulosa cells) are seen
  • Reference: Am J Clin Pathol 2014;142:675
Microscopic (histologic) images

Contributed by Vikas Mehta, M.D.

Ectatic blood vessels

Spindle arrangement of tumor cells


SF1 positivity

h-caldesmon negativity

SMA positivity

S100 positivity

Positive stains
Negative stains
Electron microscopy description
  • Myofilaments and desmosomes
  • Spindle cells showing lipid vacuoles
  • Elongated cell with spindle nucleus and small cytoskeletal filaments (Virchows Arch 2021;478:727)
Molecular / cytogenetics description
  • Recurrent chromosome arm level and whole chromosome level copy number gains indicative of ploidy shifts
  • Chromosome gains of 3, 6 or 6p, 7, 8, 9 or 9q, 11, 12, 14q, 15q, 17 or 17p, 18q, 20 and 21q and copy number loss 22q (Histopathology 2023;82:431)
Sample pathology report
  • Testis, biopsy:
    • Myoid gonadal stromal tumor (see comment)
    • Comment: The biopsy shows a spindle cell tumor with fascicular and storiform architecture. The tumor cells show varying amounts of eosinophilic to amphophilic cytoplasm, round to oval regular nuclei and inconspicuous or small nucleoli. No vascular invasion, foci of necrosis or atypical mitosis are identified. The tumor cells express SMA, S100 protein, inhibin, FOXL2, calretinin and SF1. In contrast, SOX9, WT1, AE1 / AE3 and EMA expression were not detected. The overall morphologic features and coexpression of S100 protein and SMA support the diagnosis above.
Differential diagnosis
  • Leiomyoma:
    • Diffuse broad fascicles
    • Abundant eosinophilic cytoplasm
    • Cigar shaped nuclei
    • Negative S100 reactivity
    • Positive h-caldesmon reactivity

  • Testicular fibrothecoma:
    • Collagen bundles are typically thicker and more diffuse, sometimes forming hyaline plaques
    • Variable and patchy S100 reactivity
    • Positive SOX9 reactivity

  • Unclassified spindle cell predominant sex cord stromal tumor:
    • Subtle sex cord elements identified on H&E
    • Darker, tight groupings of cells with rounded nuclei and indistinct, typically pale, often scant cytoplasm; may be arranged in nests or short cords
    • Immunohistochemical stain reactivity may show significant overlap
Board review style question #1

Myoid gonadal stromal tumor is a new entity described in the latest WHO classification of testicular tumors. A representative histologic image is shown above. Which of the following is true for these tumors?

  1. Consistent pattern of recurrent chromosome arm level and whole chromosome level copy number gains are indicative of ploidy shifts
  2. Intermixed primitive germ cells and diffuse broad fascicles are present
  3. Mitotic count of > 5/10 high power fields is common
  4. Vascular invasion is a defining characteristic of these tumors
Board review style answer #1
A. Consistent pattern of recurrent chromosome arm level and whole chromosome level copy number gains are indicative of ploidy shifts

Comment Here

Reference: Myoid gonadal stromal tumor

Papillary cystadenoma
Definition / general
Terminology
  • Also called papillary cystadenofibroma
Epidemiology
  • Second most common benign neoplasm of epididymis after adenomatoid tumor
  • Sporadic or familial
  • Unilateral or bilateral (40%)
  • Mean age 36 years
  • Associated with von Hippel-Lindau (VHL) disease; 65% with bilateral tumors have VHL vs. 18 - 20% with unilateral lesions (Arch Pathol Lab Med 2010;134:630)
Sites
  • In male GU, occurs in epididymis (develops within efferent ductules of head of epididymis)
  • In female GU, occurs in broad ligament, peritoneum; also fallopian tube and uterus
Clinical features
  • Benign, usually asymptomatic
  • If symptomatic, typically is slow growing swelling in scrotum, with only rare pain or tenderness
  • Rarely discovered during infertility workup (Arch Pathol Lab Med 2010;134:630)
Laboratory
  • No specific laboratory features
  • Negative for germ cell tumor markers
Case reports
Treatment
  • Surgical excision; does not recur
  • If bilateral, patient should be investigated for VHL syndrome
Clinical images

Images hosted on other servers:

Complex echo pattern

Gross description
  • Well circumscribed, 1 - 5 cm, gray, brown, yellow tumor nodule
  • Cut surface may show papillary fronds or may be cystic with areas of hemorrhage
Gross images

Images hosted on other servers:

Epididymal tumor

Microcystic with hemorrhage

Microscopic (histologic) description
  • Papillary in foldings project into cystic spaces covered by single to double layered cuboidal / columnar cells
  • Tubules common, colloid type cystic material common
  • Tumor cells have lightly eosinophilic to clear cytoplasm
  • May see dilated efferent ductules
  • May have clear cells resembling metastastic renal cell carcinoma but no mitoses, no necrosis, no pleomorphism
  • Papillary cystadenofibroma: prominent stroma
Microscopic (histologic) images

Images hosted on other servers:

Cystic space with papillary excrescences

Cilia

Case with clear cells

CK7

CD10

Cytology description
  • Simple and complex papillary clusters
  • Tumor cells are monomorphic, with moderate to abundant cytoplasm, well defined cytoplasmic borders; round to oval nuclei, finely dispersed chromatin, inconspicuous nucleoli
  • Vacuolations may be present
  • No features of malignancy - i.e. no mitoses, no necrosis, no atypia
  • Usually no psammoma bodies, but see Cytopathology 2014;25:279, Acta Cytol 2004;48:467
Negative stains
Differential diagnosis

Placental site trophoblastic tumor
Definition / general
  • Rare, resembles uterine tumor of same name
Case reports
Microscopic (histologic) description
  • Proliferation of intermediate trophoblasts, identical to uterine tumor
Positive stains

Polyembryoma
Definition / general
  • Mixed germ cell tumor with predominant embryoid bodies (central core of embryonal carcinoma cells, amnion-like cavity and yolk sac tumor component)
  • Elements other than the embryoid bodies, however, should constitute less than 10% of the tumor for the "polyembryoma" designation to be used (Mod Pathol 2005;18:S61)
  • Resembles embryonic yolk sac
  • Behaves like other mixed germ cell tumors
Essential features
  • Central core of embryonal carcinoma cells, amnion-like cavity and yolk sac tumor component that accounts for approximately 90% of tumor
Epidemiology
Clinical features
  • Same as germ cell tumors
  • Elevation of serum AFP and beta HCG
Prognostic factors
  • Same as mixed germ cell tumors
Treatment
  • Same as mixed germ cell tumor with teratoma
Gross images

Contributed by Delia Perez-Montiel, M.D.

40 year old man

Microscopic (histologic) description
  • At low power, the stroma can give the tumor a lobular architecture
  • Embryoid bodies are composed of a hyperchromatic disc of embryonal carcinoma
  • Surrounding the disc of embryonal carcinoma is edematous stroma forming a cavity lined by yolk sac epithelium
Microscopic (histologic) images

Contributed by Delia Perez-Montiel, M.D.



Positive stains
Negative stains
Board review style question #1
What is the difference between a mixed germ cell tumor with embryoid bodies and a polyembryoma?

  1. Mixed germ cell tumors have components other than embryonal carcinoma and yolk sac tumor, while polyembryomas have only embryonal carcinoma and yolk sac tumor.
  2. Polyembryoma is not a germ cell tumor.
  3. Polyembryomas have many different types of embryoid bodies.
  4. Proportion of embryoid bodies in polyembryomas is 90% or higher of the entire tumor.
Board review style answer #1
D. Proportion of embrioid bodies in polyembryomas is 90% or higher of the entire tumor.

Comment Here

Reference: Polyembryoma

Polyorchidism
Table of Contents
Definition / general
Definition / general
  • Multiple testes within a scrotal sac
  • Either one common or separate epididymis

Proliferative funiculitis
Definition / general
Terminology
  • Also known as pseudosarcomatous myofibroblastic proliferation of spermatic cord
  • Some studies mention relationship to: inflammatory myofibroblastic tumor, inflammatory pseudotumor, atypical myofibroblastic tumor, pseudosarcoma (Urology 2009;73:1423)
  • These terms are often used inappropriately in literature and may not truly be related to proliferative funiculitis
Epidemiology
  • Older men (ages 52 - 76 years)
Sites
Etiology
  • May be due to ischemia, torsion or extension of vasitis
Clinical features
Laboratory
  • Normal levels of α-fetoprotein, β-human chorionic gonadotropin
Radiology description
Prognostic factors
  • Can recur locally; excellent prognosis when resected with negative margins
Treatment
  • Surgical resection
Gross description
Gross images

Images hosted on other servers:

Tumor in undescended testis

Microscopic (histologic) description
  • Infiltrative spindle cell proliferation within a myxoid or loose collagenous stroma
  • Nuclei are oval to fusiform
  • Basophilic nucleoli may be present
  • Mixed inflammatory infiltrate mainly composed of plasma cells and lymphocytes
  • May contain malignant appearing glandular structures composed of keratin+ mesothelial cells (Int J Surg Pathol 2008;16:48), rarely has proliferation of mast cells (Pathol Int 2003;53:897)
Microscopic (histologic) images

Images hosted on other servers:

Spindle cell proliferation

Positive stains
Negative stains
Differential diagnosis

Regressed germ cell tumor
Definition / general
Essential features
Terminology
  • Regressed germ cell tumor
  • Burned out germ cell tumor
  • Burnt out germ cell tumor
Epidemiology
Sites
Pathophysiology
Clinical features
Diagnosis
Radiology description
Radiology images

Images hosted on other servers:

Scrotal ultrasound

Regressed
primary with
retroperitoneal
metastasis

Prognostic factors
  • Posttreatment disease free survival rate: 29% at 5 years and 12% at 10 years (J Urol 2009;182:2303)
  • No difference in prognosis between complete and partial regression (J Urol 2009;182:2303)
  • In context of regressed germ cell tumor with metastasis, pure seminoma has a more favorable prognosis (J Urol 2009;182:2303)
  • No data is available comparing prognosis of regressed versus nonregressed testicular germ cell tumors
Case reports
Treatment
Gross description
Gross images

Contributed by Debra L. Zynger, M.D.

Complete regression (pT0)

Germ cell neoplasia in situ only (pTis)

Partial regression (pT1)

Microscopic (histologic) description
Microscopic (histologic) images

Contributed by Debra L. Zynger, M.D.

Scar

Acellular matrix

Small vessels

Hemosiderin


Germ cell neoplasia in situ

OCT3 / 4

D2-40

CD117

Molecular / cytogenetics description
  • Identification of isochromosome 12p by FISH can confirm a testicular germ cell tumor metastasis although immunohistochemistry is more commonly used and has wider availability
Sample pathology report
  • Left testicle and spermatic cord, radical inguinal orchiectomy:
    • Scar (1.5 cm) and germ cell neoplasia in situ, consistent with regressed germ cell tumor (see synoptic report, pTis NX)
    • Comment: the germ cell neoplasia in situ expresses OCT3/4, D2-40, CD117 and PLAP
Differential diagnosis
Board review style question #1

Germ cell neoplasia in situ is found adjacent to a scar, consistent with regressed germ cell tumor. Positive expression using which of the following can confirm the germ cell neoplasia in situ?

  1. AFP
  2. Beta hCG
  3. CD30
  4. CK7
  5. OCT3 / 4
Board review style answer #1
Board review style question #2
What is the most common tumor subtype in a partially regressed testicular germ cell tumor?

  1. Choriocarcinoma
  2. Embryonal carcinoma
  3. Seminoma
  4. Teratoma
  5. Yolk sac tumor
Board review style answer #2

Rete testis adenomatous hyperplasia
Definition / general
Gross description
  • May be solid / cystic mass in testicular hilus; usually small, incidental microscopic finding
Microscopic (histologic) description
  • Tubulopapillary or cribriform proliferations of bland epithelium that projects into dilated channels of rete testis
  • Associated with hyaline globules (represent proteins absorbed from lumen by epithelial lining cells)
Positive stains
Negative stains
Differential diagnosis

Seminoma
Definition / general
  • Most common type of testicular germ cell tumor (up to 50%) and may occur as component of mixed germ cell tumor
  • Derived from transformed gonocytes
  • Consists of cells with well defined borders, (usually) clear cytoplasm, round to polygonal nuclei and prominent nucleolus
Essential features
  • Most frequent germ cell tumor of the testis
  • Presents in young men (30 - 49) with unilateral palpable mass
  • Typically a well demarcated, uniform neoplasm with characteristic cytological features and background of fibrous septae and lymphocytes
  • Germ cell markers (OCT 3/4, CD117) are useful in the diagnosis
  • Prognosis is stage dependent with excellent 5 year survival when clinical stage 1
Terminology
  • Seminoma: testicular primary (or mediastinal primary)
  • Dysgerminoma: same tumor but primary to the ovary
  • Germinoma: same tumor but primary to extragonadal sites (such as pineal gland)
ICD coding
  • ICD-O: 9061/3 - seminoma, NOS
  • ICD-10: C62.90 - malignant neoplasm of unspecified testis, unspecified whether descended or undescended
Epidemiology
  • Male, age 30 - 49
  • Rare in > 70 or < 20 years
Sites
  • Testicle
Pathophysiology
  • Arises from germ cell neoplasia in situ (GCNIS)
  • GCNIS cells are proposed to arise from delayed maturation of primordial germ cells / gonocytes with polyploidization resulting in a transformed germ cell
  • This progresses post puberty into seminoma (most likely GCNIS evolves into intratubular seminoma and then invasive seminoma) (J Urol 1996;155:1938)
Etiology
Clinical features
  • Usual presentation is with a mass
  • < 5% present with metastatic symptoms
  • Paraneoplastic symptoms are rare (limbic encephalopathy, hypercalcemia, polycythemia, exophthalmus) (BMJ Case Rep 2014;2014:bcr2014206893)
  • Metastases are initially retroperitoneal and then progress to mediastinal and cervical nodes; visceral metastases develop late
Diagnosis
Laboratory
  • Serum LDH and PLAP may be elevated
  • hCG levels are increased in up to 20% of cases but elevation is modest (Pathol Annu 1991;26:59)
  • AFP should not be increased, if elevated think liver disease or nonseminoma component
Radiology description
  • Uniform, well delineated and hypoechoic mass on ultrasound
  • May be microlithiasis
Radiology images

Images hosted on other servers:
Missing Image

Ultrasound with testicle mass

Missing Image

Parietal metastasis

Prognostic factors
  • Clinical stage 1 has 95 - 98% 5 year survival
    • > 40 years and advanced stage are poor prognostic features (nonpulmonary metastases)
Case reports
Treatment
  • Surgery (radical orchiectomy) for primary
  • Surveillance protocols used for stage 1 disease
  • Radiation (very radiosensitive tumor) and chemotherapy (platinum based) used in higher stage disease
  • Retroperitoneal lymph node dissection used for residual post chemotherapy disease (J Natl Compr Canc Netw 2012;10:502)
Gross description
  • Well demarcated, homogeneous, solid cream or grey tumors; surface nodularity and lobulation; may be multiple nodules
  • Necrosis or hemorrhage usually minimal
  • If regressed, only a scar may be visible
  • Usually confined to testis (90%)
Gross images

Contributed by Debra L. Zynger, M.D.
Missing Image

pT1a

Missing Image

pT1b

Missing Image

pT2



Images hosted on other servers:
Missing Image

Very large seminoma

Missing Image

Seminoma

Missing Image

Lobulated soft tan to brown tissue

Frozen section description
  • Fibrous bands with lymphocytes and clear cells
  • Intra-operative smear / touch prep preparation will show large cells with abundant cytoplasm and admixed lymphocytes; tigroid type pattern (dense and light stripes) in air dried Giemsa smear
Frozen section images

Contributed by Michelle Downes, M.D. and Debra L. Zynger, M.D.
Missing Image

Intraoperative frozen section

Missing Image

Pale, foamy cytoplasm

Microscopic (histologic) description
  • Sheets or lobular configuration of tumor with fibrous septae
  • Cells are typically pale (glycogen) but may be eosinophilic
  • Cell membranes are well defined with distinct cell boundaries
  • Nuclei are polygonal and may have a flat edge giving a squared off appearance; they contain one or more prominent central nucleoli; no nuclear overlap if well fixed tissue
  • A lymphocytic infiltrate is present (T lymphocytes) with plasma cells; germinal centers may occur
  • Granulomas noted in up to 50% of cases
  • Intercellular edema with microcystic spaces and coagulative type necrosis can be present
  • Multinucleated syncytiotrophoblasts can be seen in 20% of tumors and can produce hCG
  • Intratubular seminoma may be adjacent to invasive seminoma (30% cases)
  • Less frequent growth patterns include intertubular pattern (may not form a mass), corded growth, microcystic, tubular, signet ring appearance
  • Anaplasia referring to increased mitoses and pleomorphism within a seminoma is no longer reported (no correlation with outcome) (J Urol 1996;155:1938)
  • Fibrosis and sclerosis can indicate regressed or burnt out seminoma
  • Lymphovascular invasion is uncommon, especially in tumors under 4 cm, and should be differentiated from artifactual carry over (Am J Clin Pathol 2016;145:341)
  • Invasion of rete testis can be direct stromal invasion or pagetoid invasion; direct stromal invasion carries prognostic significance in seminoma (Am J Surg Pathol 2017;41:e22)
Microscopic (histologic) images

Contributed by Michelle Downes, M.D. and Debra L. Zynger, M.D.
Missing Image

Fibrous septae

Missing Image

Admixed lymphocytes

Missing Image

Large cells with clear cytoplasm

Missing Image

Coagulative necrosis

Missing Image

OCT 3/4

Missing Image

CD117


Missing Image

Granulomatous inflammation

Missing Image

Granuloma

Missing Image

Intratubular seminoma

Missing Image

Rete testis invasion

Missing Image

Rete testis pagetoid spread

Missing Image

Hilar fat invasion


Missing Image

Epididymal invasion

Missing Image

Syncytiotrophoblasts

Missing Image

hCG

Missing Image Missing Image

Discontinuous spermatic cord invasion

Cytology description
  • Discohesive population of large cells with moderate cytoplasm, round slightly irregular nuclei and one or more prominent nucleoli
  • Background population of small, mature lymphocytes
  • Classic “tigroid smear” seen mostly in hypercellular aspirates (J Cytol 2011;28:39)
Cytology images

Contributed by Michelle Downes, M.D. and Debra L. Zynger, M.D.
Missing Image

Intraoperative touch prep

Missing Image

Large cells with prominent nucleoli

Missing Image

Tigroid pattern

Negative stains
Molecular / cytogenetics description
Sample pathology report
  • Right testis, mass, radical orchiectomy:
    • Seminoma, 3.6 cm (see synoptic report)
    • Comment: Immunohistochemistry was performed and the tumor expresses OCT 3/4 and CD117. The cells are negative with CD30 and AFP. The immunoprofile supports the morphologic interpretation of seminoma.
Differential diagnosis
Board review style question #1
    Which of the following stains is appropriate for differentiating between seminoma and embryonal carcinoma?

  1. AFP
  2. CD30
  3. hCG
  4. OCT 3/4
  5. SALL4
Board review style answer #1
B. CD30

Comment Here

Reference: Seminoma
Board review style question #2

    A 38 year old man presents with a unilateral, painless palpable mass in the left testis. His serum AFP and hCG are normal with mild LDH elevation. The histology of his orchiectomy specimen is shown. Where would be the most typical first site for metastasis?

  1. Bone
  2. Brain
  3. Groin nodes
  4. Lung
  5. Retroperitoneal nodes
Board review style answer #2
E. Retroperitoneal nodes. This is a photo of seminoma.

Comment Here

Reference: Seminoma

Serous borderline tumor
Definition / general
  • Serous epithelial tumor of low malignant potential occurring in paratesticular region
Terminology
  • Same as ovarian counterparts
Epidemiology
  • Rare tumor but more common than malignant serous epithelial tumors
  • Mean patient age in the largest reported series was 56 years, range of 14 - 77 years (Am J Surg Pathol 2001;25:373)
Sites
  • Tunica is most common site, commonly arising from tunica albuginea
  • Tunica vaginalis, spermatic cord are other common sites
  • Tumor may extend to testis
Etiology and Pathophysiology
  • Postulated cell of origin is Müllerian metaplasia of mesothelial lining
  • Cases arising in testis are explained by Müllerian metaplasia occurring in mesothelial cell inclusions within testicular parenchyma
  • Embryological Müllerian remnants within testis and paratestis are also postulated to give rise to these tumors
Clinical features
  • Painless testicular mass in elderly male is most common presentation
Diagnosis
  • Suspected clinico - radiologically and confirmed histologically
Laboratory
  • No specific laboratory abnormality
Radiology description
  • Cystic lesion arising in paratesticular / testicular region
  • Broad papillae may be visualized but there are no specific diagnostic criteria
Prognostic factors
  • No reports of recurrence of serous borderline tumors but lesion is rare and reported follow up is only 2 years
Case reports
Treatment
  • Radical orchidectomy is treatment of choice
  • No cases reported have recurred with or without chemotherapy but need for adjuvant chemotherapy needs to be assessed for each case
Gross description
  • 1 - 6 cm grossly cystic lesion arising within paratestis / testis
  • Cyst wall is variably thickened with protrusions
  • Papillae may be visualized focally but usually are not a prominent feature
Microscopic (histologic) description
  • Histologically identical to their ovarian counterpart
  • Tumors are cystic with numerous intracystic blunt papillae lined by ciliated stratified columnar cells with minimal to mild cytologic atypia
  • Psammoma bodies may be present
  • Cyst wall shows variable amount of fibrous tissue
  • Ki67 staining index varies from 1% to 10%
Cytology description
  • Fluid smears with scattered epithelial cells
  • Occasional papillary frond may be visualized but is usually not a prominent feature
Positive stains
Negative stains
Molecular / cytogenetics description
  • No specific cytogenetic abnormality has been documented
Differential diagnosis

Serous cystadenocarcinoma
Terminology
Epidemiology
Sites
  • Paratesticular region
  • Commonly involves paratesticular soft tissue in testicular - epididymal groove, visceral tunica vaginalis and rete testis
  • Rarely is confined to testicular parenchyma
Pathophysiology
  • Same as other serous tumors of ovarian type
  • Possible origin from Müllerian metaplasia of mesothelial lining, testicular mesothelial inclusions, embryonic Müllerian remnants
Clinical features
  • Most patients present with a scrotal mass with hydrocele of 4 - 6 weeks duration
  • Less commonly patients present with a hydrocele alone
  • Presentation with metastases is uncommon
Diagnosis
  • Suspected clinico - radiologically and confirmed histologically
  • Clinico - radiological features do not sufficiently distinguish this tumor from other similar tumors in this region, which are more common
Laboratory
  • CA125 may be elevated but is not a consistent feature
Radiology description
  • USG: commonly a 2.0 - 3.5 cm paratesticular mass, associated with a hydrocele
  • If predominantly cystic, may be associated with papillary or solid area at one end
Prognostic factors
  • Has high progression rate and commonly has late metastases
Case reports
Treatment
  • Treatment of choice is high inguinal orchidectomy, followed by chemotherapy
  • Role of inguinal lymph node dissection is not established
Gross description
  • Commonly 1.5 - 3.0 cm
  • Solid, whitish tan, poorly circumscribed, often gritty masses involving soft tissue between testis and epididymis, paratesticular soft tissue or visceral tunica vaginalis
  • Papillary fronds may be visible in grossly cystic tumors
  • Invasive foci are commonly associated with a fibrous reaction
Gross images

Images hosted on other servers:

Intratesticular mass

Microscopic (histologic) description
  • Invasive, well formed papillae lined by serous cuboidal or columnar cells with eosinophilic cytoplasm and marked atypia
  • Abundant psammoma bodies
  • Areas of borderline serous tumor often present and rarely can be the dominant component
  • Most tumors have nuclear grade II or III
Microscopic (histologic) images

Images hosted on other servers:
Missing Image

WT1+

Positive stains
Negative stains
Electron microscopy description
  • Gland formation with delicate luminal microvilli and cilia
Differential diagnosis

Sertoli cell nodule
Definition / general
Essential features
  • Usually an incidental finding in adult testis
  • Commonly seen in cryptorchid testis, also identified in normal testis
  • Nodule size usually varies from microscopic to 5 mm; rarely can reach up to 10 mm
  • Comprised of seminiferous tubules containing proliferation of immature Sertoli cells
Terminology
  • Sertoli cell hyperplasia
  • Tubular dysgenesis
  • Hypoplastic zones
  • Pick adenoma
ICD coding
  • ICD-10: D29.20 - benign neoplasm of unspecified testis
Epidemiology
Sites
  • Testes
Pathophysiology
  • Exact pathophysiology of Sertoli cell nodules is unknown
  • Proliferation of Sertoli cells has been identified in the setting of
  • Presence of immature Sertoli cells in cryptorchid testes could be explained by different factors including
Etiology
  • Primary testicular lesion
  • Seen within the spectrum of testicular dysgenesis syndrome
  • Proliferation of seminiferous tubules which are unable to undergo pubertal development
  • Reference: Clin Endocrinol (Oxf) 2009;71:459
Clinical features
Diagnosis
  • Usually an incidental finding on histological examination
Laboratory
  • No significant laboratory findings
Radiology description
Radiology images

Contributed by Raman Danrad, M.D.
Ultrasound of left testis

Ultrasound of left testis

Prognostic factors
Case reports
Treatment
Gross description
Gross images

Images hosted on other servers:
Demarcated, bulging, pale yellowish nodules

Demarcated, bulging, pale yellowish nodules

Sharply demarcated, yellowish white nodule

Sharply demarcated, yellowish white nodule

Microscopic (histologic) description
  • Circumscribed, unencapsulated nodules, comprised of
    • Prepubertal sized seminiferous tubules
    • Profiles of tubules vary from round to oval to hourglass in shape
    • Lined by columnar to pseudostratified epithelium composed of Sertoli cells with elongated, hyperchromatic nuclei, 1 or more peripherally located nucleoli and scant cytoplasm
    • Tubules are devoid of lumina; however, may contain globular hyaline material deposits, due to invagination of the excess basal lamina produced by Sertoli cells
      • Sertoli cells can be arranged in a ring shaped manner around the hyaline material
      • Basal lamina (basement membrane) type material may undergo calcification
    • Isolated spermatogonia may be observed in some nodules
  • Interstitium varies from scant to well collagenized
    • Leydig cells are usually absent or low in numbers
  • References: MacLennan: Urologic Surgical Pathology, 4th Edition, 2019, Am J Surg Pathol 2010;34:1874, J Clin Pathol 2004;57:802
Microscopic (histologic) images

Contributed by Ritu Bhalla, M.D. and Jian-Hua Qiao, M.D.
Unencapsulated nodule

Unencapsulated nodule

Seminiferous tubule proliferation

Seminiferous tubule proliferation

Focal Leydig cells

Focal Leydig cells

Hyaline basement membrane deposits

Hyaline basement membrane deposits

Pseudostratified Sertoli cells

Pseudostratified Sertoli cells

Arrangement of Sertoli cells

Arrangement of Sertoli cells


Calcification

Calcification

Hyperplastic Sertoli cells

Hyperplastic Sertoli cells

Atrophic seminiferous tubules

Atrophic seminiferous tubules

Persistent immature tubules

Persistent immature tubules

Hyalinization of basement membrane

Hyalinization of basement membrane

Atrophic seminiferous tubules

Atrophic seminiferous tubules

Positive stains
Electron microscopy description
  • Hyaline material in light microscopy shows as compact multilamellar matrix in electron microscopy (J Clin Pathol 2004;57:802)
Electron microscopy images

Images hosted on other servers:
Thickened, multilamellar basement membranes

Thickened, multilamellar basement membranes

Hyaline core

Hyaline core

Sample pathology report
  • Testes, radical orchiectomy:
    • Sertoli cell nodule (see comment)
    • Comment: Minute nodular collection of seminiferous tubules comprised predominantly of Sertoli cells with intratubular eosinophilic basement membrane-like material, with background of benign testicular parenchyma.
Differential diagnosis
  • Sertoli cell adenoma and tubular hamartoma:
    • Usually larger in size
    • Almost exclusively seen in patients with androgen insensitivity syndrome (AIS)
    • Unencapsulated nodules composed of solid tubules of immature Sertoli cells
    • Sertoli cells show spherical nuclei and lack pseudostratification
    • Tubular hamartoma has a densely cellular interstitium with fusiform cells and numerous Leydig cells
    • Sertoli cell adenoma lacks interstitial Leydig cells
    • Absence of intratubular globular basement membrane deposits
  • Sertoli cell tumor, not otherwise specified:
    • Usually larger in size (> 1 cm)
    • Larger cells with vesicular nuclei and prominent nucleoli arranged in cords and tubules
    • Nuclear atypia and mitotic activity may be present
    • Absence of basal membrane thickening and germ cells
  • Intratubular large cell hyalinizing Sertoli cell neoplasia:
    • Multiple nodules composed of tubules with larger diameter
    • May show thickening of basal membranes and intratubular globular basement membrane deposits
    • Sertoli cells with higher grade of maturation (vesicular nuclei with central nucleoli and large eosinophilic cytoplasm)
    • Absence of germ cells
    • Associated with Peutz-Jeghers syndrome
  • Gonadoblastoma:
    • Nests consist of Sertoli cells and large, atypical germ cells
    • Diffuse nodular distribution
    • Germ cells positive for OCT 3/4
  • Sex cord stromal tumor of testis with entrapped germ cells:
    • Usually larger in size
    • Tubular and cord-like arrangements of sex cord cells
    • Germ cells are mostly present at the periphery and in clusters
    • Absence of prominent basement membrane deposits
  • Testes with focal Sertoli only cell tubules:
    • Contain a mix of Sertoli only tubules and tubules with germ cells
    • Sertoli only tubules are not clustered
    • Size of tubules and degree of Sertoli cell maturations is higher
    • Absent intratubular globular basement membrane deposits
  • References: MacLennan: Urologic Surgical Pathology, 4th Edition, 2019, Am J Surg Pathol 2010;34:1874
Board review style question #1


A 30 year old man presents for evaluation of a nonpainful right testicular firmness. Ultrasound imaging showed a hyperechogenic lesion and orchiectomy was performed to rule out testicular neoplasm. Gross examination reveals a well circumscribed, white-tan, firm nodule measuring 8 mm in its greatest dimension with microscopic appearance as shown in the images above. What additional intervention is necessary after diagnosis?

  1. Chemotherapy
  2. Contralateral orchiectomy
  3. No additional intervention is necessary
  4. Retroperitoneal lymph node dissection
Board review style answer #1
C. No additional intervention is necessary. The image reflects a Sertoli cell nodule, referred by some studies as macroscopic, being identified grossly with a size of 8 mm. Answers A, B and D are incorrect because Sertoli cell nodule is a nonneoplastic proliferation of immature Sertoli cells with no reported malignant potential; hence, no other intervention is required after histologic diagnosis.

Comment Here

Reference: Sertoli cell nodule
Board review style question #2


A 25 year old man presents for evaluation of left lower abdominal pain. Imaging reveals the location of the left testicle in the inguinal canal. Subsequent orchiectomy revealed an unremarkable cut surface. Microscopic findings showed multiple, well circumscribed nodules with the appearance as shown in the images above. Which of the following immunohistochemical stains is expected to be positive?

  1. hCG
  2. Inhibin
  3. OCT 3/4
  4. PLAP
Board review style answer #2
B. Inhibin. Sertoli cell nodule is composed mainly of immature Sertoli cells and inhibin helps to highlight and confirm their presence. Answers C, D and A are incorrect because OCT 3/4, PLAP and hCG are negative in Sertoli cell nodule and are helpful to rule out the presence of germ cell neoplasms.

Comment Here

Reference: Sertoli cell nodule
Board review style question #3
Which of the following is a common finding in Sertoli cell nodules?

  1. Atypical mitoses
  2. Globular hyaline material deposits
  3. Nuclei in a syncytium
  4. Numerous spermatogonia
Board review style answer #3
B. Globular hyaline material deposits. Sertoli cells produce an excess of basal lamina that invaginates into the involved tubules. Answer D is incorrect because spermatogonia, when present, are scattered. Answer A is incorrect because typical mitoses are not a common finding of Sertoli cell nodules and are more common in malignant entities. Answer C is incorrect because nuclei in a syncytium (syncytiotrophoblasts) are usually seen in germ cell tumors and are not an expected finding in Sertoli cell nodules.

Comment Here

Reference: Sertoli cell nodule

Sertoli cell only
Definition / general
  • Also known as Sertoli cell only syndrome
  • First derscribed by Del Castillo et al. (1947)
Etiology
  • Causes: idiopathic, gonadotropin deficiency, cryptorchidism, orchitis, prostate cancer hormonal therapy, radiation, Klinefelter syndrome (Arch Pathol Lab Med 2010;134:1197)
Germ cell aplasia and focal spermatogenesis
  • 2 populations of tubules; one population with small tubules exhibiting germ cell aplasia, other population has reduced spermatogenesis
  • Usually very low sperm count
  • Some prefer the term "hypospermatogenesis" for this setting (Arch Pathol Lab Med 2010;134:1197)
Microscopic (histologic) description
  • Tubules are reduced in diameter and lined only by Sertoli cells
  • Sertoli cells are perpendicular to basement membrane; may resemble palm trees waving in a breeze
  • Cells have nuclear indentations and prominent nucleoli
  • Charcot-Bottcher crystals sometimes seen as thin eosinophilic lines in various directions
  • 4 types of Sertoli cells have been identified
    1. Normal adult mature cells
    2. Immature cells
    3. Dysgenetic cells
    4. Involuting cells (Histopathology 1990;16:173)
Microscopic (histologic) images

Contributed by Asmaa Gaber Abdou, M.D.

Sertoli cell only and Leydig cell hyperplasia

Differential diagnosis

Sertoli cell tumor
Definition / general
  • Sex cord stromal tumor of testis composed of cells showing features of fetal, prepubertal, adult or atrophic Sertoli cells, at least focally
Essential features
  • Second most common type of pure sex cord stromal tumor after Leydig cell tumor
  • Most are small, unilateral tumors in adults and the vast majority are benign
  • Characterized by tubular or cord pattern, at least focally
  • Positive for SF1 and nuclear beta catenin
  • Variable and inconsistent expression of calretinin, inhibin A, androgen receptor, CD56
  • Negative for SALL4, OCT4 and other germ cell markers
  • Poor response to therapy when metastatic
Terminology
  • Androblastoma (obsolete)
ICD coding
  • ICD-O:
    • 8640/1 - Sertoli cell tumor
    • 8640/3 - malignant Sertoli cell tumor
Epidemiology
  • < 1% of testicular tumors
  • Rare before age 20; mean age 45 years
Etiology
Clinical features
Diagnosis
  • For suspected testicular mass, the first test is scrotal ultrasound (US) (sensitivity 95%, specificity 97%)
  • If US confirms a lesion, the patient should be immediately referred to a urologist for evaluation and management
  • Men 15 to 44 years old with retroperitoneal mass / metastases should undergo testicular examination and scrotal US
  • Evaluate serum markers
  • Biopsy of testicular tumors is in general not performed, to prevent tumor seeding
  • Staging:
    • In clinically stable patients, imaging may be performed before or after orchiectomy
    • Imaging: computed tomography (CT) scan of the abdomen / pelvis with intravenous contrast, chest radiograph (CXR)
    • Noncontrast chest CT should be performed if CXR is abnormal
  • Reference: Med Clin North Am 2018;102:251
Laboratory
  • Serum tumor markers:
    • Beta human chorionic gonadotropin (hCG), alpha fetoprotein (AFP) should be normal and are used to exclude nonseminomatous germ cell tumors
    • Lactate dehydrogenase (LDH) to assess global tumor burden
Radiology description
  • Ultrasound: solitary hypoechoic lesion; not helpful in distinguishing from germ cell tumors
Radiology images

Images hosted on other servers:
Hypoechoic lesion

Hypoechoic lesion

Prognostic factors
  • Vast majority are benign, ~ 5% cases are malignant
  • Tumors with sclerosis > 50% almost invariably benign (Am J Surg Pathol 2014;38:510)
  • Features associated with malignancy: extratesticular spread, size > 5 cm, lymphovascular invasion, high grade cytological atypia, tumor necrosis, mitotic index > 5/10 high power fields (Am J Surg Pathol 1998;22:709)
  • Very rarely tumors without malignant histology metastasize
Case reports
Treatment
  • Orchiectomy (total or partial) is treatment of choice
  • Retroperitoneal lymph node dissection for malignant Sertoli cell tumors
  • No specific therapy; poor response to chemotherapy or radiotherapy
  • Reference: Med Clin North Am 2018;102:251
Gross description
  • Cut surface usually homogenous white or yellowish
  • Cystic component present in ~ 30% of cases (Am J Surg Pathol 1998;22:709)
  • Benign: well circumscribed, most between 2 - 5 cm (Am J Surg Pathol 2014;38:510)
  • Malignant: > 5 cm, poor circumscription, extratesticular extension, necrosis, hemorrhage
Gross images

Images hosted on other servers:
Tumor in lower pole

Tumor in lower pole

Microscopic (histologic) description
  • Architecture: tubular (lumen), cords (no lumen), tubulopapillary present at least focally
  • Other patterns: macro or microcystic, nested, trabecular, whorled, solid, retiform, pseudopapillary; most tumors show mixed patterns
  • Nuclei: bland, uniform, round to ovoid in benign tumors; small hyperchromatic or large with prominent nucleoli in malignant tumors
  • Cytoplasm: usually clear and abundant but extremely variable, ranging from scant, foamy, eosinophilic to markedly lipidized; hyaline globules are common
  • Stroma: variable, basement membrane-like material around tubules, sclerotic, myxoid, edematous, angiomatous
  • If sclerosis > 50% → sclerotic variant (now considered subtype of Sertoli cell tumor, NOS)
  • Inflammatory cells usually absent, rarely prominent
  • References: Histopathology 2017;70:513, Ulbright: Tumors of the Testis and Adjacent Structures (AFIP Atlas of Tumor Pathology, Series 4), 1st Edition, 2014, Am J Surg Pathol 2015;39:1390
Microscopic (histologic) images

Contributed by Thomas Ulbright, M.D. and Case #18

Architectural patterns
Cords

Cords

Tubules

Tubules

Tubopapillary

Tubopapillary

Macrocystic

Macrocystic

Microcystic Microcystic

Microcystic



Architectural patterns
Whorled

Whorled

Trabecular

Trabecular


Sclerosing Sertoli cell tumor

Various images



Cells
Abundant clear cytoplasm (most common)

Abundant clear cytoplasm (most common)

Heavily lipidized

Heavily lipidized


Malignant
Marked cytologic atypia

Marked cytologic atypia

Tumor necrosis

Tumor necrosis

Lymphovascular invasion

Lymphovascular invasion

High N/C ratio, increased mitotic / apoptic activity

High N/C ratio,
increased
mitotic / apoptotic
activity

Cytology description
Cytology images

Contributed by Indiana University School of Medicine
Tubular formations, monotonous small cells (Pap)

Tubular formations, monotonous small cells (Pap)

Papillary groups and dyscohesive cells (Pap)

Papillary groups and discohesive cells (Pap)

Negative stains
Electron microscopy description
Molecular / cytogenetics description
Sample pathology report
  • Left testis, orchiectomy:
    • Sertoli cell tumor, not otherwise specified (see comment and synoptic report)
    • Comment: The morphologic features are diagnostic of Sertoli cell tumor, not otherwise specified. No features of malignancy are identified in this tumor (i.e. extratesticular spread, size > 5 cm, high grade cytologic atypia, > 5 mitotic figures per 10 high power fields, necrosis or lymphovascular invasion).
Differential diagnosis
Board review style question #1

A 60 year old man presents with a 1.1 cm mass in the left testis. The tumor is positive for SF1, nuclear beta catenin, androgen receptor, variable for inhibin A and calretinin and negative for SALL4. What is the most likely diagnosis?

  1. Adenomatoid tumor
  2. Seminoma
  3. Sertoli cell tumor, NOS
  4. Yolk sac tumor
Board review style answer #1
C. Sertoli cell tumor, NOS

Comment Here

Reference: Sertoli cell, NOS
Board review style question #2

What is the characteristic immunoprofile of a testicular Sertoli cell tumor, NOS?

  1. Positive for SF1, calretinin, chromogranin, synaptophysin; negative for SALL4 and beta catenin
  2. Positive for SALL4; negative for SF1, inhibin A and calretinin
  3. Positive for SF1, calretinin, inhibin A, beta catenin (nuclear); negative for SALL4, chromogranin, synaptophysin
  4. Positive for CD45; negative for SF1, inhibin A and SALL4
Board review style answer #2
C. Positive for SF1, calretinin, inhibin A, beta catenin (nuclear); negative for SALL4, chromogranin, synaptophysin

Comment Here

Reference: Sertoli cell, NOS

Sex cord stromal tumors-general
Definition / general
  • Includes neoplasms of pure sex cord and pure stromal type as well as those with admixtures of both components in various proportions and degree of differentiation, each strikingly different in clinical features and biology
  • Sex cord elements are usually positive for steroid producing immunohistochemical cell markers (e.g., inhibin, SF1); in addition, the presence of stromal elements is easily confirmed by a reticulin stain that shows the framework surrounding individual tumor cells
  • Some arise in the setting of androgen insensitivity syndrome (AIS) or adrenogenital syndrome (AGS), designated as tumor-like lesion occurring in specific syndrome
Essential features
  • Includes neoplasms of pure sex cord and pure stromal type as well as those with admixtures of both components
  • Uncommon testicular tumors with a greater proportion in children (25%)
  • Typically show indolent behavior; only 5% are malignant
  • Usually positive for steroid producing immunohistochemical cell markers (e.g., inhibin, SF1)
Terminology
Epidemiology
Sites
  • Testis
Pathophysiology
  • Poor gonadal development may lead to testicular cancer; this follows the hypothesis of testicular dysgenesis syndrome
  • Various histologic studies show dysgenetic features such as undifferentiated tubules, immature Sertoli cells, etc. in testicular cancer
  • Altered functioning of somatic cells (Leydig and Sertoli) due to mutations / polymorphism, along with environmental and lifestyle factors, act during early development; these dysfunctioning somatic cells lead to disturbed hormone balance leading to altered germ cell differentiation (Semin Cell Dev Biol 2015;45:124)
Etiology
Clinical features
  • Usually presents as testicular mass
  • Functional tumors are generally uncommon, except for Leydig cell tumors (unlike ovarian sex cord stromal tumors)
Diagnosis
  • Tumor histology and immunohistochemistry
Prognostic factors
  • Usually indolent behavior; 5% are malignant
  • Morphologic prognostic factors include size, mitotic rate, necrosis, cellular atypia, infiltrative borders and vascular invasion
  • Stage is the most important prognostic factor
  • Stage with TNM for malignant tumors
Case reports
Treatment
  • High inguinal orchidectomy is treatment of choice
  • Testicular sparing surgery may be attempted in small tumors
  • Retroperitoneal lymph node dissection if clinical / radiological evidence of metastatic disease
Immunohistochemistry & special stains
  • Vimentin is typically positive but not specific
  • Keratin is frequently positive in Sertoli cell tumors, while staining is focal / absent in Leydig cell tumors and granulosa cell tumors and typically negative in fibromas
  • EMA is typically negative
  • Inhibin positivity has been shown in juvenile granulosa cell tumors and Leydig cell tumors even at metastatic sites but Sertoli cell tumors are not uniformly positive (25 - 90%)
  • Calretinin in postpuberal testis is expressed in Leydig cells but less commonly in Sertoli cells
  • SF1 is highly specific (negative in germ cell tumors)
  • FOXL2 positivity and mutations have been detected in granulosa cell tumors
  • MelanA is often positive in Leydig cell tumors
  • CD99 is expressed in normal Sertoli and granulosa cells and in testicular sex cord stromal tumors but it is not a reliable marker for sex cord stromal tumors
  • S100 may stain normal Sertoli and Leydig cells and rete testis, as well as Sertoli cell tumors (including large cell variant), Leydig cell tumors and sex cord stromal tumors - unclassified
  • Fibrothecomas are often positive for inhibin (patchy to diffuse), calretinin and keratin; they can also stain for MelanA, BCL2, CD34, S100, muscle specific actin and desmin
  • PAX2 and PAX8 are typically negative
  • SALL4 is negative (positive in germ cell tumors)

Immunohistochemistry of sex cord stromal tumors
 Calretinin   Inhibin   S100   Keratin   SF1   Vimentin   SMA   FOXL2   MelanA 
 Sertoli cell tumor + / - + / - + / - + + + - / + -
 Large cell calcifying Sertoli cell tumor + + + / - + / - + + +
 Leydig cell tumor + + + / - + / - + + / - - +
 Adult granulosa cell tumor + + + / - - / + + + - / + + / - -
 Juvenile granulosa cell tumor + + + +
 Fibroma thecoma tumors + + / - + / - + +
 Mixed and unclassified sex cord stromal tumors  + + + +
 Sex cord stromal tumor, NOS  + / - + / - + - / + + + + + + / -
 Myoid stromal tumor  + / - + + +

Molecular / cytogenetics description
  • No pathognomonic findings established
  • Lack hotspot mutations in DICER1 (in contrast to ovarian sex cord stromal tumors) (Mod Pathol 2015;28:1603)

Smooth muscle hyperplasia
Definition / general
  • Benign nonneoplastic smooth muscle hyperplasia involving testicular adnexa around vessels or efferent ducts
Epidemiology
Sites
  • Epididymis is most common site
  • Can involve rete testis, tunica vaginalis, tunica albuginea, spermatic cord
Etiology & Pathophysiology
  • Exact cause not known
  • Obstruction of epididymis or vas deferens ducts hypothesized to be initiating event
Clinical features
  • Most common presentation is painless scrotal mass of long standing duration
  • May have periodic pain but usually not a prominent feature
Diagnosis
  • Suspected clinico - radiologically and confirmed histologically
Laboratory
  • No specific laboratory feature
  • Negative tumor markers for germ cell and sex cord stromal tumors
Radiology description
  • Ultrasound helps delineate the lesion, though it may be difficult to exclude involvement of adjacent testis
  • May appear solid or multicystic
Prognostic factors
  • Benign nonneoplastic lesion
  • No recurrence or progression has been reported
Case reports
Treatment
  • May be excised for diagnosis and treatment
  • No further treatment required
Gross description
  • Mean 2.5 cm, range 6 mm to 7 cm
  • Discrete or ill defined nodules, fusiform or cordlike masses, thickened or diffusely enlarged paratesticular structures
Gross images

Images hosted on other servers:

Well circumscribed multicystic mass

Microscopic (histologic) description
  • Irregular fascicles of smooth muscle in periductal, perivascular, interstitial or mixed pattern
  • No cohesive interlacing growth pattern of leiomyoma
  • May be associated with secondary myxoid change, calcification, hemorrhage, fibrosis
  • May be associated with ectasia of involved ducts, microscopic adenomatoid tumor, vasitis nodosa
  • No cellular atypia, no mitosis, no necrosis
Microscopic (histologic) images

Images hosted on other servers:

Columnar epididymal type epithelium

Cystic duct

Mass blends with ducts in epididymal tail

Immunohistochemistry & special stains
  • Positive for smooth muscle markers but immunohistochemistry not particularly useful in differential diagnosis
Differential diagnosis
  • Leiomyoma: well circumscribed interlacing fascicles of smooth muscle, which displace rather than encase / invade the adjoining structures
  • Leiomyosarcoma: malignant tumor characterized by high cellularity, mitosis, necrosis and cellular atypia and associated with invasive features

Spermatocele
Definition / general
  • Benign dilatation of efferent ductules in the rete testis or head of epididymis
  • Size ranges from a few millimeters up to 2 cm
Essential features
  • Benign dilatation of the efferent ductules in the rete testis or head of the epididymis
  • Usually idiopathic but cystic dilation from outflow obstruction may play a role
  • Diagnose by ultrasound demonstrating a well defined simple cyst
  • Lined by cuboidal, columnar, ciliated or flattened epithelium
  • Cystic contents often contain spermatozoa
Terminology
  • Often indistinguishable from epididymal cyst
Epidemiology
  • All ages affected but typically occurs in 20 - 50 year old men
Sites
  • Testis, epididymis
Etiology
Clinical features
  • Painless bulging mass separate from testicle
Diagnosis
  • Diagnosed by radiological imaging, including ultrasound and less commonly by MRI, followed by pathological evaluation of the resection specimen
Radiology description
  • By ultrasound, simple cysts are generally well defined and anechoic; however, low level echos (falling snow sign) or sedimentation levels can be seen (Clin Radiol 2008;63:929, Abdom Radiol (NY) 2020;45:587)
  • Cyst may displace the testis if it is large
  • Distinct from hydrocele, which envelops the testis but does not displace it
Radiology images

Images hosted on other servers:
Ultrasound images Ultrasound images Ultrasound images

Ultrasound images

Case reports
Treatment
  • Surgical resection, spermatocelectomy
Gross description
  • Arrives in laboratory as a collapsed, thin walled membrane (Urology 1984;24:372)
  • Round or ovoid
  • Cysts may be unilocular or multilocular and translucent (Urology 1984;24:372)
  • Most commonly arises from the head of the epididymis
Gross images

Contributed by Anil Parwani, M.D., Ph.D., M.B.A.
Orchiectomy with spermatocele

Orchiectomy
with spermatocele

Microscopic (histologic) description
  • Initially lined by single layer of cuboidal to ciliated columnar epithelium which becomes flattened as fluid accumulates (Urology 1984;24:372)
  • Thin wall of fibromuscular soft tissue
  • Lumen, cyst wall and adjacent soft tissue contains spermatozoa and proteinaceous fluid (Diagn Cytopathol 2010;38:816)
  • Associated with foreign body giant cell reaction, cholesterol clefts (Diagn Cytopathol 2010;38:816)
  • Epithelial lining may form papillary structures with fibromuscular cores lined by a single layer of columnar to cuboid epithelium with vacuolated cytoplasm
Microscopic (histologic) images

Contributed by Anil Parwani, M.D., Ph.D., M.B.A.
Spermatocele with adjacent epididymis

Spermatocele with adjacent epididymis

Cuboidal epithelium

Cuboidal epithelium

Ciliated columnar epithelium

Ciliated columnar epithelium

Flattened epithelium

Flattened epithelium

Aggregate of spermatozoa

Aggregate of spermatozoa

Macrophages containing spermatozoa

Macrophages containing spermatozoa

Cytology description
  • Cysts can contain spermiophages, macrophages with abundant distended cytoplasm containing intact spermatozoa (Diagn Cytopathol 2010;38:816)
Sample pathology report
  • Cyst, spermatocelectomy:
    • Spermatocele (see comment)
    • Comment: A cyst with bland cuboidal lining is present, compatible with spermatocele.
Differential diagnosis
  • Hydrocele:
    • Originates between the parietal and visceral layers of tunica vaginalis
    • Lining is mesothelial; lumen does not contain spermatozoa
  • Mesothelial cyst:
    • Usually originates from tunica vaginalis or tunica albuginea
  • Cystic dysplasia of rete testis:
    • Congenital lesion of newborns that is associated with ipsilateral renal agenesis
  • Sertoliform cystadenoma of rete testis:
  • Simple cyst of rete testis:
    • Bulges into the testis proper
    • Common in men undergoing renal dialysis
    • Columnar epithelium, calcium oxalate crystals, fibrosis and giant cells
  • Tubular ectasia of rete testis:
  • Dermoid cyst:
    • Rarely presents within the spermatic cord or as a paratesticular mass
Board review style question #1
Which of the following is a feature of spermatocele?

  1. Has a lining of mesothelial cells
  2. Is cytokeratin negative
  3. May have papillary infoldings with fibrovascular cores lined by vacuolated cuboidal or columnar cells
  4. Originates from the same membrane as hydrocele
Board review style answer #1
C. May have papillary infoldings with fibrovascular cores lined by vacuolated cuboidal or columnar cells. A benign papilloma consisting of papillary infoldings with fibrovascular cores lined by vacuolated or columnar cells may arise from the epithelial lining of the spermatocele. The lining can be cuboidal, columnar, ciliated or flattened epithelium, not mesothelium (answer A). Spermatocele is cytokeratin positive and can be distinguished from the mesothelial lining of a hydrocele by negative staining for D2-40 or calretinin (answer B). The origin of spermatocele is from rete testis / head of epididymis epithelium, whereas hydrocele originates from the mesothelium of the tunica vaginalis (answer D).

Comment Here

Reference: Spermatocele
Board review style question #2

A paratesticular lesion is identified. Which of the following is the origin of the entity shown above?

  1. Head of the epididymis or rete testis
  2. Pampiniform venous plexus
  3. Totipotential germ cells
  4. Tunica vaginalis
Board review style answer #2
A. Spermatoceles often originate from the epithelial lining of the epididymis. Varicoceles commonly originate from the pampiniform venous plexus (answer B). Dermoid cysts commonly originate from totipotential germ cells (answer C). Hydroceles commonly originate from the tunica vaginalis (answer D).

Comment Here

Reference: Spermatocele

Spermatocytic tumor
Definition / general
  • Spermatocytic tumor (ST) is a polymorphous triphasic germ cell neoplasm recapitulating spermatogonia development and unrelated to germ cell neoplasia in situ (GCNIS)
Essential features
  • Does not arise from germ cell neoplasia in situ
  • More common in older patients compared with patients with other germ cell tumors
  • Classically characterized by tripartite tumor cells with 3 different cellular sizes (small, intermediate, giant)
  • Immunohistochemical phenotype: CD117+, SALL4+, PLAP-, OCT3/4-, CD30-
  • Excellent prognosis
Terminology
  • Spermatocytic seminoma (not recommended by the WHO)
  • Spermatocytoma (not recommended by the WHO)
  • Spermatocytic variant of classic seminoma (not recommended by the WHO)
  • Type III germ cell tumor (not recommended by the WHO)
ICD coding
  • ICD-O: 9063/3 - spermatocytic seminoma
  • ICD-10: C62.9 - malignant neoplasm of testis, unspecified whether descended or undescended
  • ICD-11: XH80D1 - spermatocytic seminoma (inclusions: spermatocytic tumor, spermatocytoma)
Epidemiology
  • ~1% of germ cell tumors
  • Usually affects older men compared with germ cell neoplasia in situ derived tumors (mean age 52 years)
  • Wide age range, including young patients (range 19 - 92 years)
  • 30% of patients are 30 - 39 years; 35% are 40 - 59 years; 35% are ≥ 60 years (Am J Surg Pathol 2019;43:1)
  • No ethnic predisposition
Sites
  • Testes
  • 9% bilateral (synchronous or more frequently, metachronous)
  • No primitive extragonadal cases reported so far
  • Only 1 case of an ovarian spermatocytic tumor-like lesion reported in a gonadal mixed germ cell tumor (Am J Surg Pathol 2017;41:1290)
Pathophysiology
Etiology
Clinical features
  • Usually presents as a painless testicular nodule
  • Rapid tumoral growth is associated with the presence of sarcomatous differentiation (Int J Surg Pathol 2017;25:559)
Diagnosis
Laboratory
  • Not associated with elevated lactate dehydrogenase (​LDH), alpha fetoprotein (AFP) or beta human chorionic gonadotropin (beta hCG) blood concentration
Radiology description
  • Well circumscribed (Radiographics 2017;37:1085)
  • Predominantly solid with heterogenous signal, including hyperechoic and hypoechoic components
  • Multiple cystic areas
Radiology images

Images hosted on other servers:
ST (arrow) and testis (*)

ST (arrow) and testis (*)

Prognostic factors
  • Excellent prognosis overall
  • ST with sarcomatous differentiation has a poor prognosis, a high risk of metastasis of the sarcomatous component to the lungs and a high mortality rate
  • Some early reports of metastatic ST without sarcomatous differentiation have been reconsidered to be lymphomas (Cancer 1965;18:751, Cancer 1969;24:92)
  • Only rare cases of bona fide metastatic ST without sarcomatous differentiation have been reported with spread to retroperitoneal lymph nodes, to the lungs and to the brain
  • Intravascular space invasion or infiltration beyond testicular parenchyma were reported in metastatic ST without sarcomatous differentiation (Pathol Oncol Res 2017;23:223)
  • Anaplastic ST is not currently considered to be a negative prognostic factor
Case reports
Treatment
Gross description
  • Variable size (mean 7 cm; range 1.4 - 28 cm)
  • Homogenous or lobulated nodule
  • Grayish white
  • Edematous or mucoid
  • Soft
  • Necrosis and hemorrhage can be present
  • Extension beyond the tunica albuginea might be present (Am J Surg Pathol 2019;43:1)
  • Fleshy and firm areas, with or without necrosis, might represent foci of sarcomatous differentiation
Gross images

AFIP images

Mucoid tumor



Images hosted on other servers:
ST (arrow)

ST (arrow)

Cystic and mucoid tumor

Frozen section description
  • Recognition of 3 cell types (small, intermediate, giant) is key to the correct diagnosis
  • Intratubular ST in the surrounding parenchyma might be misinterpreted as germ cell neoplasia in situ
  • No other types of germ cell neoplasia should be present
Microscopic (histologic) description
Low power magnification
  • Homogenous lesions show diffuse sheets of cells without evident fibrous septa and lymphocytic infiltrates
  • Multinodular lesions show bands of loose fibrovascular tissue separating nodules (with or without lymphocytic infiltrates) and without remnants of testicular parenchyma between nodules
  • Spaces filled with clear to pink fluid can be present and they can have different shapes:
    • Irregular pseudocystic areas of variable size
    • Pseudofollicular (roundish edematous areas surrounded by neoplastic cells)
    • Microcystic spaces which separate ST into anastomosing islands with peripheral palisade of cells
  • Usually expansive neoplastic profile, although corded pattern of growth with intertubular neoplastic cells can be present at the periphery of the lesion
  • Neoplastic nodules can be boarded by a rim of fibrin (Am J Surg Pathol 2019;43:1)
  • Usually confined to testicular parenchyma, although it may involve the rete testis, the epididymis or the tunica albuginea
  • Nonneoplastic testicular parenchyma can have a mild degree of atrophy

High power magnification
  • Landmark feature - tripartite cytology with cells of 3 different sizes:
    • Small cells (6 - 8 microns)
      • Round dark nuclei
      • Narrow rim of eosinophilic cytoplasm
      • They resemble lymphocytes
    • Intermediate size cells (15 - 20 microns)
      • Round nuclei with granular to filamentous chromatin (so called spireme-like chromatin, after the thread-like chromatin at the beginning of prophase)
      • Usually no evident nucleoli, although they might be present
      • Dense eosinophilic to amphophilic cytoplasm with no glycogen
      • Poorly defined cytoplasmatic membranes
      • Clearer cells with distinct membranes can be found close to edematous areas
    • Giant cells (50 - 150 microns)
      • 1 or more nuclei with the same features of medium cells
      • Eosinophilic cytoplasm with the same features of medium cells
  • Size distribution can vary among different cases (usually medium cells > small cells > giant cells)
  • Mitoses (even atypical) and apoptotic bodies can be abundant
  • Stroma is scant, with only small strips of connective tissue
  • Lymphocytic infiltrates are scant to absent, both among neoplastic cells and within connective tissue strips
  • Granulomas are usually not present and only rarely reported
  • Intratubular extension of ST can be identified around the main lesion
  • Germ cell neoplasia in situ is not present
  • Intravascular invasion can be present
  • Not associated with other forms of germ cell tumor
  • Anaplastic spermatocytic tumor:
    • Not currently considered a WHO subtype of ST
    • No convincing clinical evidence of aggressive behavior
    • Characterized by areas of relatively monomorphic intermediate sized cells with prominent nucleoli
    • Areas of conventional ST can be found elsewhere in the tumor

Spermatocytic tumor with sarcomatous differentiation
  • Considered a WHO subtype of ST
  • Rare occurrence with poor prognosis
  • Sarcomatous component usually features a high grade spindle cell morphology
  • Other possible differentiations are rhabdomyosarcoma and chondrosarcoma
  • Sarcoma cells can interdigitate with ST cells or grow juxtaposed to ST
  • Sarcoma and ST can represent variable percentages of the lesion
  • Cases with only a minor ST residual component can be difficult to recognize
Microscopic (histologic) images

Contributed by Maurizio Colecchia, M.D., Case #448 and AFIP images
Lobulated architecture

Lobulated architecture

Tripartite cytology

Tripartite cytology

Prevalent intermediate sized cells

Prevalent intermediate sized cells

Mitoses and apoptotic bodies

Mitoses and apoptotic bodies

Short fibrous bands

Short fibrous bands

Spireme-like chromatin

Spireme-like chromatin


Edema and microcysts

Edema and microcysts

Edema and nests

Edema and nests

Intratubular growth

OCT3/4

OCT3/4

CD45

CD45


Diffuse growth pattern

Focal pseudoglandular pattern

Cystic pattern

Polymorphic cell population

Small nests, trabeculae, clusters and single cells



Filamentous chromatin

Normal spermatogenesis

Unusual features

Desmin+

Tumor nests and
cords mixed with
lymphocytes

Prominent
granulomatous
reaction and
lymphoid infiltrate

Intratubular component

Virtual slides

Images hosted on other servers:

Spermatocytic tumor

Cytology description
  • High cellularity
  • Predominantly single cells
  • Tripartite cytology with 3 different sizes of cells (small, intermediate and large), with a preponderance of intermediate sized cells
  • Absence of tigroid background (foamy material attributed to the fragile glycogen rich cytoplasm of seminomas)
  • No lymphocytes (Diagn Cytopathol 1999;20:233)
Positive stains
  • CD117
  • SALL4
  • DMRT1
  • NUT (specificity 100%, sensitivity 41% when positivity is present in ≥ 50% of neoplastic cells with equal intensity to spermatogonia internal control) (Histopathology 2014;65:35)
  • OCT2
  • SSX2
Negative stains
Electron microscopy description
  • Intercellular bridges between tumor cells identical to those between spermatocytes and between spermatids (Cancer 1969;24:103)
  • Syncytial formation
  • Scanty or absent cytoplasmatic glycogen and lipids
Electron microscopy images

AFIP images

Intercellular bridge

Nest of spermatocytic seminoma

Molecular / cytogenetics description
  • Aneuploid
  • Classically without 12p abnormalities
  • Presence of 12p amplification (hybrid genetics) might be linked to the development of metastases or anaplastic morphology (Pathology 2018;50:562, Hum Pathol 2022;124:85)
  • Amplification of chromosome 9p is the most frequent genetic abnormality, with a consequent amplification of the DMRT1 gene locus (Cancer Res 2006;66:290, Hum Pathol 2022;124:85)
  • Other gains include chromosome 1 and 20
  • Partial loss of chromosome 22 can be present
  • Mutations are infrequent
  • Activating mutations of HRAS and FGFR3 can be present, especially in elderly patients (PLoS One 2017;12:e0178169)
  • Epigenetics: loss of biparental imprinting with re-establishment of paternal imprinting
Sample pathology report
  • Testicle, partial / radical orchiectomy:
    • Spermatocytic tumor
    • Immunohistochemistry (supporting the above diagnosis): neoplastic cells are CD117+, SALL4+, OCT3/4-
    • Infiltration of rete testis present / absent; infiltration of epididymis present / absent; infiltration of hilar fat present / absent; infiltration of spermatic cord present / absent; infiltration of rete testis present / absent; infiltration of tunica vaginalis / scrotal wall testis present / absent
    • Lymphovascular invasion present / absent
    • Surgical margin involved / not involved by neoplasia
Differential diagnosis
Board review style question #1
Which of the following answers is correct about spermatocytic tumor?

  1. It can be either pure or mixed with postpubertal teratoma
  2. It does not derive from germ cell neoplasia in situ
  3. It often presents as an extragonadal neoplasia
  4. The most frequent location is the mediastinum
Board review style answer #1
B. It does not derive from germ cell neoplasia in situ

Comment Here

Reference: Spermatocytic tumor
Board review style question #2

In the case of the above histologic image, which of the following immunophenotypes would confirm a spermatocytic tumor?

  1. CD117+, SALL4+, AFP+
  2. CD117+, SALL4-, CD30+
  3. CD117+, SALL4+, OCT3/4+
  4. CD117+, SALL4+, OCT3/4-
Board review style answer #2
D. CD117+, SALL4+, OCT3/4-

Comment Here

Reference: Spermatocytic tumor

Splenogonadal fusion
Definition / general
  • Also called ectopic scrotal spleen, testicular splenic fusion
  • Rare congenital condition of gonad fusing with ectopic splenic tissue; patients usually present before age 20, 50%+ are less than age 10
  • In males, involves left testis only; occurs in females also
  • Fusion may be continuous (attaching to spleen) or discontinuous (intrascrotal splenic nodules attached to testis, spermatic cord, epididymis, appendix of testis or appendix of epididymis)
    • Continuous form is associated with limb bud anomalies such as peromelia (severe congenital anomalies of extremities identical to thalidomide embryopathy) and micrognatia (small jaw)
    • Discontinuous type is rarely associated with cardiac defects
Case reports
Gross description
  • Splenic tissue well demarcated from gonad
Microscopic (histologic) description
  • Normal splenic parenchyma with variable fibrosis, thrombi, calcification, fatty degeneration, hemosiderin
Microscopic (histologic) images

Case #309
Differential diagnosis
  • Lymphoproliferative tumor
  • Sarcoma
  • Teratoma

Staging
Definition / general
  • All testicular postpubertal germ cell tumors and malignant sex cord stromal tumors are covered by this staging system
  • These topics are not covered: spermatocytic tumor, nonmalignant sex cord stromal tumor, prepubertal germ cell tumor, hematopoietic tumors and paratesticular tumors
Essential features
  • AJCC 7th edition staging was sunset on December 31, 2017; as of January 1, 2018, use of the 8th edition is mandatory
Primary tumor (pT)
  • pTX: cannot be assessed
  • pT0: no evidence of primary tumor
  • pTis: germ cell neoplasia in situ
  • pT1: nonpure seminoma confined to testis / tunica albuginea / rete and no lymphovascular invasion (LVI)
  • pT1a: seminoma < 3 cm confined to testis / tunica albuginea / rete and no LVI
  • pT1b: seminoma ≥ 3 cm confined to testis / tunica albuginea / rete and no LVI
  • pT2: lymphovascular, hilar fat, epididymal or tunica vaginalis invasion
  • pT3: direct spermatic cord soft tissue invasion
  • pT4: direct scrotum invasion

Notes:
  • Size
    • Largest aggregate tumor size should be obtained, encompassing all adjacent nodules, even with different gross appearances rather than regarding nodules as separate smaller tumors
    • Seminoma is subclassified as pT1a / b based on a 3 cm threshold but nonpure seminoma is not
  • LVI
    • Freshly cut or minimally fixed testicular germ cell tumors have frequent tissue displacement artifact ("butter"), which confounds the diagnosis of true LVI (classified as pT2) (Am J Clin Pathol 2016;145:341)
    • Care must be taken when grossing to decrease the amount of tissue displacement artifact and overnight fixation of a bivalved specimen may be helpful
    • Histologic features supporting true LVI are tumor emboli that are cohesive, have smooth contours and adhere to the vessel wall (Am J Clin Pathol 2016;145:341)
    • LVI of the spermatic cord is pT2 not pT3
  • pT2
    • Involvement of the tunica albuginea does not impact pT classification
    • Tumor involvement of the tunica vaginalis is classified as pT2 and is rare (4/170, 2% seminoma; 0/148, 0% nonseminoma) (Histopathology 2018;73:741, Mod Pathol 2013;26:579)
    • Epididymal invasion is uncommon (6% seminoma, 8% nonseminoma) and is classified as pT2; however, it is not associated with a higher clinical stage (Histopathology 2018;73:741, Mod Pathol 2013;26:579)
    • Tumor within the soft tissue below the level of the epididymal head is considered hilar fat invasion and is classified as pT2
  • pT3
    • Tumor within the soft tissue beyond the angle between the epididymis and spermatic cord is classified as pT3
Regional lymph nodes (pN)
  • pNX: cannot be assessed
  • pN0: no regional lymph node metastasis
  • pN1: 1 - 5 involved nodes with node size ≤ 2 cm
  • pN2: > 5 involved nodes or extranodal extension or involved nodes > 2 cm and ≤ 5 cm
  • pN3: lymph node mass > 5 cm

Notes:
  • Regional lymph nodes = interaortocaval, para / periaortic, paracaval, preaortic, precaval, retroaortic and retrocaval nodes
  • The size used to classify untreated lymph nodes is the overall lymph node size rather than the size of the metastatic deposit
  • Per the AJCC chapter 1 regarding classification post therapy, "necrotic cells currently play no role in assigning the ypT and ypN"
  • Therefore, size of the metastatic deposit rather than overall lymph node is recommended for use in classifying lymph nodes after therapy
  • Additionally, most literature describes residual viable tumor size in lymph nodes post treatment rather than overall lymph node size (World J Urol 2021;39:1969, World J Urol 2021;39:839)
Distant metastasis (pM)
  • pM1a: nonregional lymph node (e.g., iliac, inguinal, pelvic NOS) or lung metastasis
  • pM1b: distant visceral metastasis

Notes:
Serum markers (S)
  • SX: serum tumor markers not available or not performed
  • S0: markers within normal limits
  • S1: LDH < 1.5 x upper limit of normal, hCG < 5,000 mIU/mL and AFP < 1000 ng/mL
  • S2: LDH 1.5 - 10 x upper limit of normal or hCG 5,000 - 50,000 mIU/mL or AFP 1,000 - 10,000 ng/mL
  • S3: LDH > 10 x upper limit of normal or hCG > 50,000 mIU/mL or AFP > 10,000 ng/mL
AJCC prognostic stage groups
  • Stage group 0:pTis N0 M0 S0
  • Stage group I:pT1 - 4 N0 M0 SX
  • Stage group IA:pT1 N0 M0 S0
  • Stage group IB:pT2 - 4 N0 M0 S0
  • Stage group IS:pT1 - 4,X N0 M0 S1 - 3
  • Stage group II:pT1 - 4,X N1 - 3 M0 SX
  • Stage group IIA:pT1 - 4,X N1 M0 S0 - 1
  • Stage group IIB:pT1 - 4,X N2 M0 S0 - 1
  • Stage group IIC:pT1 - 4,X N3 M0 S0 - 1
  • Stage group III:pT1 - 4,X N0 - 3 M1 SX
  • Stage group IIIA:pT1 - 4,X N0 - 3 M1a S0 - 1
  • Stage group IIIB:pT1 - 4,X N1 - 3M0 S2;pT1 - 4,XN0 - 3M1aS2
  • Stage group IIIC:pT1 - 4,X N1 - 3M0S3; pT1 - 4,XN0 - 3 M1aS3; pT1 - 4,X N0 - 3M1bS0 - 3


Notes:
  • While discontinuous spermatic cord invasion is classified as pM1 per the AJCC thus rendering the patient stage III, the National Comprehensive Cancer Network (NCCN) guidelines do not agree with the designation of discontinuous spermatic cord invasion as pM1 but instead as pT3 (J Natl Compr Canc Netw 2019;17:1529)
Registry data collection variables
  • Serum tumor markers (AFP, hCG, LDH)
Histologic grade (G)
  • Germ cell tumors are not graded
Histopathologic type
  • Seminoma
  • Nonseminoma
    • Embryonal carcinoma
    • Yolk sac tumor
    • Teratoma
    • Choriocarcinoma
  • Mixed germ cell tumor
  • Malignant Leydig cell tumor
  • Malignant Sertoli cell tumor
  • Malignant sex cord stromal tumors, other types

Notes:
  • 60% of testicular germ cell tumors are seminoma
  • Within a mixed germ cell tumor, frequency of tumor type is: embryonal carcinoma > teratoma, yolk sac tumor, seminoma > choriocarcinoma > other rare types (Med Oncol 2018;35:21)
Gross images

Contributed by Debra L. Zynger, M.D.

Complete regression (pT0)

Germ cell neoplasia in situ only (pTis)

Partial regression (pT1)

Teratoma (pT1)

Teratoma and EC (pT1)

Seminoma (pT1a)


Seminoma (pT1b)

Mixed germ cell tumor (pT2)

Seminoma (pT2)

No residual testis tumor (ypT0)


No residual node tumor (ypN0)

Lymph node with residual tumor (ypN1 - 3)

Discontinuous spermatic cord (pM1)

Microscopic (histologic) images

Contributed by Debra L. Zynger, M.D.

Lymphovascular invasion (pT2)

Lymphovascular invasion (pT2)

Tunica vaginalis invasion (pT2)


Epididymal invasion (pT2)

Hilar soft tissue invasion (pT2)

Lymph node metastasis (pN1)

Post treatment node (ypN0)

Post treatment node (ypN1)


Discontinuous spermatic cord invasion (pM1)

Board review style question #1

In a postpubertal testicular germ cell tumor, lymphovascular (shown above), hilar fat, epididymal and tunica vaginalis invasion are all a part of which pT category?

  1. pT1
  2. pT1a
  3. pT1b
  4. pT2
  5. pT3
Board review style answer #1
D. pT2. Lymphovascular, hilar fat, epididymal and tunica vaginalis invasion are all a part of the pT2 category for testicular germ cell tumors. Lymphovascular invasion by embryonal carcinoma is shown in the image.

Comment Here

Reference: Testis & epididymis - Staging
Board review style question #2

The postpubertal teratoma shown in the gross photo is 1.7 cm and is confined to the testis and did not have lymphovascular invasion. Which is the correct pT category?

  1. pT1
  2. pT1a
  3. pT1b
  4. pT2
  5. pT3
Board review style answer #2
A. pT1. Postpubertal germ cell tumors that are confined to the testicular parenchyma and lack lymphovascular invasion are pT1, pT1a or pT1b. Pure seminoma is stratified as pT1a and pT1b based on a 3 cm threshold. Nonpure seminomas are not stratified and therefore this tumor is pT1.

Comment Here

Reference: Testis & epididymis - Staging

Teratoma
Definition / general
  • Tumor originating from germ cells with more than one embryonic germ layer
Terminology
Epidemiology
Pathophysiology
  • Postpubertal type associated with germ cell neoplasia in situ (GCNIS) and chromosome 12p amplification (Med Surg Urol 2014;3:1)
    • Thought to arise from GCNIS
  • Prepubertal is not associated with GCNIS or chromosome 12p amplification (Med Surg Urol 2014;3:1)
    • Prepubertal type significantly less likely to metastasize or recur
Etiology
Clinical features
Diagnosis
  • Ultrasound is used to evaluate a testicle mass
  • Avoid biopsies to prevent cancer seeding
  • Radical orchiectomy is initial procedure to diagnose a testicular mass in an adult
  • Increasing rate of antenatal diagnoses (J Pediatr Surg 2006;41:1513)
  • May use intraoperative frozen sections to guide therapy in pediatric populations
Laboratory
Radiology description
  • Ultrasound is used to help exclude benign pathology
  • Imaging may demonstrate calcifications or predominantly cystic lesions
  • Computed tomography of the chest and abdomen used for staging purposes
Radiology images

Images hosted on other servers:

Testicular teratoma diagnosed in utero

Demonstrating vascularity of testicular teratoma with calcifications

Prognostic factors
Case reports
Treatment
  • Radical orchiectomy is first line therapy for testicular mass in an adult
  • Increasing rate of testis sparing surgery for prepubertal tumors (Rev Urol 2004;6:11)
  • Retroperitoneal lymph node dissection and platinum based chemotherapy are adjunctive therapies for testicular germ cell tumor
  • Treatment guidelines are based on postorchiectomy TNM stage (NCCN guidelines: Testicular Cancer)
  • Consider sperm preservation options
Gross description
  • Lobulated, with cysts of mucinous, gelatinous or serous material
Gross images

Contributed by Debra L. Zynger, M.D.

Teratoma (100%)

Mixed GCT

Microscopic (histologic) description
Microscopic (histologic) images

Contributed by Debra L. Zynger, M.D.

Cartilage surrounded by mitotically active spindle cells

Gastrointestinal epithelium

Squamous epithelium

Cellular mesenchyme

Loose mesenchyme

Neuroectoderm

Molecular / cytogenetics description
Sample pathology report
  • Right testicle, radical orchiectomy:
    • Mixed germ cell tumor, teratoma (55%), seminoma (20%), embryonal carcinoma (10%), yolk sac tumor (10%) and choriocarcinoma (5%) types (see synoptic report)
Differential diagnosis
  • Dermoid cyst:
    • Squamous epithelium, hair follicles / sebaceous glands
    • May have adjacent lipogranulomas but no other embryonic germ cell types
    • Lacks GCNIS and cytologic atypia (Arch Pathol Lab Med 2012;136:435)
  • Epidermoid cyst:
    • Nonneoplastic squamous epithelium producing keratin with lamellar targetoid appearance
    • No other embryonic germ cell types
    • Lacks GCNIS and cytologic atypia (Arch Pathol Lab Med 2012;136:435)
  • Sarcoma:
    • Can mimic teratoma with sarcoma as somatic type malignancy
    • Lacks GCNIS and other germ cell tumor components
  • Yolk sac tumor:
    • Microcystic, glandular and papillary growth patterns can mimic teratoma
    • Positive for AFP and GPC3
Board review style question #1

A 27 year old man presents with a painless enlarging testicular mass. He undergoes appropriate workup and orchiectomy confirms diagnosis of a pure postpubertal teratoma. Which of the following is likely to be true?

  1. AFP is likely elevated
  2. Cytogenetic analysis will reveal chromosomal abnormalities
  3. Few mitotic figures will be identified microscopically
  4. Hair follicles are likely to be seen microscopically
  5. He has a poor prognosis
Board review style answer #1
B. Cytogenetic analysis will reveal chromosomal abnormalities

Comment Here

Reference: Teratoma
Board review style question #2
A 37 year old woman gives birth to a male baby who is diagnosed with an undescended testicle. Further workup reveals a large intra abdominal mass, which is resected. Pathology is consistent with a pure teratoma. Which of the following is most likely present on microscopic examination?

  1. Adjacent areas of germ cell neoplasia in situ
  2. Areas with organoid morphology
  3. Many mitotic figures and cytologic atypia
  4. Sarcomatous elements
  5. Significant areas of testicular tubular atrophy
Board review style answer #2
B. Areas with organoid morphology

Comment Here

Reference: Teratoma

Teratoma with somatic type malignancy
Definition / general
  • Testicular germ cell tumor with an area of pure somatic type malignancy that arises from germ cell tumor progenitor cells and occupies ≥ 0.5 cm (approximately ≥ 1 microscopic field using 4x objective)
Essential features
  • Identified within primary testicular tumor or at a metastatic site
  • Rhabdomyosarcoma, adenocarcinoma, embryonic type neuroectodermal tumor and sarcoma, NOS are the most frequent histologies (Histopathology 2022;81:84, Am J Surg Pathol 2022;46:11)
  • Varying morphologies, including neuroglial, neuroendocrine carcinoma and neuroblastoma, may make diagnosis difficult (Histopathology 2022;81:84)
  • Must be distinguished from sarcomatoid or glandular yolk sac tumor, which represent up to 60% of postchemotherapy sarcomatous tumors (Am J Surg Pathol 2015;39:251)
Terminology
  • Current terminology: teratoma with somatic type malignancy
  • Previous terminology: teratoma with malignant transformation, teratoma with somatic malignancy
  • Currently recommended that embryonic type neuroectodermal tumor should replace primitive neuroectodermal tumor (PNET) when referring to a teratoma with this type of somatic malignancy because although it resembles CNS PNET, it lacks EWSR1 rearrangement (Am J Surg Pathol 2021;45:1299)
ICD coding
  • ICD-O: 9084/3 - teratoma with malignant transformation
  • ICD-11: 2C80.Y & XH33E8 - other specified malignant neoplasms of testis & teratoma with malignant transformation
Epidemiology
Sites
  • Testis
  • Metastatic locations (e.g., retroperitoneum, lungs)
Pathophysiology
Clinical features
Diagnosis
  • Found as metastatic disease, within the testis or both (J Urol 1998;159:133)
  • Histologic features and immunohistochemistry determine type of somatic malignancy
Radiology description
  • Imaging may suggest specific somatic malignancy in germ cell tumors (Eur J Radiol 2009;69:230)
    • Ossified lymph nodes and soft tissue reported in osteosarcoma
    • Peritoneal carcinoma seen in adenocarcinoma
    • Septal thickening and opacifications reported in a case of bronchoalveolar carcinoma
Prognostic factors
  • If confined to the testis without synchronous metastasis, somatic malignancy does not appear to confer a worse prognosis Am J Surg Pathol 2022;46:11)
  • Presence in a metastatic site is associated with poor survival (median survival: 44 months) Am J Surg Pathol 2022;46:11)
  • Median time from initial diagnosis of testicular germ cell tumor and diagnosis of somatic malignancy at metastatic site: 70 months (range: 0 - 384 months) Am J Surg Pathol 2022;46:11)
Case reports
Treatment
  • Surgical resection is the mainstay of treatment
  • Chemotherapy is based on histologic subtype of malignant transformation (J Clin Oncol 2003;21:4285)
  • Radical surgery is often utilized for advanced disease (J Urol 1998;159:133)
Gross description
Gross images

Contributed by Christopher Dall, M.D. and Debra L. Zynger, M.D.
Missing Image

Retroperitoneal sarcoma, NOS

Missing Image

Lung rhabdo-
myosarcoma

Missing Image

Iliac leiomyosarcoma

Missing Image

Embryonic type neuroectodermal tumor

Microscopic (histologic) description
  • Expansile and infiltrative pure area of atypical mesenchymal or epithelial cells that occupies ≥ 0.5 cm
  • 4 most common histologies (Am J Surg Pathol 2022;46:11, Histopathology 2022;81:84):
    • Rhabdomyosarcoma
    • Adenocarcinoma
    • Embryonic type neuroectodermal tumor
    • Sarcoma, NOS
  • Embryonic type neuroectodermal tumor appears as immature neuroectodermal elements
  • Neuroglial tumors have CNS-like morphology similar to astrocytoma, glioma, glioblastoma and gliosarcoma (Am J Surg Pathol 2019;43:422)
  • Nephroblastoma-like histology has been described
Microscopic (histologic) images

Contributed by Christopher Dall, M.D. and Debra L. Zynger, M.D.
Missing Image Missing Image

Metastatic rhabdomyosarcoma

Missing Image Missing Image

Metastatic chondrosarcoma


Missing Image Missing Image

Metastatic sarcoma, NOS

Missing Image Missing Image

Embryonic type neuroectodermal tumor within testis

Positive stains
Negative stains
Molecular / cytogenetics description
Sample pathology report
  • Right testis, radial orchiectomy:
    • Mixed germ cell tumor, embryonal carcinoma (75%), yolk sac tumor (10%), teratoma (10%), teratoma with somatic malignancy composed of embryonic type neuroectodermal tumor (5%) (see synoptic report)

  • Left lung, wedge resection:
    • Metastatic rhabdomyosarcoma, 3.2 cm (see comment)
    • Comment: The patient has a history of a mixed testicular germ cell tumor. The current tumor is consistent with testicular origin (teratoma with somatic malignancy).
Differential diagnosis
Board review style question #1

    Metastatic teratoma with somatic type malignancy in a man with a history of testicular germ cell tumor is shown in the image above. What is the histologic type of somatic malignancy?

  1. Adenocarcinoma
  2. Embryonic type neuroectodermal tumor
  3. Neuroglial tumor
  4. Rhabdomyosarcoma
  5. Squamous cell carcinoma
Board review style answer #1
Board review style question #2

    A 43 year old man with a history of testicular mixed germ cell tumor treated with orchiectomy and chemotherapy is found to have an undifferentiated mesenchymal tumor. Which finding would best support this tumor being teratoma with somatic type malignancy?

  1. Isochromosome 12p
  2. MYC amplification
  3. TERT promoter mutation
  4. Expression of ERG
Board review style answer #2
A. Isochromosome 12p

Comment Here

Reference: Teratoma with somatic type malignancy

Teratoma-dermoid cyst
Definition / general
  • Rare form of benign testicular teratoma, characterized by pilosebaceous units arranged around predominantly unicystic lesion, often containing hair
  • Noncutaneous elements may be present but lack cytologic atypia seen in usual mature teratoma (Am J Surg Pathol 2001;25:788)
Microscopic (histologic) description
  • Organoid arrangement of pilosebaceous units around epidermal epithelium
  • Lipogranulatomous reaction in testis parenchyma due to leakage of sebaceous secretion
  • Other tissue types sometimes present
  • No intratubular germ cell neoplasia (IGCNU), no immature elements

Teratoma-neuroendocrine tumor
Definition / general
  • Well differentiated neuroendocrine tumor (NET) is a low grade epithelial neoplasm with neuroendocrine differentiation
Essential features
  • Primary testicular NETs can be pure (75%) or associated with teratoma (25%)
  • Tumors show an insular or trabecular growth pattern with salt and pepper nuclear chromatin pattern
  • Most are not associated with isochromosome 12p or germ cell neoplasia in situ (GCNIS)
Terminology
  • Also known as pure NET, testicular NET, prepubertal type or postpubertal type or monodermal teratoma
  • Terminology not recommended: carcinoid, atypical carcinoid, neuroendocrine carcinoma
ICD coding
  • ICD-O: 8240/3 - well differentiated neuroendocrine tumor (monodermal teratoma)
  • ICD-11: 2F77 & XH8DS0 - neoplasms of uncertain behavior of male genital organs & neuroendocrine tumor, NOS
Epidemiology
  • Extremely rare, accounting for < 1% of all testicular neoplasms (Am J Surg Pathol 2010;34:519)
  • More common in ovaries than testes (15:1)
  • Mean age at presentation: 46 years (range: 10 - 83 years)
  • Most cases are reported in Europe and the U.S. with fewer cases from Asia and Africa
  • Metastatic neuroendocrine tumor from other sites (e.g., lung or gastrointestinal tract) to the testis has been reported
Sites
  • NET in the genitourinary tract is rare and can occur in the kidney, bladder, prostate, testicle or urethra
Pathophysiology
Etiology
  • Unknown
Clinical features
  • Most commonly presents as a testicular mass or swelling, which may or may not be associated with testicular pain
  • Bilateral involvement is uncommon and association with cryptorchidism is rare
  • 10% of cases occur in association with hydrocele (Cancer 1993;72:1726)
  • Clinical carcinoid syndrome (hot flashes, diarrhea and palpitations) has been reported in 7 - 12% of cases (Cancer 1993;72:1726, Am J Surg Pathol 2010;34:519)
  • Metastases occur via hematogenous spread to lungs, liver, bones, soft tissue, skin, heart as well as contralateral testis; however, lymphatic spread is also seen
Diagnosis
  • Diagnosis of testicular NET is made based on histologic examination of the surgical resection specimen
Prognostic factors
  • Primary testicular NET associated with testicular teratoma seems to have a better prognosis than pure NET
  • Metastatic disease is associated with atypical features, including larger tumor size (> 7 cm), increased mitotic activity and carcinoid syndrome (Am J Surg Pathol 2010;34:519, Cancer 1993;72:1726)
Case reports
Treatment
  • Orchiectomy
  • In a metastatic setting, retroperitoneal dissection and receptor targeted radiotherapy may be used; however, chemotherapy and radiotherapy are reported to provide minimal benefits
Gross description
  • Well circumscribed, solid, yellow-tan to brown and ranging in size from 0.5 cm up to 11 cm (Cancer 1993;72:1726)
  • Cystic changes or calcifications can be seen in association with teratomatous component
  • Extratesticular growth with involvement of spermatic cord occurs uncommonly
Gross images

AFIP images
Testicular NET

Solid tan mass

Microscopic (histologic) description
Microscopic (histologic) images

Contributed by Sounak Gupta, M.B.B.S, Ph.D. and Rafael E. Jimenez, M.D.
Epidermoid cyst and NET Epidermoid cyst and NET

Epidermoid cyst and NET

Retraction artifact

Retraction artifact

Multiple solid nests

Multiple solid nests

Insular growth pattern

Insular growth pattern

Nests of tumor cells

Nests of tumor cells

Cytology description
  • Isolated or sheets of neoplastic cells with granular cytoplasm, uniform round nuclei and uniformly distributed fine nuclear chromatin
Positive stains
Electron microscopy description
  • Pleomorphic to more regular round to oval neurosecretory granules
Molecular / cytogenetics description
  • DNA ploidy studies show a near diploid profile (Cancer 1993;72:1726)
  • As most cases are not associated with GCNIS, they do not show isochromosome 12p or numerical aberrations in the X chromosome, which are commonly seen in GCNIS derived germ cell tumors
    • In rare cases, assessment of isochromosome 12p may be indicated if the postpubertal type is suspected
Videos

Testicular NET

Sample pathology report
  • Testis and spermatic cord, left, radical orchiectomy:
    • Testicular well differentiated neuroendocrine tumor, prepubertal type (monodermal teratoma) and epidermoid cyst (see comment)
    • Comment: No germ cell neoplasia in situ (GCNIS) is identified in the background seminiferous tubules.
Differential diagnosis
Board review style question #1

A 35 year old man underwent a left radical orchiectomy that demonstrated a 4.0 cm cystic mass (see image). No serological markers (alpha fetoprotein [AFP], beta human chorionic gonadotropin [beta hCG] or lactate dehydrogenase [LDH]) were elevated. Regarding this entity, which of the following statements is true?

  1. CDX2 is usually positive in testicular neuroendocrine tumor
  2. Cystic changes can be seen with teratomatous components
  3. Cytokeratins are usually negative in testicular neuroendocrine tumor
  4. Glandular growth pattern is not observed
  5. These tumors are mostly found in association with germ cell neoplasia in situ (GCNIS) / isochromosome 12p
Board review style answer #1
B. Cystic changes can be seen with teratomatous components. Primary testicular neuroendocrine tumor (NET) can occur as pure NET (75%) or in association with testicular teratoma (25%). Cystic changes in these lesions can be seen with teratomatous components. Answer D is incorrect because these tumors mostly exhibit an insular or trabecular growth pattern that can be intermixed with glandular / acinar structures with luminal mucin. Answer E is incorrect because most of these cases are not associated with GCNIS and do not show isochromosome 12p or numerical aberrations in the X chromosome, which are commonly seen in GCNIS derived germ cell tumors. Answers C and A are incorrect because testicular NET expresses cytokeratins and is negative for CDX2.

Comment Here

Reference: Teratoma - neuroendocrine tumor

Testicular adrenal rest tumor / tumor of adrenogenital syndrome
Definition / general
  • Testicular tumor associated with congenital adrenal hyperplasia and chararacterised by benign behavior and typical hormone sensitivity
Terminology
  • Testicular tumors of Adrenogenital syndrome (TTAGS)
  • Also know as Testicular adrenal rest tumors (TARTs)
  • Biologically, the lesions represent hyperplasia of aberrant rests rather than a true neoplastic tumor
Epidemiology
Sites
  • Normaly descended testis and adjacent tissues
Etiology and Pathophysiology:
  • Development and pathogenesis
  • Effects of TTAGS:
    • Mechanical effects: size and duration dependent blockage of rete testis with resulting atrophy of seminiferous tubules
    • Paracrine effects: steroids produced by tumor cells may be toxic to Leydig cells or germ cells (Hum Reprod 2001;16:263)
    • The irreversible end stage of longstanding TART is tubular hyalinization with obstruction of the lumen and complete loss of germ cells and Sertoli cells; differs from ischemic hyalinization by relative preservation of Interstitial Leydig cells
Clinical features
  • Most tumors are detected on ultrasound screening of patients with Congenital adrenal hyperplasia
  • Symptoms of congenital adrenal hyperplasia:
    • Severe forms: critical adrenal insufficiency with salt losing form present in shock very early in childhood
    • Milder forms: present with varying degrees of virilization and sexual precocity in a young child, adolescent or adult
  • Most common presentation of TTAGS is bilateral presence of palpable nodules in both testes with testicular enlargement
  • Though varying degree of testicular atrophy is common, presentation with infertility is rare (Actas Urol Esp 2003;27:234)
Diagnosis
  • Diagnosis is suspected on clinical features and ultrasound examination of scrotum (Radiology 1996;198:99) and confirmed with a biopsy / surgical excision
Laboratory
  • Increased blood levels of 11-β-OH steroids in effluent testicular blood
  • Increased 11-β-hydroxylase activity in tumor tissue
  • Low α-fetoprotein, LDH and β-HCG
Radiology description
  • Tumors are uniformly hypoechoic with well defined margins
  • Multifocally and bilaterality is common (~75%)
  • Mean diameter ~16 mm (range, 2 - 28 mm, Radiology 1992;183:425)
Prognostic factors
  • Tumors are uniformly benign in behavior and many respond to ACTH suppression by glucocorticoid therapy
  • No reports of malignant transformation or recurrence after complete excision
Case reports
Treatment
  • Early tumors are responsive to glucocorticoid therapy, which suppresses ACTH levels
  • Surgery for tumors nonresponsive to glucocorticoid therapy, large tumors or if differential diagnosis with other tumors exists
  • Testicular sparing surgery is preferred to preserve fertility, as tumor is benign
Staging / staging classifications
  • Claahsen-van der Grinten et al (Best Pract Res Clin Endocrinol Metab 2009;23:209) proposed a staging system for TART development and progression, with an intent to guide treatment
    • Stage 1: presence of adrenal rest cells within the rete testis, not detectable by scrotal ultrasound
      • In healthy boys, probably regress in utero or in first year of life
    • Stage 2: adrenal rest cells may proliferate in the presence of increased concentrations of ACTH (and possibly also of Angiotensin II)
      • The adrenal rest cells may become visible by ultrasound as one or more small hypoechogenic lesions
      • Age of onset of cell growth may depend on the cumulative exposure to ACTH (and angiotensin II) concentrations over time and the number of ACTH (and angiotensin II) receptors on the adrenal rest cells
      • Complete regression of tumors with ACTH suppression by glucocorticoid therapy is possible
    • Stage 3: further growth of the adrenal rest cells will compress the rete testis
      • In pubertal or postpubertal CAH patients, oligo or azoospermia may already be found due to obstruction of the seminiferous tubules
      • Signs of gonadal dysfunction: decreased inhibin B and increased follicle stimulating hormone (FSH) and LH levels may also be present
      • Tumors are responsive to ACTH suppression, however the tumors relapse after discontinuation of ACTH suppression
      • Hence, glucocorticoid therapy is only a temporary solution
    • Stage 4: further hypertrophy and hyperplasia of the adrenal rest cells with progressive obstruction of the rete testis, leads to induction of fibrosis within the tumor and focal lymphocytic infiltration
      • Several small tumors within the rete testis will conflate, forming a single lobulated tumor, separated from the residual testicular tissue by fibrous strands
      • Differentiation of adrenal cells with loss of ACTH and angiotensin II receptors
      • Loss of response to ACTH suppression by glucocorticoids
      • Surgical excision of TTAGS could prevent further decline of testicular function
    • Stage 5: chronic obstruction subsequently will lead to destruction of the surrounding testicular parenchyma with irreversible damage of the testis
      • Only indication for surgery is the relief of pain and discomfort caused by TTAGS
  • This staging system has not been validated
  • Early treatment of CAH with adequate suppression of ACTH levels by glucocorticoid treatment from childhood, may prevent development of TTAGS
  • However, the benefit needs to be balanced against the effect of growth suppression by excess glucocorticoid therapy
Clinical images

Images hosted on other servers:

Dexamethasone treatment

Large TTAGS

Gross description
  • Usually bilateral, commonly located in hilar region of testis
  • Well circumscribed yellow to tan nodules separated by dense fibrous bands
Gross images

Images hosted on other servers:

Well circumscribed,
noncapsulated, solid
and lobulated brown lesion

Microscopic (histologic) description
  • Resemble adrenocortical cells
  • Sheets, cords or lobules of large, polygonal cells with abundant, finely granular eosinophilic cytoplasm, often with lipochrome pigment
  • Round, giant and hyperchromatic nuclei; nuclear pleomorphism can be prominent with presence of varying sized intranuclear inclusions
  • Thin fibrovascular strands run through the tumor tissue but zonation is absent
  • Mitoses are usually absent
  • Reinke crystals are absent and help to differentiate from Leydig cell tumors to an extent
  • Adjacent testicular tissue may be normal or may show evidence of atrophy in the form of reduced / absent spermatogenesis with reduced diameter of the seminiferous tubules, thickened basal membranes and deposition of fibrohyaline tissue
  • The changes are more marked with tumors of long duration
Microscopic (histologic) images

Images hosted on other servers:

H&E

H&E, IHC, CD56

H&E

Cytology description
  • Cellular smears with large polygonal to round cells with foamy cytoplasm, round centrally placed nuclei with prominent nucleoli
  • Abundant lipochrome pigment is characteristic
  • Reinke crystals are absent
  • No cytological feature to reliably distinguish from Leydig cell tumor in the absence of Reinke crystals
Immunohistochemistry & special stains
Electron microscopy description
  • Abundant smooth endoplasmic reticulum, a moderate number of mitochondria with tubulovesicular cristae, lipid droplets and lipofuscin granules in the polygonal cells
  • No Reinke crystals
Differential diagnosis
  • Leydig cell tumor
    • Less commonly bilateral (3% compared to 75% in TTAGS, Pathologica 1994;86:557, Urol Clin North Am 2000;27:519)
    • Gynecomastia is more common (~30%), while it is rare in CAH (Urol Clin North Am 2000;27:519)
    • Associated with Peutz-Jeghers syndrome, not associated with adrenal hyperplasia
    • Nonresponsive to corticosteroid therapy
    • Presence of Reinke crystals is diagnostic but seen in just 40 of cases.
    • Malignancy related changes are seen in 10% of Leydig cell tumors while they never occur in TTAGS
    • Absence of adipocyte metaplasia and extensive fibrosis
    • Androgen receptor positive and weak reactivity / negative for CD56 and synaptophysin
  • Biochemical features are NOT useful to differentiate (Urol Clin North Am 2000;27:519)

Testicular regression syndrome / anorchia
Definition / general
  • Anorchism: absence of both testis
  • Monoorchidism: absence of one testis
  • Testicular regression syndrome: phenotypically normal male with vas deferens is present
Etiology
  • Causes: cryptorchidism, possibly testicular infarct, infection, trauma, torsion or prenatal hormone induced atrophy due to overproduction of androgens
Testicular regression syndrome
  • Either no gonadal or testicular formation or regression of testicular tissue with residual fibrovascular nodule (mean 1.1 cm), calcification and hemosiderin
  • Rudimentary epididymis and spermatic cord are present
  • External genitalia depends on chronology of gonadal injury
Case reports
  • 1 year old boy with undescended testis and firm nodule near external inguinal opening (Case of the Week #279)
Microscopic (histologic) description
  • Regressed testis indicated by fibrosis, hemosiderin, calcification or Leydig cells near epididymis or proximal vas deferens (Arch Pathol Lab Med 2000;124:694)
  • Presence of only fat and connective tissue does not rule out an intraabdominal testis
Microscopic (histologic) images

Torsion
Definition / general
Essential features
  • Vascular congestion, edema and interstitial bleeding
  • Hemorrhagic infarction of the testis
ICD coding
  • ICD-10: N44.00 - torsion of testis, unspecified
Epidemiology
Sites
  • Testis
Pathophysiology
  • Twisting of the testicle along the spermatic cord results in reduced venous outflow and vascular congestion
  • Prolonged twisting impairs arterial flow and ischemia
  • Prolonged ischemia can result in necrosis
  • Reference: Am J Emerg Med 2023;66:98
Etiology
  • Abnormal or absent fixation of testicle and tunica vaginalis
  • Tunica vaginalis contains the spermatic cord and normally adheres to the posterolateral part of the testicle
  • Absent or abnormal fixation can result in twisting of the testicle, compromising the blood flow
  • 2 classic anatomic forms
    • Intravaginal (more common)
      • Puberty and adulthood
      • Abnormalities in tunica vaginalis attachment
      • Bell clapper deformity: abnormally high reflection of tunica vaginalis, allowing the spermatic cord to twist inside the tunica
    • Extravaginal
      • Infancy and childhood
      • Tunica vaginalis is not adhered to the gubernaculum
      • Both the tunica vaginalis and spermatic cord are prone to torsion
  • References: Am J Emerg Med 2023;66:98, J Ultrasound Med 2022;41:271
Diagrams / tables

Images hosted on other servers:
Bell clapper deformity leading to intravaginal testicular torsion

Bell clapper deformity
leading to intravaginal
testicular torsion

Clinical features
  • Intravaginal
    • Sudden or insidious onset of pain
    • Nausea and vomiting
    • Followed by swelling of the ipsilateral scrotum
    • Most commonly, these symptoms occur several hours after minor trauma or physical activity
    • In some instances, history of previous pain
  • Extravaginal
    • May be entirely asymptomatic
    • Sole clinical manifestation may be unilateral mass in the inguinal region or in the superior aspect of the scrotum
  • Reference: Clin Radiol 1999;54:343
Diagnosis
  • Imperative to rule out testicular torsion in a patient presenting with acute scrotum
  • High index of suspicion on the part of the physician is needed
  • Scrotal Doppler ultrasonography is the imaging study of choice (Am Fam Physician 2013;88:835)
Radiology description
  • Scrotal Doppler ultrasonography
    • Flow in the symptomatic testis may be absent, reduced or have abnormally high resistance
    • In bell clapper deformity, testis can rotate
      • Doppler ultrasound flow abnormalities can differ depending on the degree of rotation, which ranges from 180° to 270°
    • Venous outflow occlusion precedes arterial inflow occlusion
      • Early or partial torsion can result in areas with high resistance flow rather than absent flow owing to compromised venous outflow
    • If spontaneous detorsion occurs, Doppler ultrasound flow within the involved testis may increase
  • Reference: Radiographics 2020;40:529
Radiology images

Contributed by Raman Danrad, M.D.
Ultrasound of bilateral testis

Ultrasound of bilateral testis

Ultrasound of right normal testis

Ultrasound of right normal testis

Ultrasound of left torsed testis

Ultrasound of left torsed testis

Case reports
Treatment
  • Prompt restoration of blood flow is critical in cases of testicular torsion
  • Typically, there is a 4 - 8 hour window before significant ischemic damage occurs
  • Viability of testicle is difficult to predict, hence emergent surgical management is indicated
  • Reported testicular salvage rates are 90 - 100% if surgical exploration is performed within 6 hours of symptom onset
  • Orchiectomy is performed if the affected testicle appears grossly necrotic or nonviable
  • If the affected testicle is deemed viable, orchiopexy with a permanent suture should be performed to permanently fix the testicle within the scrotum
  • Contralateral orchiopexy should be performed regardless of the viability of the affected testicle
  • Reference: Am Fam Physician 2013;88:835
Clinical images

Images hosted on other servers:
Twisting of spermatic cord

Twisting of spermatic cord

Gross description
  • Depends on the degree of torsion and the time lapse since onset of symptoms
  • Dark red testis with smooth external surface and solid consistency
Gross images

Images hosted on other servers:
Smooth external surface of torsed testis

Smooth external surface of torsed testis

Dark red hemorrhagic cut surface

Dark red hemorrhagic cut surface

Microscopic (histologic) description
  • Edema, congestion and interstitial hemorrhage with venous outflow obstruction
  • Early conservation of germ cells, which later disappear
  • Initial phase (< 6 hours): dilatation of seminiferous tubules with sloughing of germ cells and vacuolization of Sertoli cells
  • Followed by pronounced hemorrhage and sloughing of all germ cells
  • Necrosis of the seminiferous epithelium
  • Hemorrhagic infarction of testicle
  • References: Maclennan: Urologic Surgical Pathology, 4th Edition, 2019, Amin: Urological Pathology, 1st Edition, 2014
Microscopic (histologic) images

Contributed by Ritu Bhalla, M.D.
Congestion

Congestion

Hilar hemorrhage

Hilar hemorrhage

Sloughing of cells

Sloughing of cells

Interstitial hemorrhage

Interstitial hemorrhage


Residual germ cells

Residual germ cells

Infarcted tubules

Infarcted tubules

Necrotic parenchyma

Necrotic parenchyma

Sample pathology report
  • Testicle, left, orchiectomy:
    • Testicular parenchyma with hemorrhagic infarction, consistent with testicular torsion
Differential diagnosis
  • Systemic vasculitides:
    • Can present in 1 of 2 patterns
      • Polyarteritis nodosa pattern
        • Fibrinoid necrosis of medium sized arteries
        • Dense transmural neutrophilic infiltrates
        • Testicular infarct and hemorrhage
      • Granulomatosis with polyangiitis pattern
        • Destructive granulomatous vascular inflammation
  • Germ cell neoplasia in situ (GCNIS):
    • Presence of undifferentiated germ cells within seminiferous tubules; the cells possess
      • Large, hyperchromatic nuclei, with 1 - 2 prominent nucleoli
      • Clear cytoplasm
    • Spermatogenesis is usually absent
    • Thickened peritubular basement membrane
    • OCT 3/4 positive
  • Regressed germ cell tumor:
    • Vascularized fibrotic scar with inflammation
    • Surrounding hyalinized or atrophic seminiferous tubules
    • Psammomatous calcifications
    • Residual germ cell neoplasm or GCNIS may be present
  • Choriocarcinoma:
    • Hemorrhage and necrosis surrounded by cytotrophoblasts and syncytiotrophoblasts
  • Lymphoma:
    • Clonal population of lymphoid cells
  • Reference: Am J Surg Pathol 2014;38:34
Board review style question #1

A 14 year old boy presents to emergency room with severe left sided testicular pain associated with nausea and vomiting. A Doppler ultrasound shows loss in blood flow in the affected testis. Subsequent orchiectomy reveals red-tan, smooth surfaced testis, with microscopic appearance as seen in the image above. Which of the following can be seen in this condition?

  1. Clonal population of lymphoid cells
  2. Elevated alpha fetoprotein (AFP) levels
  3. Hemorrhage surrounded by trophoblastic cells
  4. Initial preservation of germ cells
  5. Undifferentiated malignant cells within seminiferous tubules
Board review style answer #1
D. Initial preservation of germ cells. The patient's presentation, including imaging and pathology, is diagnostic of testicular torsion. Testicular infarct following torsion is initially associated with conservation of germ cells; however, as congestion and hemorrhage increase, these cells slough into the lumens of seminiferous tubules and ultimately the entire tubule undergoes necrosis. Answer C is incorrect because hemorrhage surrounded by trophoblastic cells is typically observed in choriocarcinoma. Answer E is incorrect because germ cell neoplasia in situ (GCNIS) is associated with undifferentiated malignant cells within seminiferous tubules. Answer A is incorrect because clonal population of lymphocytes is seen in lymphoma. Answer B is incorrect because although AFP can be elevated in several conditions, including yolk sac tumor and teratoma, it is usually not elevated in torsion.

Comment Here

Reference: Torsion
Board review style question #2
Which of the following is the most common cause of testicular torsion?

  1. Bell clapper deformity
  2. Choriocarcinoma
  3. Cryptorchidism
  4. Mumps orchitis
  5. Varicocele
Board review style answer #2
A. Bell clapper deformity is the most frequent cause of testicular torsion. Tunica vaginalis is normally attached to the testis posterolaterally. In this anomaly, the attachment is abnormally high, which allows the spermatic cord to twist itself within the tunica vaginalis, resulting in testicular torsion. Answer C is incorrect because cryptorchidism may also be associated with torsion but not as frequently as bell clapper deformity. Answers B, D and E are incorrect because varicocele, mumps orchitis and choriocarcinoma are typically not associated with testicular torsion.

Comment Here

Reference: Torsion

Varicocele
Definition / general
  • Mass of tortuous and dilated veins of pampiniform plexus and internal testicular vein of spermatic cord
  • Occurs posterior and superior to the testis, may extend into the inguinal ring
Essential features
  • Probably due to incompetent valves of left internal spermatic vein as it empties into renal vein
  • 90% on left
  • 10% bilateral
  • Right internal spermatic vein is less often involved since it empties directly into the inferior vena cava and tends not to have incompetent valves
Epidemiology
Sites
  • Right sided varicocele is unusual; isolated right sided varicocele is associated with situs inversus, venous thrombosis or inferior vena cava compression from space occupying lesion such as soft tissue sarcoma; less commonly paraganglioma (Ann Saudi Med 2016;36:148)
  • It suggests the presence of a vascular abnormality (Urology 2015;85:e39)
Etiology
  • Most cases are idiopathic
  • Renal tumor may invade the left renal vein and block the drainage of the spermatic vein
  • Possible association with maternal exposure to diethylstilbestrol (Am J Obstet Gynecol 1981;140:186)
  • Rare association with sertoliform cystadenoma (Am J Surg Pathol 2018;42:141)
  • Kidney donation usually involves a left kidney; a study showed left pampiniform venous plexus diameter increased after kidney donation in men, suggesting that this can predispose to varicocele (Transplant Proc 2009;41:2738)
  • Can be caused by May-Thurner syndrome, which is reflux in the left spermatic vein caused by iliac vein compression between the right iliac artery and the spine [Ann Vasc Surg 2017;42:305)
  • Certain genetic and epigenetic changes are associated with varicocele (J Assist Reprod Genet 2017;34:839)
Clinical features
  • Often associated with infertility; after treatment, 40 - 55% recover fertility
  • Testicular pain may be associated with sexual activity
  • If longstanding, testicular atrophy and infertility of the affected testis may result
Diagnosis
  • Based on palpation while patient is at rest compared to during Valsalva
  • Graded based on physical exam findings:
    • Grade 1: palpable only during Valsalva
    • Grade 2: palpable at rest but not visible
    • Grade 3: visible and palpable at rest
  • Ultrasound if physical exam is inconclusive (J Ultrason 2016;16:359)
Laboratory
  • May decrease Leydig cell function, causing a lower serum testosterone, which can be improved after varicocelectomy (Urology 2013;81:1213)
Radiology description
  • Ultrasound can demonstrate vessel size, reversal of blood flow and testicular volume
Radiology images

Images hosted on other servers:

Left sided grade 3 varicocele

Color Doppler ultrasonography of varicocele

Case reports
Treatment
  • Varicocelectomy
  • Some patients have improved fertility after microsurgery, perhaps because DNA fragmentation is reduced / DNA integrity is improved (Syst Biol Reprod Med 2012;58:274)
  • Ligation or occlusion of left spermatic vein at the internal inguinal ring
  • Motile sperm were found in 11 of 19 patients after varicocele repair; testicular histology was also improved (Biomed Res Int. 2015;2015:709452)
Gross description
  • Thin walled cystically dilated membrane
Microscopic (histologic) description
  • From inside to outside, changes include (Ultrastruct Pathol 2010;34:260)
    • Narrowing or obliteration of the vein lumens
    • Segmental obliteration and occasional thrombi
    • Internal elastic lamina fragmentation and invagination of intima
    • Variable thickening of vein wall; the media contains hypertrophied smooth muscle fibers and deposition of collagen bundles (Ultrastruct Pathol 2012;36:201)
  • If a testis is included: decreased spermatogenesis in tubules with germ cell degeneration and increased Leydig cells
  • Adolescent varicocele: pathologic changes found at or soon after puberty, consisting of tubular sclerosis, premature germ cell sloughing, small vessel sclerosis and variable hypospermatogenesis (Am J Clin Pathol 1988;89:321)
Microscopic (histologic) images

Contributed by Kenneth A. Iczkowski, M.D.

Orchiectomy specimen in a 30 year old with atrophic testis and varicocele

Positive stains
  • CD31 (stains endothelial layer)
Negative stains
Electron microscopy description
Differential diagnosis
  • Hydrocele is lined by tunica vaginalis mesothelium (calretinin+, CD31-) whereas varicocele is lined by endothelium (CD31+, calretinin-)
Board review style question #1
Which of the following is not associated with varicocele?

  1. Destruction of vessel's external elastic lamina
  2. Endothelial cell damage
  3. Narrowing or obliteration of the lumen of the vein
  4. Thickening of the media by collagen deposition
Board review style answer #1
A. Destruction of vessel's external elastic lamina. It is the internal elastic lamina that is destroyed

Comment Here

Reference: Varicocele
Board review style question #2

In this orchiectomy specimen in a 30 year old with atrophic testis and varicocele, what is illustrated?

  1. Hematoma on the left and vas deferens on the right
  2. Hydrocele on the left and testis tubules on the right
  3. Spermatocele on the left and rete testis on the right
  4. Varicocele on the left and rete testis on the right
Board review style answer #2
D. Varicocele on the left and rete testis on the right

Comment Here

Reference: Varicocele

Vasculitis
Definition / general
Clinical features
  • Can present with acute onset of pain and swelling, presumably related to intratesticular hemorrhage (Ir Med J 2006;99:27)
  • Systemic disease is frequently already present at the time of initial presentation of testicular vasculitis or develops shortly thereafter
  • Some cases, however, remain isolated to the testes with no evidence of progression to systemic disease (J Clin Pathol 1994;47:1121)
  • It is, therefore, unclear whether isolated testicular vasculitis represents an early site of involvement by systemic disease or a separate entity (World J Surg Oncol 2011;9:63)
  • Only several cases of isolated testicular vasculitis were reported in the literature and a presentation suggestive of a testicular neoplasm is even less common (J Clin Pathol 1994;47:1121)
  • Since testicular biopsy is contraindicated when malignancy is suspected, the diagnosis is usually reported postorchiectomy in these cases (ScientificWorldJournal 2010;10:1915, Urology 2003;61:1035)
  • Biopsy most likely diagnostic in patients with testicular symptoms (pain, enlargement or shrinking)
  • Wedge biopsy should contain capsule, tunica vasculosa and testicular parenchyma
Radiology images

Case #328

Testicular heterogeneous hypoechoic lesion

Case reports
  • 34 year old man with hypoechoic testicular lesion (Case #328)
Treatment
  • Treatment options are still debated and frequently hinge on individual factors which need to be carefully considered in each patient
  • Immunosuppressive therapy has been administered to some of the patients
  • In other cases, careful follow up to exclude progression to systemic disease was preferred (J Clin Pathol 1994;47:1121)
  • Some authors have even postulated that the excision of the affected organ might be curative, obviating the need for pharmacologic therapy (ScientificWorldJournal 2010;10:1915)
  • Close follow up, including serology, is recommended since the risk of progression to systemic disease is unknown (J Clin Pathol 1994;47:1121)
Gross description
Gross images

Case #328
Microscopic (histologic) description
  • Focal hemorrhage with early organization and small arteries around the area of hemorrhage (Case of the Week #328)
Microscopic (histologic) images

Case #328


Differential diagnosis
  • Although uncommon, isolated necrotizing testicular vasculitis remains in the differential diagnosis of hypovascular testicular lesions; long term follow up and additional research is needed in order to further elucidate the pathogenesis of isolated organ vasculitis and the subsequent risk of disease progression

Vasitis nodosa
Definition / general
  • Granulomatous condition of vas deferens that resembles spermatic granuloma of epididymis
  • Usually post vasectomy or herniorrhaphy
  • Occasionally associated with recanalization
Microscopic (histologic) description
  • Proliferating ductules and dilated tubules containing spermatozoa in wall of vas deferens
  • Also associated with small bundles of hyperplastic smooth muscle
  • Vas deferens may show irregular thickening
  • Ductules may connect with vas deferens (like diverticula or salpingitis isthmica nodosa but with inflammation and less smooth muscle hypertrophy)
  • May see perineural or vascular invasion by proliferating ductules
Microscopic (histologic) images

Images hosted on other servers:

Vasitis nodosa

Differential diagnosis

Well differentiated papillary mesothelial tumor
Definition / general
  • Well differentiated papillary mesothelial tumor is a neoplasm of uncertain malignant potential that originates from the mesothelium of the pleura, peritoneum or tunica vaginalis
Essential features
  • Mesothelial tumor without significant cytologic atypia (Ann Diagn Pathol 2019;38:43, Ann Surg Oncol 2019;26:852)
  • Composed entirely of papillary structures and tubules lined by a single layer of bland cuboidal cells with no identifiable invasion
  • Positive staining for mesothelial markers (calretinin, WT1)
  • BAP1 and MTAP immunohistochemical staining should be retained
Terminology
Epidemiology
  • Tumors arising from tunica vaginalis are rare and comprise < 1% of mesotheliomas (Mod Pathol 2022;35:1327)
  • Age range is 18 - 70 years
Sites
Pathophysiology
  • Unknown
Etiology
  • Unknown
Clinical features
Diagnosis
  • Histologic diagnosis is required
Radiology description
Prognostic factors
  • This tumor is likely benign in nature but is often morphologically identical to mesothelioma in situ, which may explain the rare reports of recurrence as mesothelioma (Am J Surg Pathol 2023;47:611)
Case reports
Treatment
  • Complete excision
Gross description
  • Peritoneal surfaces with small nodules or papillary excrescences, single to innumerable (Mod Pathol 2022;35:1327)
Frozen section description
  • These tumors are often found incidentally during other procedures, prompting frozen section evaluation
  • Expanded papillary structures with edema or myxoid change, lined by a single layer of bland mesothelial cells
  • Invasion should be excluded
  • Reference: Int J Surg Pathol 2021;29:844
Microscopic (histologic) description
  • Papillary structures with expansile cores showing myxoid features
  • Papillary structures are lined by a bland simple layer of flattened to cuboidal cells
  • Fibrovascular cores generally lack inflammation and psammoma bodies
  • Occasional gland-like or tubulocystic areas may be seen
  • There should be a lack of stromal invasion, significant atypia and mitotic activity (Mod Pathol 2022;35:1327)
Microscopic (histologic) images

Contributed by Aida Valencia, M.D. and Jennifer Gordetsky, M.D.
Numerous fibrovascular cores Numerous fibrovascular cores

Numerous fibrovascular cores

Background macrophages

Background macrophages

Broad papillae Broad papillae

Broad papillae


Papillae with myxoid stroma Papillae with myxoid stroma

Papillae with myxoid stroma

Nonpsammomatous calcifications Nonpsammomatous calcifications

Nonpsammomatous calcifications

Calretinin

Calretinin


WT1 immunostain WT1 immunostain

WT1

D2-40

D2-40

Malignant mesothelioma Mesothelioma

Mesothelioma

Virtual slides

Images hosted on other servers:

Diagnostic features

Retained BAP1 expression

Retained MTAP expression

Cytology description
  • Clusters of uniform mesothelial cells without atypia
Positive stains
Negative stains
Molecular / cytogenetics description
Molecular / cytogenetics images

Images hosted on other servers:

TRAF7 and CDC42 somatic mutations

Sample pathology report
  • Scrotum, hydrocelectomy:
    • Well differentiated mesothelial tumor
Differential diagnosis
Board review style question #1
Well differentiated papillary mesothelial tumor Well differentiated papillary mesothelial tumor Well differentiated papillary mesothelial tumor


The following tumor is incidentally found on the peritoneum during a cholecystectomy. This tumor should show which immunophenotype?

  1. BAP1 loss, calretinin+, WT1+
  2. BAP1 retained, MTAP retained, calretinin+, WT1+
  3. Cytokeratin 5/6-, D2-40+
  4. p53 null, AE1 / AE3+, PAX8+
Board review style answer #1
B. BAP1 retained, MTAP retained, calretinin+, WT1+. These images show a well differentiated mesothelial tumor, which should have retained BAP1 and MTAP. Mesothelial markers will be positive. Answer A is incorrect because while a well differentiated mesothelial tumor will express mesothelial markers, BAP1 should be retained. Answer C is incorrect because mesothelial tumors will express both CK5/6 and D2-40. Answer D is incorrect because this immunophenotype is more in keeping with an ovarian type serous tumor.

Comment Here

Reference: Well differentiated papillary mesothelial tumor
Board review style question #2
Well differentiated papillary mesothelial tumor Well differentiated papillary mesothelial tumor Well differentiated papillary mesothelial tumor Well differentiated papillary mesothelial tumor


A patient presents with scrotal swelling that has been persistent over the last few years. An orchiectomy is done. Which of the following immunophenotypes would favor a mesothelioma over a well differentiated papillary mesothelial tumor?

  1. BAP1 loss, calretinin+, WT1+
  2. BAP1 retained, MTAP retained, calretinin+, WT1+
  3. Cytokeratin 5/6-, D2-40+
  4. p53 null, AE1 / AE3+, PAX8+
Board review style answer #2
A. BAP1 loss, calretinin+, WT1+. These images show a tumor with significant cytologic atypia and invasion into the testis. This is a mesothelioma of the tunica vaginalis. Mesotheliomas will express mesothelial markers and have BAP1 loss. Answer B is incorrect because this immunophenotype is in keeping with a benign mesothelial lesion. Answer C is incorrect because mesothelial tumors will express both CK5/6 and D2-40. Answer D is incorrect because this immunophenotype is more in keeping with an ovarian type serous tumor.

Comment Here

Reference: Well differentiated papillary mesothelial tumor

WHO classification
Definition / general
  • WHO classification of tumors of the urinary and male genital organs
  • Currently on 5th edition, published in 2022
Major updates
  • Germ cell tumors derived from germ cell neoplasia in situ:
    • Teratoma with primitive neuroectodermal tumor (PNET) is now called teratoma with embryonic type neuroendocrine differentiation
  • Sex cord stromal tumors:
    • Signet ring pattern of Sertoli cell tumor is now listed as a separate tumor called signet ring stromal tumor
    • Myoid gonadal stromal tumor changed from a provisional entity to a confirmed entity
  • Paratesticular mesothelial tumors:
    • Well differentiated papillary mesothelioma was renamed well differentiated papillary mesothelial tumor
WHO (2022) (ICD-O morphology terms if different)
Germ cell tumors derived from germ cell neoplasia in situ
Germ cell tumors unrelated to germ cell neoplasia in situ
Sex cord stromal tumors of the testis
Ovarian type tumors of the collecting ducts and rete testis
Tumors of the collecting duct and rete testis
  • Adenoma of the collecting ducts and rete testis (adenoma)
  • Adenocarcinoma of the collecting ducts and rete testis (adenocarcinoma)

Paratesticular mesothelial tumors
Tumors of the epididymis
Metastatic tumors
Board review style question #1

Spermatocytic tumor, shown in the image above, is an example of which of the following categories of testicular tumors?

  1. Germ cell tumors derived from germ cell neoplasia in situ
  2. Germ cell tumors unrelated to germ cell neoplasia in situ
  3. Sex cord stromal tumors
  4. Tumor containing both germ cell and sex cord stromal elements
Board review style answer #1
B. Germ cell tumors unrelated to germ cell neoplasia in situ. Spermatocytic tumor is a germ cell tumor that is not derived from germ cell neoplasia in situ. Most germ cell tumors such as seminoma are thought to be derived from the precursor lesion germ cell neoplasia in situ. Spermatocytic tumor is not a sex cord stromal tumor and does not have sex cord stromal elements.

Comment Here

Reference: Testis & paratestis - WHO classification
Board review style question #2

Seminoma, shown in the image above, is an example of which of the following categories of testicular tumors?

  1. Germ cell tumors derived from germ cell neoplasia in situ
  2. Germ cell tumors unrelated to germ cell neoplasia in situ
  3. Sex cord stromal tumors
  4. Tumor containing both germ cell and sex cord stromal elements
Board review style answer #2
A. Germ cell tumors derived from germ cell neoplasia in situ. Seminoma is thought to be derived from the precursor lesion germ cell neoplasia in situ. It is not a sex cord stromal tumor and does not contain sex cord stromal elements.

Comment Here

Reference: Testis & paratestis - WHO classification

Yolk sac tumor
Definition / general
  • Germ cell neoplasm composed of cells / structures reminiscent of embryonic / fetal yolk sac, allantois and extraembryonal mesenchyme
Essential features
  • 2 biologically distinct entities:
    • Postpubertal type (germ cell neoplasia in situ [GCNIS] related and usually as part of mixed tumor)
    • Prepubertal type (unrelated to germ cell neoplasia in situ and usually pure)
  • Multiplicity of histological features with various mimickers
  • Most overlooked germ cell tumor subtype
  • Elevated serum alpha fetoprotein (AFP) should raise the suspicion for a yolk sac component
Terminology
  • First described / studied by Gunnar Teilum and later by Aleksander Talerman (Am J Surg Pathol 1981;5:719)
  • Complex nomenclature over time, reflecting the remarkable heterogeneity: primitive endodermal tumor, yolk sac carcinoma, endodermal sinus tumor, mesoblastoma vitellinum, orchioblastoma, polyvesicular vitelline tumor, adenocarcinoma of the infant testis, extraembryonic mesoblastoma; malignant endothelioma of perithelioma type (currently less used and not recommended by WHO 2016) (Int J Surg Pathol 2014;22:677, Histopathology 2012;60:1023)
ICD coding
  • ICD-O:
    • 9071/3 - yolk sac tumor, postpubertal type
    • 9071/3 - yolk sac tumor, prepubertal type
Epidemiology
  • Postpubertal type:
    • Almost always part of mixed tumors (44% of nonseminomas); pure form exceedingly rare (0.6% of testicular germ cell tumors)
    • Epidemiological trends common to all germ cell neoplasia in situ related tumors (15 - 40 years, rising incidence, environmental, genetic and epigenetic triggers) (Int J Mol Sci 2019;20:258)
  • Prepubertal type:
    • Rare (2 - 3 cases/year/1 million) and usually pure (mixed forms with teratoma are the exception)
    • Children < 6 years (median age 16 - 20 months)
    • Does not follow epidemiological trends / associations of postpubertal type tumors
Sites
  • Testis
  • Also found in ovary, extragonadal (along midline of the body)
Pathophysiology
  • Postpubertal type:
    • Derived from germ cell neoplasia in situ (primordial germ cells / gonocytes arrested in maturation → germ cell neoplasia in situ → reprogramming → extraembryonal differentiation)
    • Type II germ cell tumor (Nat Rev Cancer 2019;19:522)
  • Prepubertal type:
    • Not derived from germ cell neoplasia in situ
    • Hypothesis: derived from teratoma
    • Type I germ cell tumor
Etiology
  • Postpubertal type:
    • Environmental factors: testicular dysgenesis syndrome, subfertility
    • Genetic factors: single nucleotide polymorphism (SNP) variants (KITLG, SPRY4, DMRT1)
  • Prepubertal type:
    • Unknown
  • References: Int J Mol Sci 2019;20:258, Nat Rev Cancer 2019;19:522
Clinical features
  • Unilateral testicular mass, often painless (World J Surg Oncol 2014;12:400)
  • Postpubertal type:
    • More aggressive (compared to prepubertal): around 40% stage I disease; metastatic rate of 33%; stage III patients with yolk sac elements display poorer prognosis; increased prevalence in late recurrences (chemoresistance)
  • Prepubertal type:
    • Less aggressive (compared to postpubertal): 80% stage I disease; of these 6% relapse on surveillance only
    • Higher tendency to bypass retroperitoneal lymph nodes and spread hematogenously (compared to postpubertal)
Diagnosis
Laboratory
  • Elevated alpha fetoprotein in 95 - 98% of the cases (may also be elevated in 20 - 25% of teratomas via hepatoid differentiation, mucinous glands) (Open Access J Urol 2011;4:1)
  • Care in interpretation in neonates (physiologically elevated alpha fetoprotein until 6 months)
Radiology description
  • Ultrasound: heterogenous echogenicity, frequently with cystic spaces, ill defined margins and calcifications (Radiographics 2017;37:1085)
Prognostic factors
  • Classical factors for testicular germ cell tumors in general, including staging and International Germ Cell Cancer Collaborative Group grouping
  • In postpubertal mixed tumors, higher percentage of yolk sac tumor is associated with lower stage (Am J Clin Pathol 2016;145:341)
  • For stage I prepubertal type tumors: size > 4.5 cm, invasion of rete testis / epididymis and necrosis (Am J Surg Pathol 2015;39:1121)
Case reports
Treatment
  • Orchiectomy, with or without adjuvant (cisplatin based) chemotherapy
Gross description
  • Postpubertal type:
    • Poorly circumscribed, nonencapsulated, predominantly solid
    • Gray to white to yellow to tan, gelatinous surface
    • Hemorrhage, necrosis and cystic areas are common
  • Prepubertal type:
    • More homogeneous, gray to yellow, solid or mucoid, bulging and lobulated
    • Hemorrhage and necrosis are uncommon
  • References: Surg Pathol Clin 2015;8:687, Hum Pathol 2018;82:113
Gross images

Contributed by Debra L. Zynger, M.D.
Mixed germ cell tumor, postpubertal with varying % of yolk sac tumor Mixed germ cell tumor, postpubertal with varying % of yolk sac tumor Mixed germ cell tumor, postpubertal with varying % of yolk sac tumor

Mixed germ cell tumor, postpubertal with varying % of yolk sac tumor

Microscopic (histologic) description
  • Most overlooked subtypes of germ cell tumors (dedicated sampling is key) (Pathology 2018;50:88, Hum Pathol 2018;82:113)
  • Remarkable heterogeneity, multiple patterns, often combined:
    • Microcystic / reticular pattern:
      • Most common
      • Anastomosing cords of flattened cells forming a honeycomb / spider web meshwork, forming spaces enclosing mucoid / basophilic material
    • Macrocystic pattern:
      • Coalescence of smaller cystic spaces described in the microcystic pattern
    • Myxomatous pattern:
      • Resembles the microcystic pattern
      • Sparse dispersed spindled / stellate cells in a myxoid background that often contains a prominent capillary network (also called angioblastic pattern)
      • Focal differentiation into cartilage or skeletal muscle may be rarely seen and should not elicit diagnosis of teratoma
    • Sarcomatoid / spindle cell pattern:
      • Progression of the myxomatous pattern (increase in number / density of spindled / stellate cells retaining cytokeratin positivity)
      • Some cases regarded as somatic malignancy in the form of sarcoma may actually represent sarcomatoid pattern of yolk sac tumor (Am J Surg Pathol 2014;38:1396)
    • Solid pattern:
      • Sheets of polygonal clear to amphophilic cells form nodules
      • Distinct cell membrane (like in seminoma)
    • Glandular / alveolar pattern:
      • Glands or tubular structures, sometimes with subnuclear vacuoles resembling secretory endometrium or resembling intestinal epithelium
    • Endodermal sinus / perivascular pattern:
      • Predominance of Schiller-Duval bodies (see below)
    • Hepatoid pattern:
      • Sheets, solid nests or trabeculae of polygonal eosinophilic cells, with large nuclei and prominent nucleoli, resembling hepatocytes
      • Strongly alpha fetoprotein positive
    • Papillary pattern:
      • Papillary formations with or without fibrovascular cores projecting into cysts
      • Often hobnail morphology, high nuclear to cytoplasmic ratio
      • Exfoliated cells forming clusters may be seen in the vicinity
    • Parietal pattern:
      • Abundant, dense, eosinophilic basement membrane material surrounding tumor cells
    • Polyvesicular vitelline pattern:
      • Always occurs with other patterns
      • Vesicles, sometimes with constrictions (forming figure 8 shaped structures), in an edematous to fibrous stroma
  • Cytological atypia is variable across patterns; it is most significant in solid, sarcomatoid and glandular patterns
  • 50% of tumors contain Schiller-Duval bodies (papillary structures within cystic spaces, lined by cuboidal to columnar cells with a distinct central vessel)
  • Characteristic (but not pathognomonic) hyaline globules (within and outside cytoplasm), refractile and eosinophilic, which are alpha-1 antitrypsin+ and PAS diastase+
Microscopic (histologic) images

Contributed by João Lobo, M.D. and Rui Henrique, M.D., Ph.D.
Microcystic pattern Microcystic pattern

Microcystic pattern

Hyaline globules Hyaline globules

Hyaline globules

Glandular pattern Glandular pattern

Glandular pattern


Schiller-Duval bodies Schiller-Duval bodies Schiller-Duval bodies

Schiller-Duval bodies

Metastasis Metastasis

Metastasis

Virtual slides

Images hosted on other servers:

Yolk sac tumor

Cytology description
  • Variably sized cell clusters, occasional glomeruloid structures, metachromatic basement membrane material, large cells with high nuclear to cytoplasmic ratio, mucoid background, PAS positive hyaline globules (J Cytol 2015;32:53)
Positive stains
Negative stains
Electron microscopy description
  • Epithelial cells with tight junctional complexes, apical microvilli, extracellular deposits of basal lamina, glycogen
Molecular / cytogenetics description
  • Postpubertal type:
    • Gain of 12p (usually isochromosome 12p)
    • Erased genomic imprinting
  • Prepubertal type:
    • Multiple gains (1q, 20q, 22) and losses (1p, 4, 6q) but no 12p gain
    • Biparental genomic imprinting
  • References: Int J Mol Sci 2019;20:258, Nat Rev Cancer 2019;19:522
Sample pathology report
  • Right testicle, right radical orchiectomy:
    • Mixed germ cell tumor, postpubertal type, 40% teratoma, 30% embryonal carcinoma, 15% yolk sac tumor and 15% seminoma (see synoptic report)
Differential diagnosis
Board review style question #1

A 2 year old boy is submitted to orchiectomy. In the figure you may appreciate a histological section of the lesion. Which sentence is true?

  1. Most likely the tumor has gain of 12p
  2. Most likely this tumor is present in its pure form and not mixed with other subtypes
  3. Negative AFP excludes a yolk sac tumor
  4. OCT 3/4 staining will be positive in this tumor
  5. Schiller-Duval bodies are present and are definitive proof of a postpubertal type yolk sac tumor
Board review style answer #1
B. The picture represents a prepubertal type yolk sac tumor. This entity does not show 12p gain (contrarily to postpubertal type tumors of the testis). Schiller-Duval bodies are present but may be seen (or not) in both prepubertal type and postpubertal type yolk sac tumors. AFP is variably positive (but many times negative) in yolk sac tumors and OCT 3/4 is invariably negative. Prepubertal type yolk sac tumor is almost always seen in its pure form (contrarily to postpubertal type tumors, which are frequently mixed).

Comment Here

Reference: Yolk sac tumor
Board review style question #2
Which of the following is true regarding yolk sac tumor of the testis?

  1. Differential diagnosis with seminoma is particularly problematic in the microcystic pattern
  2. Glypican 3 is the most specific marker of this tumor subtype
  3. Can be germ cell neoplasia related or unrelated depending on pubertal status
  4. Only occurs in adolescents
  5. Serum AFP levels are not relevant for the diagnosis
Board review style answer #2
C. Yolk sac tumor of the testis can be germ cell neoplasia in situ related (postpubertal type) or germ cell neoplasia in situ unrelated (prepubertal type), showing a bimodal epidemiology (young adults and young children, respectively). Glypican 3 is sensitive but not a specific marker of this tumor type. Differential diagnosis with seminoma occurs when a solid pattern is extensive. There is a strong correlation between the finding of elevated serum AFP and presence of yolk sac tumor elements.

Comment Here

Reference: Yolk sac tumor
Back to top
Recent Testis & paratestis Pathology books

Amin: 2022

Behnke: 2018

Cheng: 2019

Colecchia : 2016

IARC: 2022

Manuel: 2019

Nistal: 2017

Ulbright: 2014

Ulbright: 2022

VandenBussche: 2022

Wobker: 2021

Yang: 2020

Zhou: 2022

Zhou: 2022



Find related Pathology books: cytopathology, GU/adrenal
Image 01 Image 02